大学入試の数学の問題を解くゲイ2023
■ このスレッドは過去ログ倉庫に格納されています
0003陽気な名無しさん2023/02/10(金) 17:00:41.77
>>1
ID無しのスレを乱立させてる奴は何なの?
https://medaka.5ch.net/test/read.cgi/gaysaloon/1632209309/

※IDなしスレ乱立「K-POPホモガイジ」のスレに注意
https://medaka.5ch.net/test/read.cgi/gaysaloon/1635557165/

その神経がわからん@同サロ
https://medaka.5ch.net/test/read.cgi/gaysaloon/1646803417/

IDなしスレ乱立「KPOPホモガイジ」のスレに注意
https://medaka.5ch.net/test/read.cgi/gaysaloon/1649686229/
0004陽気な名無しさん2023/02/13(月) 15:19:25.670
>>1
線分OPの長さが有理数tであると仮定して矛盾を出すわ。
2r^2 + 3s^2 = t^2
となるけど、r, s, t を分数で表した分母の最小公倍数の2乗を両辺にかけると、ある整数R, S, Tがあって
2R^2 + 3S^2 = T^2
になるわ。R, S, Tの最大公約数の2乗で両辺を割ると、互いに素な整数x, y, zがあって
2x^2 + 3y^2 = z^2
になるわ。
もしxが3で割り切れると、左辺は3で割り切れるから、右辺のzも3で割り切れることになって
右辺が3^2 = 9で割り切れることになるから、yも3で割り切れなければいけないけど
これはx, y, zが互いに素であることに反するから、xは3で割り切れない。
したがって x ≡ 1, 2, 4, 5 (mod 6)。x^2 ≡ 1, 4 (mod 6) なので 2x^2 ≡ 2 (mod 6)。
同様にyは2で割り切れないから y ≡ 1, 3, 5 (mod 6)。y^2 ≡ 1, 3 (mod 6) なので 3y^2 ≡ 3 (mod 6)。
したがって z^2 = 2x^2 + 3y^2 ≡ 2 + 3 ≡ 5 (mod 6) となるが、2乗がmod 6で5になる整数はないからこれは矛盾。
0005陽気な名無しさん2023/02/14(火) 07:05:14.070St.V
>>4
よさそうね
0006陽気な名無しさん2023/02/14(火) 08:29:23.360St.V
xは3で割り切れない
したがってx^2≡1(mod3)
したがってz^2=2x^2+3y^2≡2(mod3)
しかしmod3で平方が2と合同な整数は存在しない
0007陽気な名無しさん2023/02/14(火) 08:32:13.730St.V
>>6
よさそうね
0008陽気な名無しさん2023/02/14(火) 12:54:20.480St.V
>>6
あら、yが2で割り切れないとか、そういうこといちいち考えなくて良いのね
姐さんさすがだわ
これは今年の入試問題なのかしら?
0010陽気な名無しさん2023/02/14(火) 22:10:00.950
慈恵医大の入試問題って難しいのね
時間内に解ける受験生そんなにいないんじゃないかしら?
医学と全く関係ない能力を要求するのが意味不明ね
0015陽気な名無しさん2023/02/24(金) 22:36:38.970
急に頭の中に
辺の長さがすべて有理数の5角形が半径1の円に内接することはあるか?
という疑問が浮かんできたの
明日どこかの大学で出るかもしれないわ
0016陽気な名無しさん2023/02/24(金) 23:40:10.500
その疑問って、5角形に限定する必要あるかしら?
辺の長さがすべて有理数のn角形が半径1の円に内接することがあるnの値を全て求めよ、
とか拡張できるわね。
少なくともn=6のときは正6角形が全ての辺が1だから成り立つでしょ。
ほかの時はどう?
そもそも最小のn=3のときはどうなのかしら?
0017陽気な名無しさん2023/02/25(土) 00:24:03.020
とても興味深い疑問ね
あたしもなんでいきなり五角形の話なのかしらって思って、まず三角形と四角形の場合を考えてみたわ
a, b, c を a^2 + b^2 = c^2 を満足するピタゴラス数として
直角を挟む二辺が 2a/c と 2b/c の直角三角形を考えると、斜辺の長さが2で、半径1の円に内接するわ
そして、そういう直角三角形をふたつ用意して斜辺のところで張り合わせれば、半径1の円に内接する四角形ができるわね
ここで気になったんだけど、直角三角形以外で条件を満たす三角形はあるのかしら?

>>16
なるほど、正六角形だとうまくいくわね。正六角形以外の六角形で条件を満たすものはあるのかしら?
0022陽気な名無しさん2023/02/25(土) 16:11:38.250
>>17
自己レスだけど、三角形についてはたぶん解明できたわ。
点Oを中心とする半径1の円に△ABCが内接しているとするわ。α = ⦣A, β = ⦣B, γ = ⦣C とおくわ。
OからA, B, C に直線を引くと、3つの二等辺三角形△OAB, △OBC, △OCAができるわ。
円周角の定理から ⦣AOB = 2γ(または、これが鈍角の場合は△OABの内部から見ると 2π− 2γ)ね。
OからABに垂線を下ろして直角三角形をふたつに分けると、AB = 2sinγであることがわかるわ。
同様に BC = 2sinα, CA = 2sinβ となるわね。(三角形の外接円だから単に正弦定理からと言っても良いけど)
⦣AOB + ⦣BOC + ⦣COA = 2πだから、問題はこう言い直せるの。

α, β, γ > 0, α+β+γ = π で sinα, sinβ, sinγがすべて有理数となるものはあるか?

これを解くわ。sinαとsinβが有理数だと仮定するわ。
sinγ= sin(π−(α+β)) = sin(α+β) = sinαcosβ + cosαsinβ
もしcosαとcosβがともに有理数なら、sinγも有理数になるわね。
もしcosαとcosβのうち片方が有理数でもう片方が無理数なら、sinγは無理数になるわ。
最後にcosαとcosβがともに無理数の場合、sinαとsinβは有理数だから
0でない有理数p, qと平方数でない正の整数m, nがあってcosα = p√m, cosβ= q√n となるわ。
√mと√nが一次独立なら、sinγ= p sinα√m + q sinβ√n は無理数だし
√mと√nが一次従属なら、sinγ= r√mの形になるけど、これも無理数ね。
(以上の議論で sinα, sinβ, sinγ ≠ 0 であることに注意)
まとめると

sinα, sinβ, sinγがすべて有理数
⟺ sinα, sinβ, cosα, cosβがすべて有理数
⟺ αとβはピタゴラスの三角形に現れる角

ここで面白いことに気づいたの。この議論はα, β, γに関して対象だから
αとβがピタゴラスの三角形に現れる角なら、γもピタゴラスの三角形に現れる角であることになるわね!
結論を言うと、直角三角形でなくても、ピタゴラスの三角形に現れる角だけでできた三角形なら、条件を満たすってことね。

同様に考えると、四角形の場合は
α, β, γ, δ > 0, α+β+γ+δ = π で sinα, sinβ, sinγ, sinδがすべて有理数となるものはあるか?

五角形の場合は
α, β, γ, δ, ε > 0, α+β+γ+δ+ε = π で sinα, sinβ, sinγ, sinδ, sinεがすべて有理数となるものはあるか?

六角形の場合は
α, β, γ, δ, ε, ζ > 0, α+β+γ+δ+ε+ζ = π で sinα, sinβ, sinγ, sinδ, sinε, sinζがすべて有理数となるものはあるか?

のように言い直せるわね。
0027陽気な名無しさん2023/02/25(土) 21:41:38.750
ちょっと訂正。>>22のαやβは鈍角の可能性もあるから、正しくは

sinα, sinβ, sinγがすべて有理数
⟺ sinα, sinβ, cosα, cosβがすべて有理数
⟺ αとβはピタゴラスの三角形に現れる角か、それをπから引いたもの

ていうか気づいたの。
まずθ_1としてピタゴラスの三角形に現れる角を何か選ぶわ。φ_1 = π−θ_1 とおくと
sinφ_1 = sin(π−θ_1) = sinθ_1
cosφ_1 = cos(π−θ_1) = −cosθ_1
はともに有理数ね。
次に 0 < θ_2 < φ_1 となるピタゴラスの三角形に現れる角θ_2を選んでφ_2 = φ_1−θ_2とおくと
sinφ_2 = sin(φ_1−θ_2) = sinφ_1 cosθ_2 − cosφ_1 sinθ_2
cosφ_2 = cos(φ_1−θ_2) = cosφ_1 cosθ_2 + sinφ_1 sinθ_2
も有理数になるわ。
同様にφ_{k−1} まで作ったら、 0 < θ_k < φ_{k−1} となるピタゴラスの三角形に現れる角θ_k を選んで
φ_k = φ_{k−1} −θ_k とおくのよ。
すると θ_1, …, θ_k, φ_k は和がπでsinを取るとすべて有理数になるわ。

つまり、半径1の円の中心からピタゴラスの三角形に現れる角の2倍ずつずらして半径を描いていって
それと円周との交点たちを結んで多角形を作れば、条件を満たすものができるのよ。
じゃあ何角形までできるかだけど、それはピタゴラスの三角形に現れる角がどれだけ小さくなれるかという問題になるわ。
mを正の整数とすると
a = 2m+1, b = 2m(m+1), c = 2m^2+2m+1
はピタゴラス数だけど、a/c はmを大きくすればいくらでも0に近づけることができるから
いくらでもとんがったピタゴラスの三角形が存在するの。
だから>>16への答えは、「すべての n ≥ 3 に対して成り立つ」ね!

でも正六角形はこの方法で作れないから、これがすべてではないわね。
正六角形以外にもこの方法で作れない例はあるのかしら?
0028陽気な名無しさん2023/02/26(日) 07:37:52.800
>>17=22=27
あなた、前スレのうさぎさんじゃないかしら。
相変わらず冴えてるわね。
久しぶりに書き込まれてる議論追ったけど、
確かにそうね。
そうすると、次なる問題は、
半径1の円に内接する辺の長さがすべて有理数のn角形のうち、ピタゴラス三角形の角と一致しないことがあるnの値を全て求めよ、
になるかしら。
とりあえずn=3については否定的に解決してるわね。
だって全ての半径1の円に内接する辺の長さがすべて有理数の3角形はピタゴラス三角形であることが示されたから。

この問題、類似の疑問がどんどん出てくるわね。
辺の長さがすべて無理数のn角形が半径1の円に内接することがあるnの値を全て求めよ、とか
辺の長さがすべて超越数のn角形が半径1の円に内接することがあるnの値を全て求めよ、とか
辺の長さがすべてm次の代数的数(mは2以上の自然数)のn角形が半径1の円に内接することがあるnの値を全て求めよ、とか
きりがないわね。

それにしてもピタゴラス数って凄いわね。
いろんな所に関係してくるわね。
最初この問題見たときに何となく関係しそうだな、とは思ったけど、
見事に理論づけたあなた、さすがだわ。
0029陽気な名無しさん2023/02/26(日) 07:48:19.060
あかん、疑問が暴走しはじめたわ。
辺の長さがすべて有理数のn面体が半径1の球に内接することがあるnの値を全て求めよ、
なんて問題の次元を上げたらまた新しい問題になるわ。
次元まで一般化したらもはやどう手出ししたらいいのか見当つかないわ。
それぞれ無理数、超越数、代数的数にした問題も考えられるし、
解決どころか問題の存在を考えるだけで頭がパンクするわ。

>>15 さんは疑問をもつセンスが素晴らしいわね。
0030陽気な名無しさん2023/02/26(日) 08:01:34.610
みんな東大の6が難しいって言ってるけど
少なくとも(1)は暗算でできるわよね
0031陽気な名無しさん2023/02/26(日) 08:01:38.950
>>27
あれ?ちょっと変な気がしたわ。
和がπになるのって三角形だけよね。
だからn≧4だとその議論成り立たないのではないかしら?
一般には和は(n−2)πでしょ。
そうすると議論はどうなるかしら?
0032陽気な名無しさん2023/02/26(日) 08:11:33.070
あれ?え?
中心角にピタゴラス角の二倍を置くの?
なら正しいかしら。
なんか混乱してごめんなさいね。

中心角がピタゴラス角の二倍で書いていけば題意を満たすn角形ができる。
中心角がピタゴラス角の二倍ではないタイプで題意を満たすn角形が存在するnを求めるのが
さしあたり次の問題なのね。
n=3なら存在しないしn=6なら存在することはわかっていて、その他のnについてはどうか、なのね。

久しぶりに数学考えたら頭が整理されないまま書き込み連投してしまったわ。
申し訳ないわ。
0033陽気な名無しさん2023/02/27(月) 00:54:54.970
>>28
そうよ、usagiよ。そうね、いろいろバリエーション考えられるわね。
ちゃんとした議論じゃないけど、無理数とか超越数の場合はできそうな気がするわ。
有理数の集合は可算だけど、無理数の集合と超越数の集合は連続体の濃度を持つから
有理数よりはるかにいっぱいあるのよね。
辺がすべて有理数のn角形をちょっとずらせば辺がすべて無理数とかのができそうってのが直感ね。
>>27の式で sinθ_i, cosθ_i にいろんな数を入れれば、有理数以外の例も作れそうよね。
超越数については、そもそもどうやって具体例作ったり判定したりできるのかあたし知らないけどw

>>32
とりあえず、一辺の長さが1の正六角形を半分に切ってできる四角形は
辺の長さが1, 1, 1, 2で半径1の円に内接することに気づいたわ。
0034陽気な名無しさん2023/02/27(月) 02:46:35.190
確かにn=4なら存在するわね。
n=5ではどう考えればいいのかしら。
要はsinが全て有理数でcosが全て無理数(又は0)で、
Σ(i=1〜5)θ_i=πとなることがあるかどうかよね。

>>27の議論だと、鈍角の「ピタゴラス角の補角」があるタイプでも全て有理数の内接多角形が出来て、
それも全てピタゴラス角のタイプに含めるのよね?
今回はそれ以外のがあるかどうかよね。
直径を一辺として持つ場合とそうでない場合にわけて考えた方がいいのかしら?
直径を一辺として持つ場合、Σ(i=1〜n-1)θ_i=π/2で 考えることになるのよね?

思い付く範囲だけ書きなぐって、あとはうさぎに丸投げするわ。
何かわかったら書いてちょうだい。
0035陽気な名無しさん2023/02/27(月) 02:49:15.100
直径を一辺として持つ場合とそうでない場合にわけて考えた場合、
直径を一辺として持たない場合、四角形ではピタゴラス角でない場合は存在するのかしら?
0036usagi2023/02/28(火) 01:32:02.100
>>34
>鈍角の「ピタゴラス角の補角」があるタイプでも全て有理数の内接多角形が出来て、
>それも全てピタゴラス角のタイプに含めるのよね?
そうね。というか、角の合計がπだから、鈍角は現れるとしてもひとつだけよね?
だからその場合、それ以外の鋭角をθ_iとしてとっていけば、最後に残ったφ_{n−1}がその鈍角になるわ。
だからθ_iを選んでいくときは、あくまでピタゴラスの三角形に現れる内角だけをとっていけば良いのよ。

そして、sinの倍角の式を見て気づいたの。
sin2θ= 2sinθcosθ
sin3θ= 3sinθ−4sin^3θ
sin4θ= 4sinθcosθ−8sin^3θcosθ
sin5θ= 16sin^5θ−20sin^3θ+5sinθ
sin6θ= 32sin^5θcosθ−32sin^3θcosθ+6sinθcosθ
どうやら、奇数倍角の式はsinθの整式になって、偶数倍角の式は (sinθの整式)×cosθ になるみたいなの。

そうならば、次にことが分かるわ。
n角形を作るとき、θ_1, …, θ_{n−1}の n−1個の中心角を選ぶわけだけど、これをすべて同じ角度θにするわ。
この時θ< π/(n−1) であって sinθが有理数であるものを選ぶの。
すると、最後に残るφ_{n−1} について
sinφ_{n−1} = sin(π−(n−1)θ) = sin (n−1)θ
となるわ。
nが偶数なら、n−1は奇数だから、sinφ_{n−1} はsinθの整式で表せて、sinθが有理数だから
(cosθが無理数でも関係なく)φ_{n−1} も有理数になるのよ。
だから偶数角形の場合、この方法でいくらでも新しい例が作れるわ!
正六角形やその半分の四角形は、θ=π/6のケースだったのよ。
一方、nが奇数なら、sinφ_{n−1} は 有理数×cosθになるから、cosθが有理数でないとダメなの。
cosθが有理数の場合は、ピタゴラスの角度の場合だから、新しい例は何も増えないわ。
0037usagi2023/02/28(火) 02:02:31.170
じゃあ上に書いたsinの倍角の式の性質を証明するわ。

xの整式 A_n(x) と B_n(x) を
A_0(x) = 0
B_0(x) = x
A_{n+1}(x) = 2B_n(x) − A_n(x)
B_{n+1}(x) = 2(1−x^2)A_{n+1}(x) − B_n(x)
で定義する。任意の n ≥ 0 について
sin2nθ = cosθA_n(sinθ)
sin(2n+1)θ = B_n(sinθ)
が成り立つ。

証明
nに関する帰納法よ。n=0のときは明らかね。

和積の公式から
sin2(n+1)θ+ sin2nθ= sin(2n+2)θ+ sin2nθ= 2sin(2n+1)θcosθ
だから
sin2(n+1)θ
= 2sin(2n+1)θcosθ − sin2nθ
= 2B_n(sinθ)cosθ − cosθA_n(sinθ) (帰納法の仮定から)
= cosθ(2B_n(sinθ) − A_n(sinθ))
= cosθA_{n+1}(sinθ).

同様に和積の公式から
sin(2(n+1)+1)θ+ sin(2n+1)θ= 2sin(2n+2)θcosθ= 2sin2(n+1)θcosθ
だから
sin(2(n+1)+1)θ
= 2sin2(n+1)θcosθ− sin(2n+1)θ
= 2cos^2θA_{n+1}(sinθ) − B_n(sinθ) (すぐ上の結果と帰納法の仮定から)
= 2(1−sin^2θ)A_{n+1}(sinθ) − B_n(sinθ)
= B_{n+1}(x).
QED.

>>36で述べたやり方でもまだ作れない例があるのか気になるわ。
0038usagi2023/02/28(火) 02:12:13.290
>>36
>n角形を作るとき、θ_1, …, θ_{n-1}の n-1個の中心角を選ぶわけだけど、これをすべて同じ角度θにするわ。
って書いたけど、正確には選ぶのは「中心角の半分」ね。
0039陽気な名無しさん2023/02/28(火) 12:43:22.150
>>27
確認したわ
素晴らしいわね、あなた
0040陽気な名無しさん2023/03/01(水) 08:38:37.750
>>36
>そうね。というか、角の合計がπだから、鈍角は現れるとしてもひとつだけよね?

多角形の内角の話だから、三角形以外は角の合計はπより大きくなるわよ。
実際正6角形は全ての内角が鈍角よ。

って、あれ?>>27 の議論って中心角ではなくて内角よね。
内角の合計がπとしてすすめているから、三角形以外はこの議論あてはまらないのではないかしら。
0041陽気な名無しさん2023/03/01(水) 08:51:24.210
鈍角ってやっぱり中心角のことかしら。
そうするとそれに対応する多角形は円の中心が多角形の外部にあるタイプになるってことかしら。
ちょっとわかりづらいわ。
0042陽気な名無しさん2023/03/01(水) 09:01:51.890
中心角の話でしょ?>>27
0043Usagi2023/03/01(水) 11:27:14.960
>>40
分かりづらくてごめんなさい。あなた>>31-32とは別の方?
>>22で一応は説明したつもりだったんだけど、31-32で混乱があったから、やはり分かりにくかったのねと思って
ここ2〜3日、お絵描き機能で図を書き込もうとしてるんだけど、アクセス規制されているのかエラーになってしまって
場所をいろいろ変えてやってみても書き込めなくて困ってるの。
あたしふだん専ブラ使ってて、専ブラからだと書き込めるみたいなんだけど、専ブラではお絵描きができないのよね。

だから言葉だけの説明になってしまうけど
半径1の円に内接するn角形、例えば△ABCや六角形ABCDEFを描いてみて。
まずは円の中心Oがn角形の内部にあるものを考えましょう。
Oから円周上の各頂点に線を引くと、n個の三角形できるわ。
そのうちのひとつ△OABに注目しましょう。
OA = OB = 円の半径 = 1 だから、これは二等辺三角形で、⦣OAB = ⦣OBA になるわ。
だからOから線分ABに垂線を下ろしてその足をHとすると、⦣AOH = ⦣BOH となるわね。
θ_1 = ⦣AOH = ⦣BOH とおくと、AH = BH = sinθ_1 だから AB = AH + BH = 2sinθ_1 となるわ。
⦣AOB = ⦣AOH + ⦣BOH = 2θ_1 よね。同じように⦣BOC = 2θ_2 のようにおいていくと
2θ_1 + 2θ_2 + … + 2θ_n = 2π だから θ_1 + θ_2 + … + θ_n = π となるわ。
そして n角形の辺は 2sinθ_1, 2sinθ_2, … 2sinθ_n となるけど
これが全て有理数になるのは sinθ_1, sinθ_2, … sinθ_n が全て有理数になることと同値よね。

△ABCの場合は円周角の定理のおかげで、θ_1 = ⦣AOH = ⦣BOH が ⦣BCAと等しくなるけれど
四角形以上の場合は、内角とは関係ない話だったの。

おっしゃる通り、θ_1が鈍角の時は ⦣AOB = 2θ_1 がπを超える時で
円の中心Oがn角形の外にある場合になるわ。
この場合でも ⦣AOH = ⦣BOH = π−θ_1 だから AB = 2sinθ_1 であることに変わりないわ。
0044陽気な名無しさん2023/03/03(金) 13:09:00.6500303
>>36の方法をもう少し一般化できることに気づいたから書き込んでおくわ
nが偶数のとき、sinθが有理数でcosθが無理数であるθを選んで
各iについてθ_i = (奇数)×θであって、θ_1 + θ_2 + … + θ_{n−1} < π であるようにとるの
すると sinθ_i は有理数になるし θ_1 + θ_2 + … + θ_{n−1} が (奇数)×θ になるから
sinφ_{n-1} = sin(π-(θ_1 + θ_2 + … + θ_{n−1})) = sin(θ_1 + θ_2 + … + θ_{n−1})
も有理数になるわ。
0045陽気な名無しさん2023/03/06(月) 00:45:33.650
複素数 α,β,γ が
(α-β)^4+(β-γ)^4+(γ-β)^4=0
を満たしているとき
(α-β)^2+(β-γ)^2+(γ-α)^2
の値を求めよ
0046陽気な名無しさん2023/03/06(月) 00:48:33.470
複素数 α,β,γ が
(α-β)^4+(β-γ)^4+(γ-α)^4=0
を満たしているとき
(α-β)^2+(β-γ)^2+(γ-α)^2
の値を求めよ
0047陽気な名無しさん2023/03/06(月) 08:50:18.160
>>46
やっとできたわ。
0よ。
0049陽気な名無しさん2023/03/06(月) 16:29:05.52S
携帯なんで面倒だけど、
大雑把に書くわね。
α−β=A, β−γ=Bとするとγ−α=C=−(A+B)だから
(α-β)^4+(β-γ)^4+(γ-α)^4=0
に代入してAとBの式にして整理すると係数がぜんぶ偶数になるから
両辺を2で割って平方完成、じゃなくて4乗完成とでも言うのかしら?をして、
残りをABでくくって平方完成してからAB展開すると、
(A+B)^4−2AB(A+B)+(AB)^2=0
になるわ。因数分解できて
{(A+B)^2−AB}^2=0
2乗して0なのは0だから
(A+B)^2−AB=0
展開整理すると
A^2+AB+B^2=0
よって
B=Aω (ここでωは1の原始3乗根)
AとBとCの対称性より
C=Bω=Aω^2
よって
(α-β)^2+(β-γ)^2+(γ-α)^2
=A^2+B^2+C^2
=A^2+A^2ω^2+A^2ω^4
=A^2(1+ω^2+ω^4)
=A^2(1+ω^2+ω)
カッコの中が0だから、
=0
になるわ。
0050陽気な名無しさん2023/03/06(月) 16:31:11.59S
あ〜
(A+B)^4−2AB(A+B)+(AB)^2=0

(A+B)^4−2AB(A+B)^2+(AB)^2=0
の間違いよ〜
訂正!
0051陽気な名無しさん2023/03/06(月) 20:15:28.800
お見事よ!
まさかアンタに解けるとはね!!

今年の九州大学の問題でした
0052陽気な名無しさん2023/03/06(月) 20:16:12.440
アタシこの問題に取り組んでる時に新しい問題思いついたのよ
名作よ

誰か解ける方いたら解いて下さい
0053陽気な名無しさん2023/03/06(月) 20:20:07.130
a,b,cは整数でa=b=cではないとする
(a-b)^7+(b-c)^7+(c-a)^7

(a-b)^4+(b-c)^4+(c-a)^4
で割り切れることを示せ
0055陽気な名無しさん2023/03/07(火) 08:26:02.060
>>53
>>46 の解き方が分かった今となっては、
決して難しい問題ではないけれど、
よくそんな問題思いついたわね。
そっちの方が凄いわ。
0056陽気な名無しさん2023/03/07(火) 19:49:21.950
>>49
さすがね、すごいわ。後半だけど、同じように変数を書き換えると
(α-β)^2+(β-γ)^2+(γ-α)^2
= A^2 + B^2 + {-(A+B)}^2
= 2(A^2 + AB + B^2)
だから、A^2+AB+B^2=0 からすぐに0だとわかるわ。要は
(α-β)^4+(β-γ)^4+(γ-α)^4
= (1/2){(α-β)^2+(β-γ)^2+(γ-α)^2}^2
だったのね。
でもなんでこんな等式が成り立つのかよく分からないんだけど
何か意味があるのかしら?
0057陽気な名無しさん2023/03/07(火) 20:32:11.040
>>55
思わせぶりな書き方しないでちゃんと書いてよ!って思ったけど>>46と同じやり方するのね?
じゃあ A = a-b, B = b-c とおいて書き換えてみると
(a-b)^7+(b-c)^7+(c-a)^7
= A^7 + B^7 + {-(A+B)}^7
= -7AB(A^5 + 3A^4B + 5A^3B^2 + 5A^2B^3 + 3AB^4 + B^5)
= -7AB(A+B)(A^4 + 2A^3B + 3A^2B^2 + 2AB^3 + B^4)
ってなるのね? 決して難しくはないって、あなたこの計算ふつうに難しいわよw
一方
(a-b)^4+(b-c)^4+(c-a)^4
= 2(A^4 + 2A^3B + 3A^2B^2 + 2AB^3 + B^4)
だから AB(A+B) が 2で割り切れることを示せばいいのね。
AとBのどちらかが偶数なら、もちろんこれは2で割り切れるし
AとBのどちらも奇数なら、A+B が偶数だから、やはり2で割り切れるのね。
作者さんもこの解法を想定していたのかしら?
0058陽気な名無しさん2023/03/07(火) 23:21:24.150
>>57
そんな難しいことしてないわよ。
問題が「割り切れることを示せ」だったから、7乗の式を展開した式(もちろん−7ABではくくった後の式だけど)を、
4乗の式の因数のA^2+AB+B^2で二回割れるはずだと思って割ってみたのよ。
案の定割りきれて最後にA+Bが出てきたわ。
数式の割り算の筆算は多少面倒でも難しくはないでしょう?
それで7ABC(A^2+AB+B^2)^2って因数分解できて割り切れることがわかったのよ。
ABCが偶数になるのはすぐわかるからね。
0059陽気な名無しさん2023/03/08(水) 09:27:18.500
>>57
あなたの計算見ててふと思いついたんだけど、
>>58 にあるような展開整理して数式の割り算するのではなく、
A^7 + B^7 + {-(A+B)}^7
から因数(A+B)をくくりだしたいのだったら、
X^n+Y^nはnが奇数の時
(X+Y){X^(n-1)-X^(n-2)Y+・・・+Y^(n-1)}
って因数分解される性質を使えば、
A^7 + B^7
=(A+B)(A^6-A^5B+A^4B^2-A^3B^3+A^2B^4-AB^5+B^6)
だから、
A^7 + B^7 + {-(A+B)}^7
=(A+B){A^6-A^5B+A^4B^2-A^3B^3+A^2B^4-AB^5+B^6-(A+B)^6}
として6乗を展開整理すれば
-7AB(A+B)(A^4 + 2A^3B + 3A^2B^2 + 2AB^3 + B^4)
は出てくるわ。

もし
A^4 + 2A^3B + 3A^2B^2 + 2AB^3 + B^4
をさらに因数分解して
(A^2+AB+B^2)^2
まで変形しようとすると、知ってないと難しいと思うけど。
0060陽気な名無しさん2023/03/08(水) 09:29:35.460
>>58
>4乗の式の因数のA^2+AB+B^2
ってのは>>49 の変形から来てるのよね。
(a-b)^4+(b-c)^4+(c-a)^4
=A^4+B^4+{-(A+B)}^4
ここでA^4+B^4について、
(A+B)^4=A^4+4A^3B+6A^2B^2+4AB^3+B^4だから
A^4+B^4=(A+B)^4ー(4A^3B+6A^2B^2+4AB^3)
でしょ。だから
A^4+B^4+{-(A+B)}^4
=(A+B)^4ー(4A^3B+6A^2B^2+4AB^3)+(A+B)^4
=2{(A+B)^4ーAB(2A^2+3AB+2B^2)}
になるじゃない。それで
2A^2+3AB+2B^2
=2A^2+4AB+2B^2-AB
=2(A^2+2AB+B^2)-AB
=2(A+B)^2-AB
となるから、最初の式は
2{(A+B)^4ーAB(2(A+B)^2-AB)}
=2{(A+B)^4−2AB(A+B)^2+(AB)^2}
=2{(A+B)^2−AB}^2
=2(A^2+AB+B^2)^2
と変形できるわ。だから
A^2+AB+B^2で二回割れて残りが偶数ならいいのね。
0061陽気な名無しさん2023/03/08(水) 15:25:44.830
>>58
たしかに難しいというよりは、面倒とか大変って感じかもしれないけど
あたし計算嫌いだから、7乗の展開とか5乗の式の因数分解とか
見ただけでやる気なくなりそうになるから、そういう意味で難しいわ
もしテストでこんな問題出されたらきっとすぐ諦めるわw

>>59
なるほど、それもうまいわね
あたしは A^5 + 3A^4B + 5A^3B^2 + 5A^2B^3 + 3AB^4 + B^5 を因数分解するにあたって
係数が左右対称なのを利用したわ
全体を B^5 で割って x = A/B とおいて =0 をつけると
x^5 + 3x^4 + 5x^3 + 5x^2 + 3x + 1 = 0
ってなるじゃない? ネットで調べて知ったんだけどこういうの相反方程式っていうらしいわ
奇数次の場合、x = -1 を代入すると成立するから、因数定理で (x+1) で割り切れるの
元の話に戻ると、Aに-Bを代入すると0になるから(A+B)で割り切れるということね
それで A^5 + 3A^4B + 5A^3B^2 + 5A^2B^3 + 3AB^4 + B^5 を A+B で割って>>57は出したわ
最初ふつうに筆算を書いてやって大変だったけど、よく考えると簡単にできることに気づいたわ
係数が 1, 3, 5, 3, 1 だから A+B で割った商については
A^4 の係数は1
A^3B の係数は 3 - 1 = 2
A^2B^2 の係数は 5 - 2 = 3
AB^3 の係数は = A^3B の係数 = 2
B^4 の係数は = A^4 の係数 = 1
って考えるといいの(商の係数も左右対称になるのね)

ちなみに相反方程式に戻ると
x^5 + 3x^4 + 5x^3 + 5x^2 + 3x + 1 = (x+1)(x^4 + 2x^3 + 3x^2 + 2x + 1) = 0
ってなるけど
x^4 + 2x^3 + 3x^2 + 2x + 1 = 0
を解くには、まず全体を x^2で割って
x^2 + 2x + 3 + 2/x + 1/x^2 = 0
とするの。x + 1/x = t とおくと、x^2 + 1/x^2 = t^2 - 2 だから
0 = x^2 + 2x + 3 + 2/x + 1/x^2 = t^2 - 2 + 2t + 3 = (t+1)^2
したがって t+1 = 0 つまり x + 1/x + 1 = 0 を解けばいいのね。そして
x^4 + 2x^3 + 3x^2 + 2x + 1
= x^2(t+1)^2
= x^2(x + 1/x + 1)^2
= (x^2 + x + 1)^2
もわかるわ。x = A/Bから元の話に戻ると>>60の因数分解も出るわね。
0062陽気な名無しさん2023/03/09(木) 11:41:25.350
ところで、A = a-b, B = b-c とおくと A^2+AB+B^2 = a^2+b^2+c^2-ab-bc-ca だけど
一般に (a-b)^n+(b-c)^n+(c-a)^n が a^2+b^2+c^2-ab-bc-ca で何回割れるのか気になって
Wolframで n = 20 まで試してみたんだけど、どうやら2以上のnに対して

・n ≡ 0 (mod 3) なら a^2+b^2+c^2-ab-bc-ca を因数に持たない
・n ≡ 1 (mod 3) なら (a^2+b^2+c^2-ab-bc-ca)^2 を因数に持つ
・n ≡ 2 (mod 3) なら (a^2+b^2+c^2-ab-bc-ca)^1 を因数に持つ

となるみたいなの! 誰かこれを証明してくれないかしら?
アタシって個々の具体例の計算だけ見せられても
本質的な理由や意味が分からないと納得できないタイプなの

まず n ≡ 0 (mod 3) の場合だけど
もし (a-b)^n+(b-c)^n+(c-a)^n = A^n+B^n+{-(A+B)}^n が
a^2+b^2+c^2-ab-bc-ca = A^2+AB+B^2 を因数に持って
A^n+B^n+{-(A+B)}^n = (A^2+AB+B^2)f(A, B)
となったとして、B = Aω なら -(A+B) = Aω^2 だから左辺は 3A^n になって
右辺は A^2+AB+B^2 = 0 だから 0 になって
3A^n = 0 ができるけど、これは恒等式じゃないから矛盾よね
n ≡ 0 (mod 3) の場合はこれでいいわよね?

n ≢ 0 (mod 3) の場合は、 B = Aω なら A^n+B^n+{-(A+B)}^n = 0 よね
両辺をA^nで割ってx = B/Aとおけばxのn次方程式ができるけど
x = ω がその根なわけだから、このn次式はωの最小多項式である x^2+x+1 で割り切れるのよね?
そうすると、A^n+B^n+{-(A+B)}^n は A^2(x^2+x+1) = A^2+AB+B^2 で割り切れることになるわ
(ちょっとよくわからないんだけど、A^nで割るから、A = 0 の場合を除外して議論しなきゃいけないのかしら?)

だとすると残る疑問は、n ≡ 1 (mod 3) の時はちょうど2回(?)割れて
n ≡ 2 (mod 3) の時は1回だけ(?)割れるのはなぜ?ってことになるわ
0063陽気な名無しさん2023/03/09(木) 18:14:28.720
t=A/Bとすると
A^n+B^n+{-(A+B)}^n=B^n(t^n+1+(-t-1)^n)
A^2+AB+B^2=B^2(t^2+t+1)
実数係数の多項式f(t)が(t^2+t+1)^2=((t-ω)(t-ω*))^2で割り切れる
⇔ f(ω)=f'(ω)=0
なのでf(t)=t^n+1+(-t-1)^nの微分がt=ωで0になるかを見ればよい
0064陽気な名無しさん2023/03/09(木) 21:48:32.780
>>63
あら、姐さん素晴らしいヒントありがとう!早速計算してみたわ。
まず -ω-1 = ω^2 だから f(ω) = ω^n + 1 + ω^{2n} で
これは n ≡ 0 (mod 3) なら3、n ≢ 0 (mod 3) なら0になるわね
次に f’(t) = n(t^{n-1} - (-t-1)^{n-1}) だから
f’(ω) = n(ω^{n-1} - (ω^2)^{n-1}) = n(ω^{n-1} - ω^{2n-2}) で
n ≡ 1 (mod 3) なら n-1 ≡ 0, 2n-2 ≡ 0 だから f’(ω) = n(1-1) = 0
n ≡ 2 (mod 3) なら n-1 ≡ 1, 2n-2 ≡ 2 だから f’(ω) = n(ω-ω^2) ≠ 0
これで f(t) は n ≡ 1 なら t^2+t+1 で2回割れて、n ≡ 2 なら1回しか割れないことがわかるわね!
さらに f’’(t) = n(n-1)(t^{n-2} + (-t-1)^{n-2}) だから
f’’(ω) = n(n-1)(ω^{n-2} + (ω^2)^{n-2}) = n(n-1)(ω^{n-2} - ω^{2n-4}) で
n ≡ 1 (mod 3) のとき n-2 ≡ 2, 2n-4 ≡ 1で f’’(ω) = n(n-1)(ω^2-ω) ≠ 0 なので
この時も3回は割れないことがわかるわ!

この結果を利用すれば (a-b)^n+(b-c)^n+(c-a)^n の因数分解も見通しがよくなるわね

n = 4 のときは n ≡ 1 (mod 3) だから (a^2+b^2+c^2-ab-bc-ca)^2 を因数に持つけど、これは4次式だから
(a-b)^4+(b-c)^4+(c-a)^4 = (定数) × (a^2+b^2+c^2-ab-bc-ca)^2
だとわかるわ。あとは例えば両辺の a^4 の係数を比較すればこの定数が 2 だとわかるわ

n = 7 のときも (a^2+b^2+c^2-ab-bc-ca)^2 を因数に持つわ。
そしてnが奇数の時 (a-b)^n+(b-c)^n+(c-a)^n は交代式だから差積で割れて
(a-b)^7+(b-c)^7+(c-a)^7 = (a-b)(b-c)(c-a)(a^2+b^2+c^2-ab-bc-ca)^2 × (対称式)
となることがわかるけど、次数を考えるとこの対称式は定数で
例えば両辺の a^6b の係数を比較するとこの定数が 7 だとわかるのね

同様に 5 ≡ 2 (mod 3) から
(a-b)^5+(b-c)^5+(c-a)^5 = 5(a-b)(b-c)(c-a)(a^2+b^2+c^2-ab-bc-ca)
もすぐわかるわね
0065陽気な名無しさん2023/03/11(土) 10:30:32.340
あら、計算してくれたのね
どうもありがとう

しかし、こういう興味深い性質が成り立つ
本質的な理由はアタシにはまだ分からないが……
0066陽気な名無しさん2023/03/12(日) 14:36:06.310
考え事をしていたらまた問題を思い付いたの
完全に受験レベルだけど、受験生はどう解くかしら?

x,y,zは複素数で、
-2x+y+z, x-2y+z, x+y-2z
が複素数平面上で表す点たちが正三角形をなす
このときx,y,zが表す点たちはどのような図形を創出するか?
0067陽気な名無しさん2023/03/13(月) 13:54:27.740
>>66
これって、「こんな風に解くんじゃない〜?」
って答えて欲しいのかしら?
それともガチで解いた方がいいのかしら?
0068陽気な名無しさん2023/03/13(月) 17:15:28.810
ガチンポでお願いします
0069陽気な名無しさん2023/03/14(火) 10:09:14.210
じゃあガチでいくわよ。
二通りの解法を考えたので、考えた順に書くわね。
まず解法1:これは受験生はどう解くかなって考えた方法。
A=-2x+y+z、B=x-2y+z、C=x+y-2z として、
それぞれを、Aを原点に移すように平行移動する。
A→AーA=0
B→BーA=3x-3y
C→CーA=3x-3z
これらが正三角形をなすんだから、
CーAはBーAを、原点を中心として60°又はー60°回転させたもの。
つまり
3x-3z=(3x-3y)(1/2±√3i/2)
両辺をー3で割って
z-x=(y-x)(1/2±√3i/2)
この式は、z-xはy-xを、原点を中心として60°又はー60°回転させたもの
という意味になる、つまり原点、y-x、z-xは正三角形をなすので、
それぞれをxだけ平行移動させると
原点→0+x=x
y-x→y-x+x=y
z-x→z-x+x=z
これら、つまりx,y,zも正三角形をなす。
0070陽気な名無しさん2023/03/14(火) 10:10:04.800
次に解法2:これは解法1を考えてから気づいたもので、
気づけばこんなに簡単にシンプルにできるんだ、って思った方法。
-2x+y+z, x-2y+z, x+y-2zが正三角形をなす
⇔-3x, -3y, -3zが正三角形をなす(-x-y-z平行移動)
⇔x, y, zが正三角形をなす(原点中心の相似比3:1の逆側への相似)
0071陽気な名無しさん2023/03/15(水) 07:39:50.970
>>70
んまー、あなたさすがねやっぱり
この方法があったとは🤭
>>69もすごく実践的な方法をありがとうございます

https://pbs.twimg.com/media/Fq7LqfdaEAEdSXI.jpg
この九大の問題調べてたら
https://manabitimes.jp/math/855
を見つけたの
それで
-2x+y+z, x-2y+z, x+y-2zが正三角形
⇔(-3x+3y)^2+(-3y+3z)^2+(-3z+3x)^2=0
⇔(x-y)^2+(y-z)^2+(z-x)^2=0
⇔x,y,zが正三角形
になるな、と思ってたの
0072陽気な名無しさん2023/03/15(水) 23:42:27.850
>>70
シンプルで素晴らしいわ

>>71
ケチつけるつもりじゃないんだけど、あなたの問題だと複素数同士の掛け算が出てこないから
複素数を使う意味があんまりないわよね。単純にベクトルの話にして

-2OA+OB+OC, OA-2OB+OC, OA+OB-2OCを位置ベクトルとする3点が正三角形のとき△ABCはどんな三角形か

でよかったんじゃないかしら
0073陽気な名無しさん2023/03/16(木) 04:08:09.680
>>70 の解法だとベクトルで十分だけど、
>>69>>71 は一応乗法使っているわ。
71さんは解法として二乗をつかった、
リンク貼ってある定理使う解法を想定
してたから複素数の必要があったのでしょうね。

ただ71の定理って、一般の受験生は
多分知らないし、証明を見てみると
結局69でやったこと(60°だから〜みたいな)
をショートカットしてるような内容よね。
受験生はどう解くのかしら、
ってのの答えはやっぱり69じゃないかしら。

高校数学の美しい物語って、結構
高校レベル越えていたり、
越えていなくても高校生は
普通知らないようなこと、
よく書いてあるわよね。
あれが基準で受験問題出されると、
受験生には厳しいのではないかしら。

なんにしてもネタ投下してくれた
>>66 には感謝するわ。
0074陽気な名無しさん2023/03/16(木) 11:39:46.900
>>73
そうね、でも>>69の掛け算は複素数平面での位置関係を表現しているだけで、問題を解くために特に必要な計算ではなくない?
あたしも69に似た方法で解いたんだけど頭の中は完全にベクトルで解いたわ
ちょっと書いてみるわ
-2OA+OB+OC, OA-2OB+OC, OA+OB-2OCを位置ベクトルとする3点をP, Q, Rとすると
PQ = OQ - OP = (OA-2OB+OC) - (-2OA+OB+OC) = 3(OA - OB) = 3BA
同様に QR = 3CB, RP = 3AC
PQ, QR, RPが正三角形の3辺だからBA, CB, ACはその1/3の大きさの正三角形の3辺だとわかるわ

(α-β)^2+(β-γ)^2+(γ-α)^2 = 0 は複素数であるからこそ可能な表現だけどね
0075陽気な名無しさん2023/03/16(木) 12:07:19.200
こう書くべきだったわ
PQ = -3AB, QR = -3BC, RP = -3CAだから、AB, BC, CAは1/3の大きさの正三角形の3辺
0076陽気な名無しさん2023/03/16(木) 12:09:26.770
>>74
その解き方はむしろ70に近いかと。
そうでもないかな。
69も71で使っている定理も、
なす角が60°であることを使っている点が
74の解き方と決定的に異なると思うのよ。
74は三辺が等しいことだけで示せているから
60°使う必要がないでしょ。

位置関係といってしまえば、
掛ける複素数を極表記すれば
複素数の掛け算は位置関係を表す、
とも言えるのではないかしら。
69で掛け算の計算が必要なかったのはたまたまよ。
掛け算の形に表せたから両辺3で割ることもできたんだしね。
69は三角形のうちの二辺しか考えてないのよ。
そして71で使った定理の証明は複素数の乗除を使ってるわ。

70と74の解法ならベクトルでいいけど、
69と71はベクトルでは使えないわ。
0077陽気な名無しさん2023/03/16(木) 12:22:12.650
>>76
言われて気づいたけど、あたし3辺が等しいとか考えてなかったわ
>>75に書き直した方を見てもらえればと思うんだけど、△PQRと△ABCは相似だから
△PQRがどんな種類の三角形でも、△ABCは同じ形で1/3の大きさの三角形(を180度回転させたもの)になるって言いたかったの
0078陽気な名無しさん2023/03/16(木) 12:49:16.520
>>77
言いたいことはわかるわ。
その言いたいことが、70の最後2行の同値変形に
ちょうど対応してると思うのよ。
0079陽気な名無しさん2023/03/16(木) 22:24:58.440
>>78
確かにそうね。でも70の最初の-x-y-z平行移動というのが自然に思いつかないわ。

あたしが最初に思ったのはこうだったの。>>69のA, B, Cを使わせてもらうと
B-A = -3(y-x), C-B = -3(z-y), A-C = -3(x-z)
はA, B, Cの作る正三角形の3辺に相当する複素数で、平面上のベクトルと同一視できるわ
y-x, z-y, x-z はx, y, zの作る三角形の3辺に相当するもので、上のベクトルを-1/3倍したものだから
これはA, B, Cの作る正三角形を1/3にしたものだとすぐわかるわって思ったの
これじゃあ解答としては認められないかしらね?
最初からベクトルを使った方がわかりやすく書けるのよね

A, B, Cの相対的な位置関係を調べるのがあたしの考えだったから、69の方が近い発想なの
もしあたしが受験生で、上の解答が認められなくてあくまで複素数平面の話をしなきゃいけないということなら
なんでこんな無駄なこと書かなきゃいけないのかしらって思いながら、69みたいなものを書いた可能性が高いわ

70は鮮やかだけど-x-y-z平行移動するのが謎ねって思ったけど、ちょっと考えてみたわ
A, B, Cの作る三角形の重心は (A+B+C)/3 = 0 よね
一方、-3x, -3y, -3z の作る三角形の重心は {(-3x)+(-3y)+(-3z)}/3 = -x-y-z ね
だから、これは重心の位置が 0 から -x-y-z になるように平行移動するって意味だったのね
やはり背後にあるのはベクトル的な考え方よね

ところで、ベクトルは今まで数学Bに入っていたそうだけど
文科省が統計かなんかを数学Bに入れるために
新課程ではベクトルは数学Bから追い出されて新しくできる数学Cに移るそうよ
けれどベクトルは文系の人にとってもはずせないから
結局、文系の人も数学Cまで勉強しなきゃいけないことになるんですって
文科省のやることってほんと馬鹿馬鹿しいわよね
そもそもベクトル知らないと高校の物理の勉強に問題あると思うの
だからベクトルはむしろ中学で教えた方が良いと思うわ
あとあたし、数学Iとか数学Aとか名前を見ても何をやるのか不明で
課程が変わると同じ名前でも中身が全然違うとか、そういうのすごく嫌い
0080陽気な名無しさん2023/03/17(金) 09:38:00.970
>>79
70の最初の-x-y-z平行移動はね、69を考えた後に、
三頂点を一つの文字で表せるように平行移動できないかな、って考えて出てきただけのものなの。
あなたはこれを重心の移動できちんと説明づけてくれたけど、
最初そんなきちんとした考えでやったわけではないのよ。

あなたは正三角形ってことでわりと最初から、三辺、ベクトル、がキーワードだったようだけど、
あたしは複素数で正三角形ってことは、ひとつの頂点を原点にすると
他二つはcos(±60°)+isin(±60°)をかけると移り合うな、
ってのが最初の発想で、そこから出てきた解法が69だったの。
最初のキーワードはcos(±60°)+isin(±60°)だったのよ。

それで69ができてから、どうせ平行移動するんなら、もっと簡単に行かないかな、と思って、結局
平行移動と相似だけで説明できてしまったのが70なんだけど、
ベクトルについては全く頭になかったわ。
重心による説明はベクトル使うと確かにきれいにできるわね。
あたしはそこまで考えていなかったから感心したわ。

新課程の情報、それ本当なの?ベクトルを数学Cにするのは酷いわね。
中学校はもっと初歩的なことしかやっていないから難しいとしても、
高1とかでやってもいい内容に思えるわ。
数学Iは中学の方程式、関数の続きだからあまりいじれないけど、
三角比って数学Iで必要かしら?三角関数とセットじゃダメ?とか思うわ。
数学Aは確率、整数、図形って高1でやるべきことの中の、
中学の続き以外の補集合みたいなイメージね。ここもちょっといじれそう。
従来の過程では数学Iにデータなんちゃら、数学Aに確率、数学Bに確率分布
みたいになっていたじゃない?これら一か所にまとめられないかしら?
まとめて数学Bにして、数学Aの空いたところにベクトル持ってくるとか。
そうすると、あと仲間外れになるのは何かしら?数列?
数列って本来解析で使うんだから、文系の人にはあまり必要ないのでは?
それこそ高3でもいいのではないかしら。ああでも帰納法の考え方は欲しいかしら。
それともブロック状に固めるより細く長く、同時にいろんなものをやった方がいいのかしら?
0081陽気な名無しさん2023/03/17(金) 20:42:38.08F
>>80
>新課程の情報、それ本当なの?ベクトルを数学Cにするのは酷いわね。
そうなのよ。そのせいで共通テストもこれまでII•Bだったものが新課程ではII•B•Cになるそうよ。
確かに、本来ひと続きの内容のものがばらばらにされて違う科目に入れられているのおかしいわよね
昔は「確率•統計」とか「幾何」みたいにちゃんと内容を表す科目名が使われていた時期もあったのよね?
そっちの方が学問的に正しいあり方だと思うわ
確率なら確率でまとまっている方が教科書とかも絶対に勉強しやすいはずだしね
勉強方法としてはいろいろ同時平行がいいかもわからないけど、科目の分け方はそうすべきよ
生徒だって数学Bみたいな名前じゃワクワク感なくてやる気おきないし
あとで数学Bって何習ったんだっけ?て考えても思い出せなくなるわ
(現にあたしが思い出せないもの)

あたしはそもそも高校のカリキュラムが詰め込みすぎでバランスおかしいと思ってるの
他の科目もそうだろうけど、特に数学は中学に比べて高校で量も難しさも増えすぎじゃない?
内容が多くて難しいほど勉強するのに時間がかかるんだから、もっと前倒しすべきだわ
特に小学校って、6年間もかけて負の数を含まない四則演算の計算法しか習わないのアホすぎるわ
だいたい今の時代、計算なんか機械にやらせればすむことよね
あたし小学生のとき一番得意で好きな科目は算数だったけど、授業が一番つまらないのも算数だったわ
小学校の算数で一番大変だったのは2年生で九九を覚えることで、あとはバカみたいだったわ
小学生って、言語能力が発達してないだけで別にバカじゃないのよね
小学生のうちに負の数の計算、1次方程式、1次関数、連立1次方程式、1次不等式くらいまで教えるべきよ
そしたら中学の間に今の高1くらいまでのこと(とベクトル)を勉強できて
高校ではもっと落ち着いてじっくり勉強できるようになると思うわ
0082陽気な名無しさん2023/03/17(金) 20:50:08.540
これだけ算数の落ちこぼれがいる国で授業内容の前倒し?
ありえないわ
0083陽気な名無しさん2023/03/18(土) 12:28:20.530
>>82
あたしの偏見だけど、落ちこぼれる人ってカリキュラムを多少簡単にしたところで
やっぱり落ちこぼれる人が多いと思うのよ
それにどんなカリキュラムにしても落ちこぼれる人をなくすことはできないわ
そこを基準にして物事を作るとどんどんおかしくなると思うの
高校数学は小学校算数の何十倍も量が多くて難しいのに
小学校の半分の期間で習得させようというカリキュラムの方が歪んでいるし
中高で数学嫌いを生む原因になっている可能性も高いと思うの
逆に小学校の算数は知的刺激なさすぎで、小学生が算数に興味を持つことを難しくしてると思うわ
小学生の知性に対する侮辱よ!
0084陽気な名無しさん2023/03/18(土) 13:45:59.090
つうか調べたら最近は1次不等式や2次関数も中学では習わないの?
ますます高校に詰め込まれてて無理ゲーっぽくなってるじゃない
高校生がほんと可哀想だわ
1次不等式って小学生でも理解できることなのに
0085【B:92 W:77 H:94 (D cup) 147cm/76kg age:31】 535페이지 16번2023/03/18(土) 22:04:29.760
アタシも久しぶりに問題作ってみたわ
お暇なら考えてみて


以下の条件を全て満たす複素数x,y,zを求めよ
(1+x+y+z+cos(2π/5)+isin(2π/5))²=5(1+x²+y²+z²+cos(4π/5)+isin(4π/5))
(1+x+y+z+cos(2π/5)+isin(2π/5))³=25(1+x³+y³+z³+cos(6π/5)+isin(6π/5))
(1+x+y+z+cos(2π/5)+isin(2π/5))⁴=125(1+x⁴+y⁴+z⁴+cos(8π/5)+isin(8π/5))
1<|x|≦|y|≦|z|
0086陽気な名無しさん2023/03/20(月) 23:47:00.610
>>85
その名前欄のへんなのは何?
0087陽気な名無しさん2023/03/25(土) 13:13:03.490
誰も書き込まないわね。
スレが落ちそうだわ。
>>85 はそろそろヒントでも出したらどうかしら?
0088陽気な名無しさん2023/03/25(土) 16:13:48.840
難しいわよね。せっかくだからアタシの考えたこと書いておこうかしら。
まず 1<|x|≦|y|≦|z| という条件がなければ、ω = cos(2π/5)+isin(2π/5) として
x = ω^2, y = ω^3, z = ω^4 とすれば
1+x+y+z+cos(2π/5)+isin(2π/5) = 1+ω^2+ω^3+ω^4+ω = 0
で式がすべて 0 = 0 となって成り立つから、そうでない答えが求められてるのよね。

原理的には x, y, zの対称式でできているから A = x+y+z, B = xy+yz+zx, C = xyz とおいて
3つの式を A, B, C で表すことができるから、それをA, B, Cについて解けば
あとは t^3 - At^2 + Bt - C = 0 の解が x, y, z になるとは思うけど、計算を考えると途方に暮れるわね

あとちょっと思ったのは、p = (1+x+y+z+ω)/5 とおいて計算すると
5(p+p^2+p^3+p^4) = (x^5-1)/(x-1) + (y^5-1)/(y-1) + (z^5-1)/(z-1)
となるわね。左辺は p≠1なら 5(p^5-1)/(p-1) - 5 に等しいわ。
何も役立たないかもしれないけど。
0090Usagi2023/03/25(土) 19:12:41.190
>>89
何これ、「うさぎはどこへ逃げた?」って質問者はあなたなの?
これアタシのことなの? なんか嫌な感じね
ていうか>>88はアタシなんだけど、この解答者は>>88の真ん中に書いた方針で頑張ってやったみたいね
アタシは対称式に書き直す途中で嫌になってやめたけどw
アタシはもちろん暗算でも解けないわよ

コテハンにしてると、こういうふうになんだか敵対的な態度取られるみたいだし
ものぐささんも名無しに戻ってひとりだけコテハンなのも寂しいからやめたの
うさぎは寂しいと死ぬのよ
それにコテハンの書き込みだらけだと新参者が書き込みにくく感じるかもしれないとも思ったから
0091陽気な名無しさん2023/03/25(土) 20:41:33.650
なにこれ、全然大学受験レベルじゃないじゃない。
0092陽気な名無しさん2023/03/25(土) 20:49:15.750
同サロでこんな長文書いてるのキチガイとここのインテリくらいよw
0093陽気な名無しさん2023/03/25(土) 20:53:58.850
えっ、これ大学受験レベルでしょ?
0094sageterandkp2023/03/25(土) 22:10:17.260
まあとにかくリンク先の答えがかなりのヒントよ
0096陽気な名無しさん 158ページ 74番2023/03/25(土) 23:39:29.070
不適のものも含めてらすかるさんが求めてくれた値と1とe^(2πi/5)とは複素数平面上でどのような位置関係にあるのかしら?

アタシたちはちょうど複素数平面上の正三角形の話をしていたところだったわよね
0097Usagi2023/03/26(日) 01:46:46.450
>>89
ていうかあなたは、こことリンク先の掲示板で同時に問題を出してたわけ?
しかも自分で作っておきながら、しれっと「解き方、答えを教えてください」って書き込んだの?
何のために?
それによく見ると問題が違うわね。右辺の係数が違うわ。
0098Usagi2023/03/26(日) 05:07:58.420
失礼、よく見たら左辺の { } の中にも /5 がついているから同じ問題ね。
>>96
不適のものは調べていないけど、正解を調べたら正5角形であることがわかったわ。
それで考えたら、秘密がわかったかもしれないわ。
aとb (≠0) を複素数、ω = e^(2πi/5) とするわ。
1, ω, ω^2, ω^3, ω^4 は正5角形よね。
これらにbを掛けた b, bω, bω^2, bω^3, bω^4 はこれに相似だから正5角形になるわ。
それを並行移動した a+b, a+bω, a+bω^2, a+bω^3, a+bω^4 も正5角形よね。
ω^4+ω^3+ω^2+ω+1 = 0 だから
{(a+b)+(a+bω)+(a+bω^2)+(a+bω^3)+(a+bω^4)}/5 = a
で、これがこの正5角形の重心なのね。さらに
(a+b)^2 + (a+bω)^2 + (a+bω^2)^2 + (a+bω^3)^2 + (a+bω^4)^2
= 5a^2 + 2ab(1+ω+ω^2+ω^3+ω^4) + b^2(1+ω^2+ω^4+ω^6+ω^8)
= 5a^2 + 2ab(1+ω+ω^2+ω^3+ω^4) + b^2(1+ω^2+ω^4+ω^1+ω^3)
= 5a^2
となるわ。同様に n ≢ 0 (mod 5) なら
(a+b)^n + (a+bω)^n + (a+bω^2)^n + (a+bω^3)^n + (a+bω^4)^n = 5a^n
となるわね。
1<|x|≦|y|≦|z|という条件がなければすぐ思いつく答え x = ω^2, y = ω^3, z = ω^4 は a = 0, b = 1の場合で
そしてこの問題の正解になっているのは a = 1+ω, b = -1 の場合なのよ。
おそらく他の不適解も、1とωを頂点に持つ正5角形なんじゃないかしら。
(1とωが隣同士でない頂点となる正5角形がふたつ描けると思うわ。)
ただ、以上は正5角形の頂点が与式を満たすことを確かめただけだから
逆に与式を満たすものが正5角形の頂点に限られるのかどうかはわからないけど。
0100陽気な名無しさん2023/03/26(日) 14:25:23.950
高校レベルじゃないのは事実だと思うわ。
スレタイと内容が合ってないわ。
スレタイ「超高校レベルの数学でうさぎにチャレンジ」とでもした方が
スレの実態と合ってるんじゃないの?
0101【B:101 W:94 H:97 (H cup) 157cm/92kg age:55】 287ページ 27番2023/03/26(日) 15:52:08.830
(1) >>85が成り立つとき、5点1,ω,x,y,zを平行移動しても同様の関係が成り立つことを示しなさい
たとえば
(1+a)^4+(ω+a)^4+(x+a)^4+(y+a)^4+(z+a)^4
=1+4a+6a^2+4a^3+a^4
+ω^4+4ω^3a+6ω^2a^2+4ωa^3+a^4
+x^4+4x^3a+6x^2a^2+4xa^3+a^4
+y^4+4y^3a+6y^2a^2+4ya^3+a^4
+z^4+4z^3a+6z^2a^2+4za^3+a^4
=……
を計算して(     )^4/125の形にできるだろうか?

(2) (1)のaをうまく選ぶと
(1+a)^2+(ω+a)^2+(x+a)^2+(y+a)^2+(z+a)^2=0
(1+a)^3+(ω+a)^3+(x+a)^3+(y+a)^3+(z+a)^3=0
(1+a)^4+(ω+a)^4+(x+a)^4+(y+a)^4+(z+a)^4=0
とできることを示しなさい

(3) 基本対称式とべき乗和の関係を表す式は一般に「(     )の恒等式」と呼ばれています
(     )に入る人名として最も適切なものを以下から選びなさい
あ. ニュートン
い. アインシュタイン
う. ヒルベルト
え. ネーター
お. ノイマン

(4) (2)のよう2乗、3乗、4乗のべき乗和が0になるような5つの複素数が根となる多項式は係数にどのような特徴があるか答えなさい

(5) >>85の問いに答えなさい
0102 【利用料 103238 ₲】 883ページ 78番2023/03/26(日) 17:11:05.880
(1) >>85が成り立つとき、5点1,ω,x,y,zを平行移動しても同様の関係が成り立つことを示しなさい
たとえば
(1+a)^4+(ω+a)^4+(x+a)^4+(y+a)^4+(z+a)^4
=1+4a+6a^2+4a^3+a^4
+ω^4+4ω^3a+6ω^2a^2+4ωa^3+a^4
+x^4+4x^3a+6x^2a^2+4xa^3+a^4
+y^4+4y^3a+6y^2a^2+4ya^3+a^4
+z^4+4z^3a+6z^2a^2+4za^3+a^4
=……
を計算して(     )^4/125の形にできるだろうか?

(2) 以下を満たすaが存在することを示しなさい
(1+a)^1+(ω+a)^1+(x+a)^1+(y+a)^1+(z+a)^1=0
(1+a)^2+(ω+a)^2+(x+a)^2+(y+a)^2+(z+a)^2=0
(1+a)^3+(ω+a)^3+(x+a)^3+(y+a)^3+(z+a)^3=0
(1+a)^4+(ω+a)^4+(x+a)^4+(y+a)^4+(z+a)^4=0

(3) 基本対称式とべき乗和の関係を表す式は一般に「(     )の恒等式」と呼ばれています
(     )に入る人名として最も適切なものを以下から選びなさい
あ. ニュートン
い. アインシュタイン
う. ヒルベルト
え. ウサギ
お. ノイマン

(4) (2)のように1乗、2乗、3乗、4乗のべき乗和が0になる5つの複素数を根にもつ5次多項式は係数にどのような特徴があるか答えなさい

(5) >>85の問いに答えなさい
ただしcos(2π/5)やe^(2πi/5)などは必ずしも真の値を求める必要はなく、このまま表してかまわない
0103 【年収 1032 万】 445페이지 76번2023/03/26(日) 17:14:45.160
>>100
こうやって一歩ずつ考えてみるとアンタたちにもこの問題は完全に高校数学の範疇だと理解できるはずよ?
(3)はまあ豆知識として付け加えただけで、他は全て高校数学
0104Usagi2023/03/27(月) 01:28:35.190
問題出しっぱなしじゃなくて、解説してくれてありがとう
高校までの知識で理解できるのかもしれないけど、こういう誘導なしで解ける人なんて高校生じゃなくてもほぼいないでしょ
というか誘導ありでも解ける人おそらく人口の1%もいないと思うわw
まあレベルはどうであれ、ちゃんと読者に分かるように説明してくれるならアタシは歓迎するわ

ただ、攻撃的?な態度をとるのはやめてね
前スレでは、だんだん問題が出されるとアタシが解かなきゃいけないような気がしてきちゃって
ちょっとストレスになったりしてきてバカみたいだったから
名無しになる方が気楽な面もあったけど、逃げた、とか言いがかりつけないで
アタシはただ楽しく数学の話がしたいだけなの

でも今回のあなたの問題はとても興味深かったわ
>>98で与式を満たすことが正5角形の頂点であるための必要条件なのはわかったけど
>>102の考え方で、十分条件でもあるとわかるのね
一般化すると複素数 z_1, z_2, …, z_n が正n角形を作るための必要十分条件は、1以上n未満のすべてのkに対して
{(z_1 + z_2 + … + z_n)/n}^k = (z_1^k + z_2^k + … + z_n^k)/n
が成り立つことと言えるのね? 最初nが素数の時だけかと思ったけど、素数でなくても成り立つわよね?
これを使うと、α, β, γが正3角形を作る条件は
{(α+β+γ)/3}^2 = (α^2+β^2+γ^2)/3
⟺ 2α^2+2β^2+2γ^2−2αβ−2βγ−2γα = 0
⟺ (α−β)^2+(β−γ)^2+(γ−α)^2 = 0
となって>>71のリンク先の条件が出てくるわね!
0105Usagi2023/03/27(月) 01:54:16.390
ところで、らすかるさんの計算力と根性はほんと凄いわね
アタシは一生かかってもできない計算ね
あなた、らすかるさんを利用するだけしておいて
>>102みたいな解き方があることを教えてあげないのはちょっとひどいんじゃない?
0106陽気な名無しさん2023/03/30(木) 20:03:47.680
「1または2は素数である。」

真か偽か?
0108陽気な名無しさん2023/03/30(木) 22:25:51.790
いま映画公開記念でセーラームーンSuperSがyoutubeで毎日一話ずつ無料配信されているの。
第3話を観ていたら、育子ママのレモンパイをみんなで分けるシーンの会話が気になったの。

まもる「10等分って難しいんだよな」
亜美 「えっ、360を10で割るんだから、一切れを36度にすれば…」
美奈子「やめて、食べる時に数字の話は」

10等分するには2等分して5等分すればいいのよね。
角の2等分は定規とコンパスで作図できるけど、5等分はできたかしら?って思ったの。
角θが与えられた時、θ/5 を作図できることは cos(θ/5) を作図できることと同値なのね。
cosθ = 16cos^5(θ/5) − 20cos^3(θ/5) + 5cos(θ/5) だから
これは5次方程式 16x^5−20 x^3+5x = cosθ を作図で解くことに等しいけれど
定規とコンパスで作れるのは加減乗除と平方根だけだから、一般的にこれは無理なのね。
と思いきや、この場合 36度 = π/5ラジアン だから話が違うの!
cosπ = −1 だから、方程式は 16x^5−20x^3+5x+1 = 0 となるけど
16x^5−20x^3+5x+1 = (x+1)(4x^2−2x−1)^2
だから、加減乗除と平方根だけで解けるからこれは作図できるのね。
さすが天才少女亜美ちゃんだわ! 美奈子が話を遮らなければ

「一般に角の5等分は定規とコンパスで作図できないけれど
 36度 = π/5ラジアン で、5はフェルマー素数だから作図可能なの」

とか言ったに違いなかったわ。

細かいことだけど、美奈子みたいに数字(numeral)を数(number)と混同するのはやめて欲しいわね!
0109陽気な名無しさん2023/03/30(木) 22:26:47.680
ところで調べたら、定規とコンパスでは一般に2次方程式までしか解けないけれど
なんと折り紙を使うと3次方程式と4次方程式まで解けるんですって。
さらに、2つの折れ目を同時に作るテクニックを許容すると、折り紙で任意の角の5等分を作図できるそうよ。
langorigami.com/article/angle-quintisection/
langorigami.com/wp-content/uploads/2015/09/quintisection.pdf
リンク先に図があるわ。https://をつけてね。
0110陽気な名無しさん2023/03/31(金) 05:27:07.760
「1または2は素数の部分集合である。」

真か偽か?
0111陽気な名無しさん2023/03/31(金) 13:39:25.170
>>110
これはそもそも日本語として通じなくて意味不明よ。英語にすると
1 or 2 is a subset of ( ). の最後に何が入るの?

もし the set of prime numbers が入るなら
「1または2は素数の集合の部分集合である」と書くべきで、これは偽

もし a prime number のつもりなら「1または2はある素数の部分集合である」と書くべきよ
フォン•ノイマンによる自然数の集合論的定義を仮定すればこれは真だけど
他の定義もあるから真とは言い切れないわね

前スレ842の問題文なんかは、2通りの解釈があるために数学の問題としては不適切ってだけで
日本語としては何もおかしくなかったけど、今回のは違うわ。
日本語として通じないの。したがって命題ではなく真偽も議論できないわ。
0112陽気な名無しさん2023/04/02(日) 13:44:09.540
「1または2は素数である。」
を真だと解釈する人って、例えば整数なんかの話で
3の倍数または4の倍数は偶数である
とか出てきたらどう読むの?
真?

1以上3以下の偶数または10以上20以下の奇数は素数である

これも真?
0113陽気な名無しさん2023/04/02(日) 14:37:52.390
ここは数学のスレなんだから、
数学の問題っぽく見せても結局
日本語の問題になるようなことを
書き込むんじゃないわよ!
0114陽気な名無しさん2023/04/04(火) 00:40:01.540
そもそも「1または2は素数である」って解釈ひとつしかないわよね?
真だと解釈する人とか限定しているの意味わからないわ
0115陽気な名無しさん2023/04/06(木) 18:09:54.370
くだらない書き込みは無視して、スレタイ通り
大学入試の数学の問題に戻りましょ。
今年の京大の文系の問題だそうよ。

3の実3乗根をtとするとき、
55/(2t^2+t+5)
の分母を有理化せよ。
0116 【36.1m】 2023/04/06(木) 22:13:25.240
55/(2t^2+t+5)
=55t/((t+3)(t+2))
=3*5*11/(t+3)-2*5*11/(t+2)
=11/2*(t^2-3t+9)-10(t^2-2t+4)
=-9t^2/2+7t/2+19/2
0117陽気な名無しさん2023/04/06(木) 22:41:23.580
>>116
すごいわね。どうしたらそういう発想が出てくるのか教えてほしいわ。

ところで、セーラームーンSSの第9話観たら、亜美が衛に数学の勉強を教わってて、問題が映ったの。

〔練習問題80〕下の図の△ABCにおいて, ⦣B = 33°, ⦣C = 67° である.
2辺AB, BCの中点をそれぞれP, Qとし, AからBCにひいた垂線の足をDとする.
このとき, ⦣DPQの大きさを求めなさい.

〔練習問題83〕座標平面上の3点A(0, 12), B(-16, 0), C(x,0)を頂点とする三角形ABCがある.
⦣A = 90°として, 次の問いに答えなさい.
(1) xの値を求めよ.
(2) 頂点Aより角Cの外角の二等分線lに下した垂線の足をHとするとき, 点Hの座標を求めよ.
(3) 直線lの方程式を求めよ.
(4) 中心Dが辺BC上にあり, 辺, A[…] に接する半円をかくとき, 半円の半径はいくらか.
(この […] の部分が見切れてるけど, 何が入るのかしら?
「AB, AC」かしら? 「円」じゃなくて「半円」なのが謎よね)

衛が〔練習問題83〕を指差しながら
衛 「つまり、問題はここの二次関数なんだ」
亜美「あ、そっかぁ、先に空間における相対関係を考慮すれば良かったんですね」
って会話してたんだけど、問題の内容と会話合ってるのかしら?w

その様子を見ていたうさぎの手元にあった問題がこれ↓

〈例題2〉
xの二次関数 y = x^2 (x ≧ 0) のグラフと, 直線 y = x のグラフとは, 原点(0, 0)とP(1, 1)を共有する.
いま, y = x^2 (x ≧ 0) 上に点(0, 0), (1, 1)以外の点Q(a, a^2)をとり, 直線 y = x に関する点Qの対称点をRとする.
(1) 点Rの座標は ( , ) である.
(2) △PQRが正三角形になるのは a = ( ) のときである.
(一部見切れてるけど、おそらくこうね。穴埋め式の問題みたいね)

どの問題も難しくない?
アタシ、〔練習問題83〕はすごく時間かけてなんとか解けたわ
〈例題2〉は解き方を考えたけど、中学の知識で解けるのか分からないわ
そして〔練習問題80〕はさっぱり分からないわ!
中学の数学って難しいのね。これ大学受験生にやらせても解けない人の方が多そうじゃない?
姐さん方は解けるかしら?
0118ageterandkp2023/04/06(木) 23:08:45.670
80は簡単過ぎるでしょ
0119 【19.8m】 286페이지 47번2023/04/06(木) 23:17:47.710
🔺ABDは直角三角形なのでAP=BP=DP
よって🔺PBDは二等辺三角形 ∴∠PDQ=33°
PQ//ACなので∠PQB=67°
したがって∠DPQ=∠PQB-∠PDQ=34°
01201152023/04/06(木) 23:46:44.510
>>117
>>116 は足しかに見事な解法だと思うわ。
でもこの問題、いろいろな解き方があって、
自分の好みや自分のレベルに見合った解き方をすればいいのよ。
分母が因数分解できれば部分分数にわけなくても有理化は可能だし、
因数分解しないで有理化することもできるし、
因数分解しない場合でも公式を知ってる場合の少し楽な方法もあれば、
そうでない力業で解く方法もあるし。

解法がたくさんあるというだけでも、この問題は良問だと思うわ。
>>166 ほどエレガントでなくても、別解にチャレンジしてくれる人はいないかしら。
0121Usagi2023/04/07(金) 01:50:06.310
やだ、アタシ間違ってメール欄に名前書いてたわw

>>119
あら、素晴らしいわ。全然気づかなかったわ
そうか、PからBDに下ろした垂線の足をEとすると
△BEPは△BDAに相似で、BP=PA だから BP : BA = 1 : 2
したがって BE : BD = 1 : 2 なので BE=ED
なので△BEPと△DEPは合同な直角三角形だから BP=DP となるのね?
って思ったけど、△BEPと△DEPが合同な時点で ∠PDQ = ∠PBE だと分かるわね。

AP=BP=DPってすぐ分かったのは、タレスの定理の逆を使ったってこと?
△ABDが直角三角形だから、DはABを直径とする円の周上にある
PはABの中点だからこれは円の中心
したがって AP=BP=DP=(円の半径) ってことかしら?
難しいわw
0123Usagi2023/04/07(金) 09:33:03.810
アタシの感覚だと
「3の倍数または4の倍数は偶数である」=「3の倍数は偶数であるか、または4の倍数は偶数である」
偽と読むことはできないわ。
もし「3の倍数は偶数であり、かつ4の倍数は偶数である」という解釈が欲しいなら
「3の倍数または4の倍数であるものは偶数である」
「3または4の倍数は偶数である」
「3の倍数、そしてまた4の倍数は偶数である」
などと書くべきよ。
「カメまたはウサギは哺乳類である」を偽と思う人っていないと思うわ?
0124Usagi2023/04/07(金) 11:09:14.840
>>120
55t/((t+3)(t+2)) = 55t(1/(t+2) - 1/(t+3)) と分解して
1/(t+2)の分子分母に(t^2-2t+4)を掛けて、1/(t+3)の分子分母に(t^2-3t+9)を掛けるのもあるわね

部分分数にわけないというのは、55t/((t+3)(t+2))の分子と分母に
直接(t^2-3t+9)(t^2-2t+4)を掛けるということかしら?

他に考えたのは、55/(2t^2+t+5) = at^2+bt+c (a, b, c ∈ ℚ) と一通りに表せるから
55 = (2t^2+t+5)(at^2+bt+c)
= 2at^4 + (a+2b)t^3 + (5a+b+2c)t^2 + (5b+c)t + 5c
t^3 = 3だから
55 = (5a+b+2c)t^2 + (6a+5b+c)t + (3a+6b+5c)
係数を比較して
5a+b+2c = 0
6a+5b+c = 0
3a+6b+5c = 55
この連立方程式を解くと a = -9/2, b = 7/2, c = 19/2 と出せるわ。
けどアタシ計算苦手だから途中で間違えまくって大変だったわw

それから上で使った定理の証明に戻ってユークリッドの互除法を試してみたわ
3の3乗根の最小多項式 t^3-3 と 2t^2+t+5 に互除法を使うと
t^3-3 = (1/2t-1/4)(2t^2+t+5) + ((-9/4)t-7/4)
2t^2+t+5 = (-(8/9)t+20/81)((-9/4)t-7/4) + 440/81
だから
440/81
= (2t^2+t+5) - (-(8/9)t+20/81)((-9/4)t-7/4)
= (2t^2+t+5) - (-(8/9)t+20/81)((t^3-3) - (1/2t-1/4)(2t^2+t+5))
= (-(4/9)t^2+(28/81)t+76/81)(2t^2+t+5) + ((8/9)t-20/81)(t^3-3)
ここで t^3-3 = 0 だから
440/81 = (-(4/9)t^2+(28/81)t+76/81)(2t^2+t+5)
したがって
1/(2t^2+t+5) = (81/440)(-(4/9)t^2+(28/81)t+76/81)
55/(2t^2+t+5) = (81/8)(-(4/9)t^2+(28/81)t+76/81)
= -(9/2)t^2 + (7/2)t + 19/2
はー、この計算も死ぬ程しんどかったわw

他にどんな方法があるのかしら? 公式って何? 受験生の標準的な解き方は?
0125陽気な名無しさん2023/04/07(金) 12:43:51.970
>>124
さすがね、いろんな解法考えたわね。
>直接(t^2-3t+9)(t^2-2t+4)を掛ける
それも想定していたわ。それから
>55/(2t^2+t+5) = at^2+bt+c (a, b, c ∈ ℚ) と一通りに表せるから
これは高校の範囲では証明されていないから、
このように表せるとしたら、みたいな表現にしていく方が受験的にはいいのではないかしら。
一応この解法も想定していたわ。
ユークリッドの互除法使う方法は想定していなかったわ。
なるほどその方法も(計算のしんどさは別として考え方としては)いいわね。

あとあたしが想定していたのは、
分母分子t倍すると分母が因数分解できることに気づかない場合の、愚直な方法よ。
分母分子ナントカ倍してパッときれいに有理化することは難しそうだから、
とりあえずtの次数を下げられないかしら、という発想。
分母分子に(at+b)の形のものをかけて分母をtの一次式にまずは下げてみよう、というもの。
t^3は3で置き換えられるからt^2の項が消えるようなa,bを考えると、
(-2t+1)を分母分子にかけると分母がtの一次式になることがわかるわ。
それで与式が55(2t-1)/(9t+7)と変形出来て、
あとは公式(a+b)(a^2-ab+b^2)=a^3+b^3を利用してa=9t, b=7として有理化できるわ。

その他に公式を使う方法としては、ちょっとハイレベルな受験生なら知ってる公式、
a^3+b^3+c^3-3abc=(a+b+c)(a^2+b^2+c^2=ab-bc-ca)
を使って、a=2t^2, b=t, c=5として分母分子(a^2+b^2+c^2=ab-bc-ca)倍して一発で有理化する方法。
計算が楽かどうかは別として、一発で有理化できるという点では面白い方法だと思うわ。

とりあえずあたしが想定していた方法はそんなところよ。
0126陽気な名無しさん2023/04/07(金) 12:48:25.920
打ち間違いよ。わかるわよね。
a^3+b^3+c^3-3abc=(a+b+c)(a^2+b^2+c^2=ab-bc-ca)
ではなくて
a^3+b^3+c^3-3abc=(a+b+c)(a^2+b^2+c^2-ab-bc-ca)
に訂正よ。
0127 【利用料 103238 ₲】 554ページ 89番2023/04/07(金) 20:42:56.760
55/(2t^2+t+5)
=55/((t^2+1)(t+2))
=(t^4-t^2+1)/2*(t^2-2t+4)
=(-t^2+3t+1)(t^2-2t+4)/2
ここで終わっちゃダメなのかしら?
分母を有理化ってどういうことなのかしら?
分子も整理しなければならないのかしら?
分母の有理化という語句の中に分子の整理もする必要があるという意味が含まれてるのかしら?
0128Usagi2023/04/07(金) 23:37:49.080
>>125
なるほど、いろいろな工夫が考えられるのね、面白いわ〜
最後の公式を使うやり方は、ちょうどうまくt^6とt^3の項と定数項だけになるのね
こんなのなかなか思いつかないわね

>>127
言葉の意味だけ考えれば分子の整理はしなくてもいいはずだけど
実際はしないと減点されそうな予感
0129【B:78 W:60 H:88 (A cup) 151cm/51kg age:26】 695페이지 24번2023/04/15(土) 08:57:12.630
一年のうちのある日が月曜日である確率はだいたい1/7だろう

ある整数が5の倍数である確率が1/5だといってもそれほど不思議ではない

自然数nの最大の素因数が√nより大きい確率ってどれくらいなのかしら?
0130陽気な名無しさん2023/04/22(土) 01:38:15.770
>>129
とりあえず小さい数からいくつか見てみた限りでは2/3くらいっぽいけど、
本当はもっと沢山見た上で予想を立てて、
その予想がきちんと証明出来なければならないのよね。
そこまでやる気力はねーわ。

誰かやってくんねーかしら。
0131Usagi2023/04/22(土) 05:15:56.460
あたしに解けるわけないからググってみたけど、どうやら log 2 になるらしいわよ
https://math.stackexchange.com/questions/375270/size-of-largest-prime-factor
あたしには理解できないけどね。これ↓も参考になるらしいわよ
https://dms.umontreal.ca/~andrew/PDF/msrire.pdf
どうやらKarl Dickmanという謎の人物の1930年の論文の定理によるらしいわ
これもう専門家じゃなきゃ無理ってレベルの話でしょw

そんなことよりセーラームーンの残りの問題は誰も解いてくれないの?
ちなみに練習問題83の(4)の見切れてた部分はやっぱり「AB, AC」なんだと思うわ
「半円」なのは、特に意味ないけどきっと三角形の内部に収めるためね
0132Usagi2023/04/22(土) 05:18:29.470
Dickmanって「ちんぽマン」ってことかしらw
0133陽気な名無しさん2023/04/22(土) 05:52:36.520
>>131
log2ってずいぶん小さいのね。
20〜30くらいやってみた感じとえらい違いだわ。
数が大きくなるとどんどん確率下がってくるのかしら。
それにしても大学入試の問題とは全然レベルが違いすぎるから、
この問題はスレ違いね。
このスレに出すなら、せめて100くらい試してみて予想が立てられて、
あとは帰納法で証明できるくらいの問題じゃないとダメよね。

セーラームーンのは図を描かなきゃならなさそうで面倒だわ。
暇なときがあったらやってみようかしら。
0134Usagi2023/04/22(土) 09:33:46.590
>>133
log 2 ≈ 0.69 だから、上に書かれている2/3はかなりいい線いってるんじゃないの?
0135陽気な名無しさん2023/04/22(土) 10:08:48.620
>>134
log2でググって最初に出てきた値を鵜呑みにしてたけど、
よく見てみたら常用対数だったわ。
本当は、というか普通底が省略されてたら自然対数よね。
自然対数で調べ直したらあなたの書いてくれた値だったわ。

てゆーかグーグル!底省略したときに常用対数を先頭に出すんじゃないわよ!
0136 【猿】 2023/04/23(日) 00:57:54.750
そんなに難しいはずはないと思うが……
アタシが読んでる本の2章の演習問題だから
0137陽気な名無しさん2023/04/23(日) 06:59:56.600
なんて本よ?
洋書の専門書とかじゃないでしょうね?
0138陽気な名無しさん2023/04/25(火) 02:48:58.36S
26歳の芦屋市長。

ここの3兄弟すごすぎだわ

長男は灘高で成績がトップで生徒会長してて
東大とハーバードどっちも受かって、
東大に4ヶ月だけ冷やかしで通って中退して
ハーバード大学卒業
三男も今、灘高で生徒会長してる
次男も甲陽学院からカナダの
ブリティッシュコロンビア大学に留学

https://aya02082101.com/takasimaryousuke-brother/
0139陽気な名無しさん2023/04/25(火) 04:45:16.260
周りの無能さに耐えられなくなって豊田真由子のように失墜すると思うわ
0140陽気な名無しさん2023/04/25(火) 09:34:51.540
>>131
セーラームーンの残りの問題ってどれかしら、と思って読み返してみたわ。
〔練習問題80〕は>>119 さんが解いてるでしょ。
〔練習問題83〕は>>117 で「すごく時間かけてなんとか解けたわ」って言ってるわよね。
〈例題2〉は同じく117 で「解き方を考えたけど、中学の知識で解けるのか分からないわ」って言ってるわ。

残りの問題ってどれかしら?「〈例題2〉が中学の知識で解けるか?」かしら?
だとしたら、アタシの解き方では、「センスのいい中学生なら解ける」だわ。
PQ^2 = (a-1)^2+(a^2-1)^2でしょ。
QR^2 = 2(a^2-a)^2でしょ。
PQ^2 = QR^2 を満たすaを求めればいいのよね。
センスのいい中学生なら、PQ^2 も QR^2 も(a-1)^2を因数に持つことに気づき、
a=1の時は正三角形にならないから両辺(a-1)^2で割ればいい事はわかると思うわ。
そうすると aについての二次方程式になるから、あとは解の公式で解けるわ。

個人的には〔練習問題83〕(4) が一番面倒だったけど、
うさぎがそんなに苦しむほどの問題かしら?って思ったわ。
0141Usagi2023/04/25(火) 18:00:53.120
>>140
〈例題2〉の解き方はなるほどね〜
その方法は、4次方程式になって因数定理を知らないなら無理じゃない?とか思ってたんだけど
たしかに展開しないで因数分解すれば上手くできるわね。にしても中学生にはかなりハードル高そうよ
ちなみにあたしが最初に考えた解き方はこう
y = x とx軸のなす角が45° でPQとy = x のなす角度が30°だから
PQとx軸がなす角度は75°でその傾きは
tan 75° = (tan 45° + tan 30°)/(1 − tan 45° tan 30°) = (1+1/√3)(1−1⋅1/√3) = 2+√3
したがってPQの式は y = (2+√3)(x−1)+1 となるわ
これと y = x^2 を連立させると x = 1, 1+√3 とでるから a = 1+√3 とわかるわ
(もうひとつのx軸となす角度が15°の直線は y = x^2 と x ≧ 0 で交点を持たないわ)
この解き方も三角関数の加法定理を使ってるから中学の範囲じゃないわねって思ってたの

〔練習問題83〕は、みんなはどう解くのかしらって思ったの
あたしは(2)で苦労したの。一方(4)は、あることに気付いたら一瞬でできたわ
姐さんはどう解いたの?
0142陽気な名無しさん2023/04/25(火) 20:02:06.250
だって基本的にこの問題に出てくる三角形は3:4:5の三角形でしょ。
AC=15なんだからlに関してAに対称な点は(24,0)でしょ。
そしたらHはAと(24,0)の中点なんだから(12,6)でしょ。
全然難しくないと思ったけど。
あたし何か勘違いしてるかしら?
0143陽気な名無しさん2023/04/25(火) 20:23:23.080
うさぎって中学受験しなかったのね
01441422023/04/25(火) 20:41:25.390
>>143
あなたの書き込み、何か刺があるわよ。
アタシも中学受験はしてないけど。
0145Usagi2023/04/25(火) 21:06:50.720
>>142
AHとx軸の交点をEとすると、△HCAと△HCEが合同ってことなのよね
でもアタシそれに気づくのに時間かかったのよw
そしてなるほどね、HはAとEの中点だからすぐわかるわね
アタシはEを求めてからも、Hからx軸に下ろした垂線の足をFとして
△EHFと△EAOが相似だとか面倒くさく考えてたわw
(4)はどうやって解いたの?

>>143
それがね、アタシ中学受験してたのw

んもう、アタシがセンスないのバレちゃったわね(知ってたかw)
AHを伸ばしてx軸との交点を考えるとか、発想力よね
幾何はセンスが要るからアタシは苦手みたい
でも解けると面白いから好きなの❤
0146陽気な名無しさん2023/04/25(火) 22:56:34.930
>>145
DからAB,ACに垂線下ろすと正方形が出来るのよ。
あたしもう寝るから詳しくは明日にでもまた書き込むわね。
0147陽気な名無しさん2023/04/26(水) 07:11:14.090
お待たせしました。
(4)をどうやって解いたか書き込むわよ。
A[…] の部分はAB, ACとして解くわね。

>>146 でも書いたけど、DからAB,ACに垂線下ろすの。
それぞれの足をP,Qとでもしようかしら。
そうすると∠APD=∠AQD=90°で仮定より∠PAQ=90°
だから四角形APDQは長方形なんだけど、
DP=DQ=半径(=r)だからこの四角形は正方形になるわ。
それでDP=AP=rであり、AP+PB=AB=20だからPB=20-AP=20-r
一方三角形BDPは3:4:5の三角形なのでDP:PB=3:4
つまりr:(20-r)=3:4なので、あとはこの比例式を解いてr=60/7

最初いろいろ考えた時は面倒に感じたけど、
整理するとかなり簡単ね。
うさぎの「一瞬でできた」という解法はどんなのかわからないけど、
この解法もなかなかシンプルでしょ?
0148Usagi2023/04/26(水) 15:22:18.900
>>147
ありがとう。面倒どころかすごいエレガントじゃない!
たぶんこれが模範解答なんでしょうね。
アタシも同じ補助線引いてたけど、正方形になってるの気づかなかったわw

そしてアタシ問題をちょっと勘違いしてたわ。
アタシDの座標を求めて満足してたんだけど、半径をきかれてたのね。
アタシが考えてたのはこうよ。中学の範囲じゃないかもだけど。
Dと直線ABの距離 = 半径 = Dと直線ACの距離 だから
Dは、直線ABと直線ACからの距離が等しい点の集合、つまり角Aの二等分線上にあるの。
つまりADは角Aの二等分線よ。
したがって角の二等分線の定理から BD : CD = AB : AC = 4 : 3 となって
BD = BC × (4/(4+3)) = 25 × (4/7) = 100/7 とわかるの。
アタシここで終わりにしてたんだけど、ここで姐さんの解法を参考にすると
半径 = PD = BD × (3/5) = (100/7) × (3/5) = 60/7 と計算できるわ。

ちなみに(2)でHの座標を求めるのが(3)を解くための誘導になっているけど
角の二等分線の定理を使えば他のやり方で(3)を解くこともできるわ。
直線BAの延長線とIの交点をGとすると、外角の二等分線の定理から
BG : AG = CB : CA = 5 : 3
BA = 20 だから BG = 20 × (5/(5-3)) = 50
したがって
Gのx座標 = BG × (4/5) − BO = 50 × (4/5) − 16 = 24
Gのy座標 = BG × (3/5) = 50 × (3/5) = 30
これで直線Iは C(9, 0) と G(24, 30) を通る直線としても式を求められるわ。
0149陽気な名無しさん2023/04/26(水) 22:12:23.780
あたしも最初Dの座標求めるのと勘違いしたわ。
二等分線の性質は多分中学でやってたと思うけど。
外角の方はちょっとマイナーだと思うけど。
0150陽気な名無しさん2023/04/27(木) 08:33:45.240
じゃあ大学入試問題いくわよ。

n^3-7n+9が素数となる整数nをすべて求めよ。(京大)
0151陽気な名無しさん2023/04/27(木) 12:27:15.810
簡単すぎんか
n^3-n-6n+9
でn^3-nが6の倍数だから素数になるとしたら=3
ということなんだろう?
0152陽気な名無しさん2023/04/27(木) 19:13:56.940
大学入試の問題のスレで京大の問題を簡単すぎって、
ならもっと難しい大学入試の問題を出してみてよ。
大学入試の問題にチャレンジする人たちにとってはこれでも難しいんだから。
やる気を削ぐようなこと言わないで!
0153陽気な名無しさん2023/04/27(木) 19:32:12.830
わかったわ!!
どこの大学か知らないけど!

nは正の整数、x,yは整数で
|x^2-(n^2+1)y^2|
が平方数ではないものとする
このとき、
|x^2-(n^2+1)y^2|≧2n
が成り立つことを示せ
0154陽気な名無しさん2023/04/29(土) 01:19:22.810
どこの大学か知らないけど、って、
そもそもこれって本当に大学入試の問題なの?
0155陽気な名無しさん2023/04/29(土) 16:31:53.690NIKU
それより誰か京大の問題解いてくれないかしら?
0156陽気な名無しさん2023/04/30(日) 05:43:02.310
>>153
本当にそんなこと成り立つの〜?
反例がすぐ見つかるんじゃないの〜?
と思いながらいじっていたら出来ちゃったわ。

要するにx^2と(n^2+1)y^2の差が、
平方数か又は2n以上であることを示せればいいのよね。

まず条件を満たす任意のy, nに対して
{(n+1)y}^2≧(n^2+1)y^2≧(ny)^2
が成り立つのは、パッと見だけでも明らかだし、
きちんと示すのもすぐにできるわ。
だからx^2と(n^2+1)y^2の差が最小になるのは
x=(n+1)yの時か又はx=nyの時になることがわかる。

case1:x=(n+1)yの時
|x^2-(n^2+1)y^2|=2ny^2
これはy=0の時平方数で、そうでない時2n以上。

case2:x=nyの時
|x^2-(n^2+1)y^2|=y^2
これは平方数。

以上でx^2と(n^2+1)y^2の差が、
平方数か又は2n以上であることが示されたわ。
0157陽気な名無しさん2023/04/30(日) 07:16:19.550
不正解
0158陽気な名無しさん2023/04/30(日) 07:37:10.81S
>>157
不正解の場合は、どこが間違えてるって指摘しなきゃダメよ。
0159陽気な名無しさん2023/04/30(日) 07:48:44.560
>だからx^2と(n^2+1)y^2の差が最小になるのは
>x=(n+1)yの時か又はx=nyの時になることがわかる。


この箇所が恐ろしいくらい根拠不明
0160陽気な名無しさん2023/04/30(日) 07:57:12.79S
>>159
なるほどね。
それであなたはその根拠不明なことが「恐ろしい」のね。
せっかく正しい指摘しても変な煽りすると印象悪いわよ。
0161陽気な名無しさん2023/04/30(日) 08:08:00.48S
きっと早起き姐さんかその他の誰かが修正解答考えてくれるのではないかしら。
あたしは京大の問題にでもチャレンジしてみようかしら。
GWだし。
0162陽気な名無しさん2023/04/30(日) 09:01:43.920
修正出来るのかしら?
根本的、致命的なミスに思えるわ
0163Usagi2023/04/30(日) 11:43:03.820
なんだ、150と153 は別人なのね? なんかおかしいと思ったわ。

細かく場合分けしてみたの。
まず、y = 0 なら |x^2 - (n^2+1)y^2| = x^2 が平方数になるから y ≠ 0 ね。

(1) |x| = n|y| の場合
|x^2 - (n^2+1)y^2| = y^2 が平方数になるからこれは関係ないわ。

(2) |x| < n|y| の場合
x^2 - (n^2+1)y^2 ≤ (n|y|-1)^2 - (n^2+1)y^2 = -y^2 - 2n|y| + 1 = -(|y|+n)^2 + n^2 + 1
だけど、|y| ≥ 1 だから右辺は負になるわ。したがって
|x^2 - (n^2+1)y^2| - 2n = (|y|+n)^2 - n^2 - 1 - 2n = (|y|+n)^2 - (n+1)^2 ≥ 0
したがって |x^2 - (n^2+1)y^2| ≥ 2n

(3) |x| > n|y| の場合
Y = |y|, |x| = nY+k (k ≥ 1), そして
f(Y) = x^2 - (n^2+1)y^2 = (nY+k)^2 - (n^2+1)Y^2 = -Y^2 + 2nkY + k^2
とおくわ。
すべての Y≥1 について |f(Y)| が平方数でないなら |f(Y)| ≥ 2n であることを示すわ。

(3a) Y = 2nk の場合
|f(Y)| = |f(2nk)| = k^2 が平方数になるからこれは関係ないわ。

(3b) Y < 2nk の場合
Y ∈ [1, 2nk-1] だけど、f(Y)はYに関する2次関数でその軸のY座標は Y = nk なので
[1, 2nk-1] のちょうど真ん中に軸があるのね。したがって
f(Y) ≥ f(1) = f(2nk-1) = 2nk + k^2 - 1 ≥ 2nk ≥ 2n だから |f(Y)| ≥ 2n

(3c) Y > 2nk の場合
f(Y) ≤ f(2nk+1) = -2nk - k^2 - 1 < -2nk ≤ -2n だから |f(Y)| ≥ 2n

これでできてるかな。もっと簡単にできるのかしら?
つーか難しすぎよ。誘導なしで大学入試で出題されるのは想像できないわ。
0164陽気な名無しさん2023/04/30(日) 12:48:06.55S
だからきっと大学入試問題じゃないのよ。
ウソツキなのよ。
0165Usagi2023/04/30(日) 21:31:10.810
やだ、見直したら(3c)で計算間違いしていたから解けてなかったわ。
f(2nk+1) = -2nk + k^2 - 1 だったわね。これほんと難しいわ!
0166陽気な名無しさん2023/04/30(日) 22:42:39.290
前スレか前々スレかそれよりも前か忘れたけど
数学的な間違いを指摘されたらいきなり
「アタシ彼氏いるし」
って言い出した人いたわよね

あれかなり気持ち悪かったわ
0167陽気な名無しさん2023/04/30(日) 22:56:04.790
そんな話題いきなり持ち出してくるあんたが気持ち悪いわよ!
0168陽気な名無しさん2023/04/30(日) 23:02:47.430
いきなりっていうか
>>156-160見たら思い出してしまったのよ
そんなやつもいたな、って
0169陽気な名無しさん2023/04/30(日) 23:38:42.740
数学スレで彼氏いないアピールされても困るわ
0170陽気な名無しさん2023/05/01(月) 00:06:04.180
わかったわかったw
なにか言っちゃいけないこと言ったみたいねアタシ
ごめんなさい
0171Usagi2023/05/02(火) 12:39:49.620
>>168
そもそも156と158は別人でしょ? 末尾が違うじゃない

ていうか153は釣りだったってアタシやっと気づいたの
これはペル方程式がらみの話で大学受験生に解けるような問題じゃないんでしょ?
大学入試問題だと偽って156みたいに挑戦した人を馬鹿にしてほくそ笑んでるのかしら?
単純に不快だわ
0172【B:78 W:75 H:94 (A cup) 153cm/58kg age:18】 139페이지 74번2023/05/02(火) 20:42:11.470
意外と難しいわね
元ネタはなんなのかしら

アタシも考えてみて一応派手に解けたんだけど、
なんだかもっと他の解き方がありそうで
>>163を読もうとしてみたものの場合分けと計算を追うのに挫折…

というわけで他の掲示板で聞いてみたわ
https://www2.rocketbbs.com/11/bbs.cgi?id=yosshy&mode=res&resto=85359
らすかるさん教えてくれるかしら?
教えてくれるといいんだけど…
0173陽気な名無しさん2023/05/02(火) 23:03:08.070
なんだか、大学入試レベルを超える問題を出すのも違うし、
それを他のサイトで質問するのも違うと思うわ。
ここのスレは大学入試レベルの問題をここの人たちだけで解くべきではないかしら。

大丈夫よ、本当に大学入試レベルだとしたら必ずうさぎが解けるから。
うさぎが解けないのは結局大学入試レベルを超えてるんだと思うわよ。
0174陽気な名無しさん2023/05/05(金) 12:25:04.1500505
結局>>150 の京大の問題は誰も解かないの?
0175Usagi2023/05/07(日) 12:38:12.970
>>172
何よ、カチカチ山ってw 失礼しちゃうわ!
派手に解けるってどういう意味かわからないけど、解けたなら解法を書き込みなさいよ💢

>>174
161さんが解かないみたいだから、あたしが解くわね
といっても151さんがほとんど解いているわよね
151さんの言っているのは
n^3-7n+9 = n^3-n-6n+9 = (n-1)n(n+1) - 3(2n-3)
で(n-1)n(n+1)も3(2n-3)も3の倍数だから、与式が素数になるとしたら3でしかない
したがって
n^3-7n+9 = 3
n^3-7n+6 = 0
(n-1)(n-2)(n+3) = 0
だから n = 1, 2, -3 が答えね
n^3-7n+9 が3の倍数であることを示すには、nを3で割った余りで分類しても良いわ

でも標準的な解答って何なのかしら?
151さんの方法は思いつけば簡単だけど、思いつかなければできないし
nを3で割った余りで分類ってのも、どうして3で考えるのか謎だし
論理的思考力というより思いつきを要求する問題って、入試問題としてどうなのかしらって思うのよね
だってそれってただのパズルじゃない?
0176Usagi2023/05/07(日) 12:52:44.190
じゃあ、解いたついでにアタシがとっておきの問題出していい?
大学入試問題じゃないけど、高校までの知識で解けるわ
ただ難易度はどの大学入試問題よりも上ね

円周上にn個の点をとって、そのうちどの2点の間も直線で結ぶとき
円の内部が最大で何個の領域に分割されるか、その個数をnで表せ
0177陽気な名無しさん 293페이지 15번2023/05/07(日) 17:02:09.950
2019年の産業医大の大問2の(7)を検索して見てみてね
一般項はnC4+nC2+1になるわ

その年の産業医大は大問3も面白そうね
0178陽気な名無しさん2023/05/10(水) 07:51:21.350
産業医大の大問3面白かったわ。
27ね。

でも>>177 は出典を教えてくれたことは感謝するけど、
なんの説明もなく答えだけ書くのってどうなのかしら?
このスレ的にはOKなのかしら?
とりあえず真似させてもらって答えだけ書いたけど、
本当にこのスレこれでいいの?
0181陽気な名無しさん2023/05/10(水) 12:28:58.980
>>175
標準的な解答というか、
取っ掛かりがない場合はいくつかの数で試してみるのは鉄則ではないかしら。
そしたら「あら、どれも3の倍数だわ」って気づくでしょ。
気づけばあとはmod3で考えるなりすれぱいいと思うわよ。

>>180
他のサイト頼るのやめない?
出題も解答も解説もスレの中で完結させましょうよ。
0182陽気な名無しさん2023/05/13(土) 17:00:53.860
なんだか東大や京大の問題でも簡単すぎるってすぐ言われちゃうし、
でもスレタイ的に高校数学の範囲内でなければならないと思うし、
それで思いついたのが数学オリンピックの問題なんてどうかしら?
一応数学オリンピックの問題は高校数学の範囲内よね?
いくわよ。

p+q=(p-q)^3
を満たす素数の組(p, q)を全て求めよ。

どうかしら?これでもまだ簡単かしら?
0183陽気な名無しさん2023/05/13(土) 17:43:20.060
2q=(p-q)^3-(p-q)≡0 (mod6)
∴q=3
p^2-3^2=(p-3)^4
ピタゴラス数には5の倍数が含まれる
もしもp-3が5の倍数なら(p-3)^4≧625
隣接する平方数との差は9よりはるかに大きい
∴p=5

5+3=(5-3)^3
0184陽気な名無しさん2023/05/14(日) 07:27:14.630
>>183
あなた面白い解き方するわねえ。
一行目は与式の両辺からp-qを引いたのね。
三行目はちょっと考えちゃったけど、
与式にq=3を代入したものの両辺に
p-3を掛けたのね。
それで四行目、
ピタゴラス数には5の倍数が含まれるの?
本当に?なぜ?
って思ったけど、ピタゴラス数の一般式を
mod5で見たら納得できたわ。
でも、これって一般的な高校生が
知っているレベルの知識かしら?
確かに間違いはなさそうだけど、
もう少し一般的な高校生の知識の範囲で
何とかならないものかしら?
0185陽気な名無しさん2023/05/14(日) 12:32:20.100
んま、この年老いたアタシに高校生ぽく解けとおっしゃるの?

しょうがない、やってみましょう
高校生風にフェルマーの小定理を使えばいいわね

p^2-3^2=(p-3)^4
p-3が5の倍数だとするとp+3は5の倍数ではないから
両辺の素因数分解の5の指数が符合しない
∴p^2-3^2≡1 (mod5)←フェルマーの小定理
∴p=5
0186陽気な名無しさん2023/05/15(月) 16:57:45.080
フェルマーの小定理は高校の範囲じゃないから
0187【B:78 W:59 H:90 (A cup) 143cm/63kg age:44】 953페이지 91번 (ワッチョイW 838a-Mrcq)2023/05/15(月) 19:27:40.07ID:ubwqpQw+0
アタシも問題出すわ
某国の国際数学オリンピック選手選抜に使われた問題だって
日本でいうところの本選なのかしら?まさか予選?
元の問題は難しいから少しだけシンプルにしてみたわ

ωを1の虚数3乗根とする
複素数から複素数への関数fが、任意の複素数aと任意の複素数b≠0に対して
f(a+b)+f(a+bω)+f(a+bω^2)=0
を満たしているという
このとき、任意の複素数zに対して
f(z)=0
であることを示せ
0188Usagi2023/05/15(月) 22:53:04.940
与えられた条件は、複素数平面上で任意の正三角形を考えたとき
その3頂点でのfの値を合計すると0になるってことよね。
zを任意の複素数として、zを重心に持つ正三角形ABCを考えるわ。
各辺を3等分した点の間に、各辺に平行になる直線だけを引くと
正三角形ABCは合同な9個の正三角形に分割されて、Zは3本の線が交わる点になるわね。
辺ABを1:2に内分する点をP、辺ACを1:2に内分する点をQとすると
△APQと△zPQは正三角形だから、条件から
f(A) + f(P) + f(Q) = 0 = f(z) + f(P) + f(Q)
したがって f(A) = f(z) となるわ。同様に f(B) = f(C) = f(z) よ。
最後に正三角形ABCに条件を使うと 0 = f(A) + f(B) + f(C) = 3f(z)
したがって f(z) = 0 ね。
0189陽気な名無しさん2023/05/15(月) 23:11:42.790
うさぎ凄いわさすがだわ。
0192Usagi2023/05/16(火) 20:09:05.18F
>>190
あら、ふつうに褒めてくれるのね。ありがと
元の難しいバージョンってどんなだったのかしら

ていうか、あなたカチカチ山姐さんよね
187を解いたご褒美に、あなたの153の解法を教えてくれないかしら
らすかるさんも解けないみたいだしやっぱり難問なのよ
0193Usagi2023/05/16(火) 20:32:47.65F
>>191
姐さん、ふだんどんな文章書いてるのかしらw
この問題、もっとシンプルに、どんな複素数 z, w に対しても f(z) = f(w) であることを示してもいいのよね
なぜって、平面上のどんな2点の間にも、正三角形ふたつを貼り合わせてできるひし形を
角度60度の2頂点がその2点となるように描くことができるから
あとは188と同じ考え方で f(z) = f(w) がわかるのよね
ポイントは、2点を頂点に持つふたつの正三角形が、その2点以外の頂点を共有しているってことよね
正方形だと同様の議論は作れないから、正方形でも成り立つのかは分からないわ

けど、3次元に拡張することはできるわね
3次元空間から複素数(または実数)への関数fが、任意の正四面体ABCDに対して
f(A) + f(B) + f(C) + f(D) = 0 を満たしているなら、任意の点Pに対して f(P) = 0 が言えるわ
0194 522페이지 28번 (ワッチョイW 838a-Mrcq)2023/05/18(木) 04:03:50.11ID:x9Y1uISy0
元の問題は正n角形なのよ


ζを1の原始n乗根とする
複素数から複素数への関数fが、任意の複素数aと任意の複素数b≠0に対して
f(a+b)+f(a+bζ)+f(a+bζ^2)+…+f(a+bζ^(n-1))=0
を満たしているという
このとき、任意の複素数zに対して
f(z)=0
であることを示せ
0195Usagi2023/05/18(木) 06:31:08.090
>>194
あら、そうなのね!
上に正方形とかだとよく分からないって書いたけど、ちょっと考えてみるわ
0196Usagi2023/05/18(木) 21:16:51.420
今日電車に乗りながら思いを巡らせていたら閃いたわ
zをひとつの頂点とする正n角形を考えるの
他の頂点を A_1, …, A_{n-1} とするわ
この正n角形を z を中心に 2kπ/n 回転させることを考えるの
このとき各頂点 A_i が移る先を A_i(k) で表すことにするわね(A_i(0) = A_i よ)
回転後、元の正n角形のn個の頂点は z, A_1(k), …, A_{n-1}(k) に移るけど、これらも正n角形を作るから
f(z) + f(A_1(k)) + … + f(A_{n-1}(k)) = 0 よね
この式の k = 0 から k = n-1 までの総和をとると
nf(z)
+ f(A_1(0)) + f(A_1(1)) + … + f(A_1(n-1))
+ f(A_2(0)) + f(A_2(1)) + … + f(A_2(n-1))
+ …
+ f(A_{n-1}(0)) + f(A_{n-1}(1)) + … + f(A_{n-1}(n-1))
= 0
となるわ。ところが各 i について
A_i(0), A_i(1), …, A_i(n-1)
は A_i を z を中心に 2π/n ずつ回転させたものだから、これらは正n角形を作るから
f(A_i(0)) + f(A_i(1)) + … + f(A_i(n-1)) = 0 となるの
したがって、上の式は nf(z) = 0 となって f(z) = 0 が結論されるわ

正三角形とか正方形の場合は、つい正三角形や正方形に特有の性質を考えてしまうから
こういう一般的な議論は思いつきにくいのね
0198陽気な名無しさん2023/05/19(金) 22:19:43.670
このように回転させてみようなどとよく思いつくものだけ
0199 620페이지 45번 (ワッチョイW f38a-YC6R)2023/05/20(土) 11:41:29.27ID:x77OwKA/0
n∈ℤ, x∈ℤ, y∈ℤ が
|x² - (n²+1)y²| < 2n
をみたしているとき
|x² - (n²+1)y²|
は平方数であることを示せ


この方が問題として綺麗かしらね
アタシの好みなだけだけど
0202陽気な名無しさん2023/06/08(木) 01:00:00.320
問題を拵えてみたの。
完全に大学受験レベルだけど、誰か解けるかしら?
解いてくれたら嬉しいわ。


問題.
nを1より大きな奇数とし、
a=(1/2)(√n+1/√n)^2
とする。不等式
(x-1)(a-[x])>[x](x-[x])
をみたす実数xの範囲を求めよ。
[x]はx以下最大の整数を表している。
0203Usagi2023/06/10(土) 15:13:42.720
難しいわね
一応解けたけど、場合分けが多くてなんかスッキリしないの

まず a = 1 + n/2 + 1/(2n) で n は1より大きい奇数だから、a は 2より大きく、整数でない数ね
もし [x] ≤ 0 なら x-1 < [x] ≤ 0 かつ a-[x] > x-[x] ≥ 0 だから (x-1)(a-[x]) ≤ [x](x-[x]) となってダメね
もし [x] > a なら (x-1)(a-[x]) < 0 ≤ [x](x-[x]) だからこれもダメ
したがって 1 ≤ [x] < a が必要なの
ここで d = x-[x] とおくと、問題の不等式は
([x]+d-1)(a-[x}) > [x]d
つまり
(2[x]-a)d < ([x]-1)(a-[x])   (★)
と書き換えられるの

・1 ≤ [x] < a/2 の場合
2[x]-a < 0 かつ 0 ≤ d < 1 だから(★)の左辺は0以下、右辺は0以上だから、(★)が成立

・a/2 < [x] < a-1 の場合
1 < a-[x]
⇔ [x] < [x](a-[x])
⇔ 2[x]-a < ([x]-1)(a-[x])
となるけど、2[x]-a > 0 かつ 0 ≤ d < 1 だから (2[x]-a)d ≤ 2[x]-a < ([x]-1)(a-[x]) となって(★)が成立

・a-1 < [x] < a の場合
このとき [x] = (n+1)/2 だから、これを(★)に代入して整理すると
{(n^2-1)/2n}d < (n^2-1)/4n となり、これは d < 1/2 と同値
したがって x = [x]+d < (n+1)/2 +1/2 = n/2 + 1

以上をまとめると 1 ≤ [x] < n/2 + 1

はー大変だったわ。もっと簡単に解けるのかしら?
0204Usagi2023/06/10(土) 15:24:14.910
あ、最後書き間違ったわ
1 ≤ x < n/2 + 1 が答えよ
0205陽気な名無しさん2023/06/11(日) 01:25:00.490
うさぎ、あなたってほんと素晴らしい実力者よね。
x=1は入らないでしょう。
でも、もうほぼ正解といっていいわよね。
0206陽気な名無しさん2023/06/11(日) 01:41:12.290
全く場合分けせずに解く方法もあるわ。

(x-1)(a-[x])>[x](x-[x])
(x-1)(1+(n+1/n)/2-[x])>[x](x-[x])
(x-1)((n+1/n)/2-[x-1])>([x-1]+1)(x-1-[x-1])
y=x-1とおくと
y((n+1/n)/2-[y])>([y]+1)(y-[y])
y(n+1/n)/2-y[y]>y-[y]+y[y]-[y]^2
[y]^2-2y[y]+[y]-y+y(n+1/n)/2>0
nが3以上の奇数なのであらためてnを自然数として2n+1とおきなおすと
[y]^2-(2y-1)[y]-y+y(2n+1+1/(2n+1))/2>0
記号が錯綜するけどy=a/2とおくと
[a/2]^2-(a-1)[a/2]-a/2+a/4 (4n^2+4n+2)/(2n+1)>0
[a/2]^2-(a-1)[a/2]+an^2/(2n+1)>0 …(★)
f(x)=x^2-(a-1)x+an^2/(2n+1)とおくと、y=f(x)の頂点のx座標は(a-1)/2で、
[a/2]は(a-1)/2に最も近い整数だから、(★)が成り立つということは、
すべての整数mに対してf(m)>0が成り立つということよね。

と、いうわけでこれは大昔の東大入試の問題なのよ。
http://server-test.net/math/php.php?name=tokyo&v1=1&v2=1997&v3=1&v4=2&y=1997&n=2
https://www.youtube.com/watch?v=2XzOe34FPn0
https://ameblo.jp/mathisii/entry-12352941501.html

どこ見ても下手糞な解き方しか載ってなかったわ。
アタシが昔使ってた問題集にはもっと簡単な解き方が載ってたのに。
0207Usagi2023/06/12(月) 01:56:14.570
>>205
あらやだ!
そうね、 [x] = 1 かつ d = 0 のときは除かなきゃいけなかったわ

元ネタの東大の入試問題難しすぎね
東大受験生でも本番でこれ解けた人きっとほとんどいないわよね
日本の大学って高校生に何を求めてるのかしらw
0208陽気な名無しさん2023/06/12(月) 15:44:02.400
これ証明してよ

0=∞=1
0209陽気な名無しさん2023/06/12(月) 23:53:45.220
東大入試は差がつかないでしょうね
みんな同じようなところができて、同じようなところができない
でも女子入学者が増えるからこの難しさでいいらしい
0210Usagi2023/06/13(火) 02:15:43.650
>>209
どういうこと? 難しくてみんなができないからってこと?
でも難易度下げた方が女子の割合って増えるんじゃないかしら
IQの分布でも、極端に高い人や低い人の割合は男の方が大きくて、女は平均付近に集まるっていうし
日本で東大とかの女子率が異様に低いのって必要以上に難しい入試制度のせいよ
アメリカなら有名校でも女子率もっと高いっていうか、男女同じくらいなことが多いんじゃないかしら
0211陽気な名無しさん2023/06/13(火) 21:20:10.520
数学で差がつかないと国語や英語が得意な女子が入りやすくなる
0212陽気な名無しさん2023/06/14(水) 14:32:12.070
x=f[n](θ) (n=1,2,3,...),
y=g[n](θ) (n=1,2,3,...)
はみな実数から実数への関数で、周期2πをもち、C^∞級であるものとする。
さらに、n→∞のとき、f[n](θ)はcosθへg[n](θ)はsinθへ一様収束するものとする。
このとき、
lim[n→∞]∫[0→2π]√((dx/dθ)^2+(dy/dθ)^2)dθ
は2πになるだろうか?
0213陽気な名無しさん2023/06/14(水) 15:35:41.62S
東大だと今年の物理は流石にやりすぎよね。数学に関しては最近は共通テストが難易度高めで国公立二次が易化傾向で差が付かないのよね。2010年くらいの難易度に落ち着くと力の差がはっきりしそうね。
0214Usagi2023/06/14(水) 18:48:45.79F
>>212
ならないでしょ
単位円周に沿って細かく波打ちながら一周する曲線を考えるの
nが大きくなるとき、波打つ幅も波同士の間隔も小さくなるようにするの
波打っているからこの曲線の長さは2πより大きいわ
曲線上の点のx軸からの角度がθの時のx座標とy座標をf[n](θ), g[n](θ)とすれば
f[n], g[n] はn→∞のときcosθとsinθにそれぞれ一様収束するわ
問題の積分は曲線の長さを表しているけど、nが大きくなるときに
波同士の間隔が十分に小さくなっていくように設定すれば、曲線の長さが短くならないようにできると思うわ
具体的な式とか計算は勘弁して

こういう問題を出すのはやめてほしいわ
ふつうのオカマが楽しく取り組める問題にしてほしいの
0216Usagi2023/06/19(月) 01:44:14.020
じゃ、あたしがふつうのオカマが楽しく取り組める問題出すから、よければ誰か解いてみて。

A子、B子、C子の3人はD子と友達になりました。3人がD子に誕生日をきくと、D子は

「2001年の2/17、3/13、4/13、5/15、6/17、
 2002年の3/16、4/15、5/14、6/12、8/16、
 2003年の1/13、2/16、3/14、4/11、7/16、
 2004年の1/19、2/18、5/19、7/14、8/18 のうちのどれかよ」

と答えました。そしてD子は、A子に月だけを、B子に日だけを、C子に年だけを教えました。
すると3人が次のように会話を始めました。

A子「あたしはD子の誕生日がいつなのか分からないけど、B子が分からないことも分かるわ」
B子「あたしもまだ分からないわ」
C子「あらそう?」
B子「そうよ。でもC子、あんたがそれを聞いてもまだ分からないことも分かるの」
C子「あたしもまだ分からないけど、A子がまだ分からないことも分かるわ」
A子「じゃあ分かったわ」
B子「あたしも分かった」
C子「あたしも〜」

さてD子の誕生日はいつでしょうか。
0217Usagi2023/06/19(月) 03:13:47.130
読み直したら会話の日本語がおかしかったから訂正するわ。

A子「あたしはD子の誕生日がいつなのか分からないけど、B子も分からないことは分かるわ」
B子「あたしもまだ分からないわ」
C子「あらそう?」
B子「そうよ。でもC子、あんたがそれを聞いてもまだ分からないことはあたし分かるの」
C子「あたしもまだ分からないけど、A子もまだ分からないことは分かるわ」
A子「じゃあ分かったわ」
B子「あたしも分かった」
C子「あたしも~」
0218陽気な名無しさん2023/06/21(水) 11:24:46.000
「2001年の2/17、3/13、4/13、5/15、6/17、
 2002年の3/16、4/15、5/14、6/12、8/16、
 2003年の1/13、2/16、3/14、4/11、7/16、
 2004年の1/19、2/18、5/19、7/14、8/18

A子「あたしはD子の誕生日がいつなのか分からないけど、B子が分からないことも分かるわ」

B子が分からないことがA子に分かるということは、上の中で日が1つしか現れない月はA子には教えられなかったということ
例えば、もしもA子に4月が教えられていたら、誕生日は
2003 4/11、2001 4/13、2002 4/15
の可能性があり、万が一11日がB子に教えられていればB子は誕生日が特定できるわけだから、B子も分からないと主張出来ない
したがって、4月と6月は除外される

「2001年の2/17、3/13、5/15
 2002年の3/16、5/14、8/16、
 2003年の1/13、2/16、3/14、7/16、
 2004年の1/19、2/18、5/19、7/14、8/18

B子「あたしもまだ分からないわ」
C子「あらそう?」

B子が分からないということは、B子に教えられたのは15日と17日ではないということ

「2001年の3/13
 2002年の3/16、5/14、8/16、
 2003年の1/13、2/16、3/14、7/16、
 2004年の1/19、2/18、5/19、7/14、8/18

B子「そうよ。でもC子、あんたがそれを聞いてもまだ分からないことも分かるの」

C子がまだ分からないということが分かるということは、B子に教えられたのが13日ではないということ
もしもB子に13日が教えられ、C子が2001年を教えられていた場合C子もまだ分からないとは言えない

 2002年の3/16、5/14、8/16、
 2003年の2/16、3/14、7/16、
 2004年の1/19、2/18、5/19、7/14、8/18

C子「あたしもまだ分からないけど、A子がまだ分からないことも分かるわ」

A子がまだ分からないことが分かるということは、2004年ではないということ
もしもA子に1月が教えられていた場合A子は誕生日が特定できてしまう
A子が絶対に分からないことがC子に分かるということは、C子に教えられたのは2004年ではない

 2002年の3/16、5/14、8/16、
 2003年の2/16、3/14、7/16、

A子「じゃあ分かったわ」

これは3月ではないということ

 2002年の5/14、8/16、
 2003年の2/16、7/16、

B子「あたしも分かった」

これは16日ではないということ

 2002年の5/14
0219陽気な名無しさん2023/06/21(水) 11:52:13.950
C子「アタシもまだ分からないけど、A子もまだ分からないことは分かるわ」
A子「じゃあ分かったわ」
B子「んまっ、いやらしい!アタシまだ分かんないわ。C子、アンタは?」
C子「アタシも分かっちゃった♡」
B子「ブス!」

これでもいいのかしら?
0220Usagi2023/06/21(水) 19:17:32.59F
>>218
姐さん完璧よ! すばらしいわ。

これは「シェリルの誕生日」っていう、2015年の14歳向けのシンガポールとアジアの数オリの問題があって、
それの発展バージョンとして作られた「デニーズの仕返し」っていう問題なの。
https://en.wikipedia.org/wiki/Cheryl%27s_Birthday#Second_sequel:_%22Denise's_Revenge%22

元の問題文では、B子が
「あたしもまだ分からないけど、C子もまだ分からないことは分かるわ」
って言ってるんだけど、この文の後半部分が問題作成者の意図した効果を発揮するためには、
ここが解釈される時点で文の前半部分の内容がC子の知識に入っていないといけないから、
上の会話では、B子の発言を分けて、C子がまず前半を理解していることをはっきりさせたの

>>219
なるほどね、それもアリね! その場合、C子がわかったことによって結局B子もわかるわね。
設定をうまく変えれば、ひとりだけ誕生日がわからないで取り残される問題も作れそうね。
0221陽気な名無しさん2023/07/01(土) 16:00:06.070
. . *
* * . * . * . *
. * . * . .
. . *
* * . * . * *
. * . 💭 * . .
________________🚶🏻‍🚶🏻‍♂_________

整数 a,b,c が |a^2-b^2-2abc|<2c を満たしている。

abc が偶数であることを示せ。
0222陽気な名無しさん2023/07/12(水) 23:25:25.26S
il||li (OдO`) il||li恐ろしいスレを開いてしまったわ
0223陽気な名無しさん2023/07/12(水) 23:32:51.030
ようこそ
0225陽気な名無しさん2023/07/22(土) 09:25:14.960
書き込めたわ。

それでは東大2023年文系第一問いくわね。



k を正の実数とし、2次方程式 x^2+x-k=0 の二つの実数解を a, b とする。

k が k>2 の範囲を動くとき、

a^3/(1-a) + b^3/(1-b)

の最小値を求めよ。
0226陽気な名無しさん2023/07/22(土) 11:47:36.890
a^3/(1-a)+b^3/(1-b)
=-a^2-a-1+1/(1-a)-b^2-b-1+1/(1-b)
=-2k-2+3/(2-k)
→-∞ (k→+∞)
∴最小値はない
0227陽気な名無しさん2023/07/22(土) 12:07:54.400
あらごめんなさい、出題ミスだわ。

a^3/(1-a) + b^3/(1-b)

ではなくて、

a^3/(1-b) + b^3/(1-a)

だったわ。
0228陽気な名無しさん2023/07/31(月) 08:41:09.390
解く人いないわねえ
0229陽気な名無しさん2023/08/01(火) 12:45:37.830
うさぎはtalkに行ったのかしら?
0230Usagi2023/08/01(火) 21:57:59.41F
いるわよ! Talkには行っていないわ
やっぱりJane Styleの乱のせいで書き込める人減っちゃったのかしら?

根と係数の関係 a+b = -1, ab = -k を使うと
a^3/(1-b) + b^3/(1-a)
= {(1-a)a^3 + (1-b)b^3} / (1-a)(1-b)
= {a^3+b^3 - (a^4+b^4)} / {1 - (a+b) + ab}
= (2k^2 +7k + 2)/(k-2)
= 2k + 11 + 24/(k-2)
= 15 + 2(k-2) + 24/(k-2)
ここで k > 2 だから、2(k-2) と 24/(k-2) は正だから相加相乗平均の不等式から
与式 ≥ 15 + 2√{2(k-2)・24/(k-2)} = 15 + 8√3
等号が成立するのは 2(k-2) = 24/(k-2) のときで、k > 2 だから k = 2 + 2√3 のときね
これで合ってるかしら?

東大にしては簡単すぎない? >>206の問題とはえらい違いね
0231陽気な名無しさん2023/08/03(木) 13:09:49.330
本来このスレはこのくらいのレベルがちょうどいいと思うわよ。
これより難しくなると、うさぎや一部の専門家っぽい人しかできなくなっちゃう。
0232陽気な名無しさん2023/08/03(木) 13:22:48.400
よかった!うさぎいたのね
やっぱり鋭い論客がいないとスレって盛り上がらないのよね
Twitterスレにくまちゃんがいるようにこのスレにはうさぎがいてくれないと
0233Usagi2023/08/03(木) 22:54:38.260
くまちゃんがどういうコテなのか知らないからそこはわからないけど、そんなふうに言っていただいて嬉しいわ
でもアタシばっか解いててもスレが盛り上がっているとは言えないから、他の人もどんどん解いてほしいわ
5chすっかり過疎っちゃったからしょうがないかしらね
0234陽気な名無しさん2023/08/03(木) 23:17:18.470
だから、東大にしては簡単、位のレベルがちょうどいいのよ。
東大の難しい方のレベルになると、もううさぎしか解く人いなくなるかもよ。
0235Usagi2023/08/04(金) 02:14:17.180
>>234
簡単って書いたのは、別にもっと難しいの出して、て意味じゃなかったの
でもこれだったら解ける人いっぱいいそうなのに誰も解かなかったからやっぱり人少ないのねって思ったわ
もっと簡単な小中学校レベルの話でもアタシは歓迎なの
難易度よりも興味深さよね
最近のニュースで小6が三角形の面積の問題で正答率が2割ってのがあったわ
www.asahi.com/articles/ASR7X56R8R7WUTIL015.html

問題文には、テープの上の直線と下の直線は並行って書いてあるんだけど
でもこれ結構難しいっていうか意地悪よね
3辺すべての長さが書かれているんだもの
大人にやらせてもどれだけできるかわからないわ
0236陽気な名無しさん2023/08/04(金) 07:20:21.120
>>235
これ底辺以外見る必要ないじゃないの。
普通の数学の問題は余計な情報は書かれていないことが多いから
それに慣れている人から見たらこの問題は意地悪に見えるかもしれないけど、
この問題はそもそも必要な情報を取捨選択できるかを問うてる問題だと思うわよ。

ちなみに3辺の長さが矛盾していないか計算してみたけど、正しかったわ。
テープの幅は3cmね。
0237陽気な名無しさん2023/08/04(金) 08:26:38.790
自然数nに対して、3^nを2^nで割った余りをa[n]とする。
数列{a[n]}(n=1,2,3,...)は有界か?


メルカリで見つけた問題。
0238陽気な名無しさん2023/08/04(金) 18:30:18.360
はあ……。
アタシまた難しすぎるだなんだ言われて怒られちゃうのね……。

憂鬱。
0239Usagi2023/08/04(金) 19:44:09.450
>>236
>ちなみに3辺の長さが矛盾していないか計算してみたけど、正しかったわ。
アタシもまずそこが気になって調べたのよ。
余弦定理とか知らない小学生だったら自分で確認できないから、すごくもやもやして問題を意味不明に感じると思うし
そもそも未知数を未知数のまま扱う代数的思考を要求しているから小学生にはハードル高いと思うの
こんなのできない今の小学生ヤバいとか言われてるんだけど、たぶん大人にやらせても正答率高くないと思うのw
0240Usagi2023/08/04(金) 21:37:12.430
>>237
メルカリで見つけたってどういうこと? これ大学入試問題なの?

もし有界であると仮定すると、ある自然数mがあって、すべてのnに対して a[n] ≤ m となるわ。
3^n を 2^n で割った商の整数部分を q[n], 小数部分を d[n] とおくと
(3/2)^n = q[n] + d[n] そして 3^n = q[n] 2^n + a[n] だから d[n] = a[n]/2^n よ。
a[n] ≤ m だから n→∞ のとき d[n] ≤ m/2^n → 0
したがってある自然数 N があって n ≥ N であるすべての n に対して d[n] < 1/3 が成り立つわ。

ここで s を奇数として q[N] = (2^k)s と表したとき、すべての i ≥ 0 に対して、i ≤ k ならば
q[N+i] = (3/2)^i q[N]
d[N+i] = (3/2)^i d[N]
が成り立つことを帰納法で示すわ。
i = 0 のときは明らかね。
i < k のときは、帰納法の仮定から
(3/2)^{N+i} = q[N+i] + d[N+i] = (3/2)^i q[N] + (3/2)^i d[N]
だから
(3/2)^{N+i+1} = (3/2)^{i+1} q[N] + (3/2)^{i+1} d[N]
だけど、i+1 ≤ k だから (3/2)^{i+1} q[N] は整数、そして
(3/2)^{i+1} d[N] = (3/2) d[N+i] < (3/2)(1/3) = 1/2 < 1 だから
q[N+i+1] = (3/2)^{i+1} q[N]
d[N+i+1] = (3/2)^{i+1} d[N]
となって帰納法完了ね。

このことから
q[N+k] = (3/2)^k q[N] = (3/2)^k (2^k)s = (3^k)s
(3/2)^{N+k} = q[N+k] + d[N+k] = (3^k)s + d[N+k]
だけど (3^k)s は奇数だから
(3/2)^{N+k+1} = (3/2)((3^k)s - 1) + 1/2 + (3/2) d[N+k]
において (3/2)((3^k)s - 1) は整数、そして
1/2 + (3/2) d[N+k] < 1/2 + (3/2)(1/3) = 1
だから
q[N+k+1] = (3/2)((3^k)s - 1)
d[N+k+1] = 1/2 + (3/2) d[N+k]
となるわ。ところが 1/2 + (3/2) d[N+k] > 1/3 だから
d[N+k+1] > 1/3 となってこれは矛盾よ。
したがって {a[n]} は有界ではないわ。
0241陽気な名無しさん2023/08/04(金) 22:36:11.470
>>240
>(3/2)^{N+k+1} = (3/2)((3^k)s - 1) + 1/2 + (3/2) d[N+k]

これってどういう計算されてんですか?


何か良い問題がないかメルカリで模試や問題集を探してて見つけた問題。
数学オリンピックの問題だと思います。
0242Usagi2023/08/04(金) 23:54:14.560
ごめんなさい、そこ間違っていたわ。その上の式
(3/2)^{N+k} = (3^k)s + d[N+k]
の両辺に 3/2 をかけて
(3/2)^{N+k+1} = (3/2)(3^k)s + (3/2) d[N+k] = (3/2)((3^k)s - 1) + 1 + 1/2 + (3/2) d[N+k]
だから
q[N+k+1] = (3/2)((3^k)s - 1) + 1
d[N+k+1] = 1/2 + (3/2) d[N+k]
としなければいけなかったわ。
もっと簡単に解けるのかしら?
0244陽気な名無しさん2023/08/05(土) 09:44:21.510
うさぎはいつも問題解いてくれる優しい人だから>>153,>>221の出典も明かしておこうかねえ。
どう思う?今さら知りたい?
探すのに少し時間かかるけど…
0245Usagi2023/08/06(日) 14:06:15.340
>>243
なにやらマニアックな問題集あるのね
画像の右側が解答になっていたことに、ついさっきまで気付かなかったわ
気付いたけど読めないし欠けてるw
読めない記号があると思ったら ord っていう記号があるのね?
https://ja.wikipedia.org/wiki/指数_(初等整数論)
数学オリンピックって中高生向けじゃないの?
アタシも知らなかった記号だけど参加者には常識なのかしら
この模範解答を復元して理解するのにはちょっと時間がかかりそうだわ
でも割と複雑で、思いつきが必要そうな解答っぽいわね
正直、アタシもし「余りが10^2021より大きいことがあるか」って出題されてたら
「なにその謎の数は?」って思って解けなかったと思うわ
あなたの出題の「有界か」って聞かれ方だと、解答を書きあげるのにはそれなりに時間かかったけど
解き方の方針は割とすぐ思いついて、シャワー浴びながら「たぶんこんな感じでできそうだわ〜」って思ったわ
「有界である」の方が強い命題なのに、そっちの方が解きやすいって不思議よね

>>244
そうね、出典も知りたいけど、何より解き方教えてほしいわ
アタシの他にも問題に取り組んでみた人はみんな知りたがってると思うわ
時間がある時に書き込んでくれたら嬉しいわ
0246Usagi2023/08/06(日) 14:23:09.470
誤 「有界である」の方が強い命題なのに
正 「有界でない」の方が強い命題なのに
0247Usagi2023/08/06(日) 20:15:20.610
アタシちょっと早とちりしたみたい
ordは上のwikipediaに出てくるものとは違うみたいね
心眼使ってみたら、解答の一行目で ord を定義しているみたくて、おそらく

正の整数nに対し、nが2^kで割り切れるような最大の非負整数kを ord_2 n と書く

とか書かれているようだわ。疲れた。これ以上心眼使うの無理w
0248陽気な名無しさん2023/08/09(水) 20:48:12.850
ordの使い方なんてその時それぞれ定義するんじゃないの。
wikiでどう書いてあるのかは知らないけど。

その前にそもそも有界って概念自体が高校の範囲超えてるんじゃない?
0249陽気な名無しさん2023/08/09(水) 20:50:08.120
>>153,>>221って、解けそうで解けないって感じだから、
かなり良問だと思うんだけど、
出典は興味深いわ。
0250陽気な名無しさん2023/08/09(水) 20:56:45.940
アタシ今日家帰る途中に本屋寄って
受験生でもないのに最新の受験情報チェックしてたんだけど
今年の京大理系の1番の問2って相当出来悪かったのね

文系の1の問2はこのスレでも話題になったけど
簡単そうに見えるのに(実際アタシたちみたいな大人には簡単)実は受験生には解けない問題出すのって
なかなか出題のセンスがあるんじゃないかしら
0251Usagi2023/08/10(木) 15:36:18.690
>>250
それ>>23にあるやつね?

整式 x^2023 - 1 を整式 x^4 + x^3 + x^2 + x + 1 で割ったときの余りを求めよ.

>アタシたちみたいな大人には簡単
そうなの? 京大受験生にとって難しいなら世の中の9割以上の大人は解けないと思うけど

アタシが考えた解き方は
x^2023 - 1 = (x - 1)(x^2022 + x^2021 + … + x + 1)
で右側のかっこの中身を x^4 + x^3 + x^2 + x + 1 で割ると x^2 + x + 1 余るから、
(x - 1)(x^2 + x + 1) = x^3 - 1 が答えになるかしら?
他の解き方もあるの?
0252Usagi2023/08/10(木) 16:16:18.130
x^2023 - 1 - (x^3 - 1)
= x^3 (x^2020 - 1)
= x^3 (x^{5⋅404} - 1)
= x^3 (x^5 - 1)(x^{5⋅403} + x^{5⋅402} + … + x^{5⋅2} + x^5 + 1)
で x^5 - 1 が x^4 + x^3 + x^2 + x + 1 で割り切れることから
x^2023 - 1 を x^4 + x^3 + x^2 + x + 1 で割った余りは x^3 - 1 である
というふうにも解けるわね。
でもこれは答えがわかってたからできただけで最初から思いつくのは難しいわ
0253陽気な名無しさん2023/08/11(金) 08:04:56.520
整式 x^2023 - 1 を整式 x^4 + x^3 + x^2 + x + 1 で割ったときの余りを求めよ.

こーいうのって、普通に高校でやった考え方だと、
x^2023 - 1 =( x^4 + x^3 + x^2 + x + 1 )Q(x)+ax^3+bx^2+cx+d
って置くんじゃないの?
それでx^4 + x^3 + x^2 + x + 1が0になるものを代入するのよね。
1の原始5乗根をζとおくと、これ及びこれの2乗、3乗、4乗が該当するから、
代入すると
ζ^3 -1=aζ^3+bζ^2+cζ+d
ζ -1=aζ+bζ^4+cζ^2+d
ζ^4 -1=aζ^4+bζ+cζ^3+d
ζ^2 -1=aζ^2+bζ^3+cζ^4+d
これすべて満たすようなa, b, c, dを求めればいいのよね。
って、見た瞬間にa=1、b=c=0,d=-1が満たすことはわかるから、
余りは x^3 - 1 って解くのが普通の高校生の解法ではないかしら。
でも高校生でζなんて使わないかしら。
0254Usagi2023/08/11(金) 14:12:43.040
そうね、そのやり方はめんどくさそうだからアタシは考えなかったんだけど
実際にやってみると、最後の連立方程式の答えを見た瞬間に思いつければできるのね
思いつけないと計算地獄になるけどw
「普通の」高校生の解法じゃない気がするわ
ネットで検索してみたらいろいろ出たけど、これ↓が数学的で良いと思ったわ
https://akiyamath.com/2023/03/kyoto_2023-1/
0255陽気な名無しさん2023/08/11(金) 16:23:21.540
>>254 のリンク先の方法って、
>>253 の方法と基本的な方針は同じだけど、
革新的に洗練させたって感じ。
0256陽気な名無しさん2023/08/11(金) 20:40:35.340
アタシたち数学好きの大人には、って意味ね
>>253のようにやって身動きとれなくなった受験生が多かったらしい
>>251-252のような柔軟な式変形が案外出来ない、ってことなのよね
0257陽気な名無しさん2023/08/12(土) 13:40:35.130
nを自然数とする。1個のさいころをn回投げ、出た目を順に
X[1],X[2],……,X[n]
とし、n個の数の積X[1]X[2]……X[n]をYとする。
p,qはどちらも1以上6以下の整数とする。
Yがpで割り切れるという事象とYがqで割り切れるという事象が
独立となるためのp,qに関する必要十分条件は何か?

>>23の3番を見てふと気になったこと
0258陽気な名無しさん2023/08/12(土) 14:43:14.000
と、いうか
独立となるためのn,p,qに関する必要十分条件は何か?
と質問すべきだったわ
0260陽気な名無しさん2023/08/12(土) 18:16:13.070
あ、n=1は別か…。
0261陽気な名無しさん2023/08/12(土) 22:10:39.340
違うか!
京大も独立ではないね。
勘違いごめんなさい!!
0262陽気な名無しさん2023/08/14(月) 07:20:26.380
>>257

>>23 の3番は難しくないのに、
こうやって一般化して独立性まで考えるとなると
とたんに難しくなるわね。
難しくなるというか、確率の問題で独立性について
あまり扱いなれていないからそう感じるのかしら。

どちらにしても独立について復習するいい問題ね。
0263陽気な名無しさん2023/08/14(月) 08:23:47.150
アタシ案外差がついたんじゃないかという気がするわ
本番中に(2)を3で割り切れる確率×5で割り切れる確率とした人けっこういそうな気がするの
0264陽気な名無しさん2023/08/14(月) 10:05:58.80S
(2)は、
3でも5でも割りきれない確率と、
3で割りきれる確率、5で割りきれる確率
を足して、1を引けばいいんでしょ?
0265Usagi2023/08/15(火) 02:02:06.500
>>264
それでいいと思うわ
がんばって表作ってみたんけど、うまく表示されるかしら?

         | 5で割り切れる | 5で割り切れない |
—————————————————————————————
3で割り切れる  |    ア    |    イ     |
—————————————————————————————
3で割り切れない |    ウ    |    エ     |
—————————————————————————————

15で割り切れる確率 = ア
= エ + (ア + イ) + (ア + ウ) - (ア + イ + ウ + エ)
= 3でも5でも割り切れない確率 + 3で割り切れる確率 + 5で割り切れる確率 - 1

となるわね。でも次のように考えた方が簡単かしら?

15で割り切れる確率 = ア
= 1 - (イ + ウ + エ)
= 1 - ((ウ + エ) + (イ + エ) - エ)
= 1 + エ - (ウ + エ) - (イ + エ)
= 1 + 3でも5でも割り切れない確率 - 3で割り切れない確率 - 5で割り切れない確率
= 1 + (3/6)^n - (4/6)^n - (5/6)^n
0266Usagi2023/08/15(火) 03:07:56.270
>>257
とりあえず、pとqのいずれも1または素数の場合は、
s = 1から6までの数の中でpを約数に持たないものの割愛
t = 1から6までの数の中でqを約数に持たないものの割愛
とおいたとき、
1から6までの数の中でpもqも約数に持たないものの割愛 = st
となることが必要十分条件でいいかしら?

上の表が p = 3, q = 5 のケースと思ってもらうと

Yがpでもqでも割り切れる確率 = ア
= 1 - (ウ + エ) - (イ + エ) + エ
= 1 - Yがpで割り切れない確率 - Yがqで割り切れない確率 + Yがpでもqでも割り切れない確率
= 1 - s^n - t^n + (st)^n
= (1 - s^n)(1 - t^n)
= Yがpで割り切れる確率 × Yがqで割り切れる確率

となるから、Yがpで割り切れる事象とYがqで割り切れる事象は独立になるわ。

逆にこれが成立するためには、

Yがpでもqでも割り切れない確率 = (st)^n

でなくてはいけなくて、これはつまり

1から6までの数の中でpもqも約数に持たないものの割愛 = st

であるということになるわ。

例えば、p = 3, q = 5 のときは
s = |{1, 2, 4, 5}|/|{1, 2, 3, 4, 5, 6}| = 4/6
t = |{1, 2, 3, 4, 6}|/|{1, 2, 3, 4, 5, 6}| = 5/6
1から6までの数の中で3も5も約数に持たないものの割愛
= |{1, 2, 4}|/|{1, 2, 3, 4, 5, 6}| = 3/6 ≠ (4/6)×(5/6)
だから条件が満たされないわ。

p = 3, q = 2 のときは
s = |{1, 2, 4, 5}|/|{1, 2, 3, 4, 5, 6}| = 4/6
t = |{1, 3, 5}|/|{1, 2, 3, 4, 5, 6}| = 3/6
1から6までの数の中で3も2も約数に持たないものの割愛
= |{1, 5}|/|{1, 2, 3, 4, 5, 6}| = 2/6 = (4/6)×(3/6)
だから条件が成立して独立になるわ

pやqが合成数、つまり4や6の場合は複雑になりそうね。
0267陽気な名無しさん2023/08/15(火) 07:59:21.640
割愛ってなんかステキね。
割合を愛しているのね。
0268陽気な名無しさん2023/08/15(火) 10:13:40.000
割愛ってふつうはキャッツアイと読まないかしら
0269陽気な名無しさん2023/08/15(火) 10:45:53.460
>>266
なるほど…
p,qは素数でm≧pqのとき
(m-[m/p])(m-[m/q])=m(m-[m/p]-[m/q]+[m/(pq)])
すなわち
[m/p][m/q]=m[m/(pq)]
が成り立てばm面体サイコロで
Yがpで割り切れることとYがqで割り切れること
が独立になるのかしら

p,qが素数でもなくてよいなら
[m/p][m/q]=m[m/lcm(p,q)]
なのかしら?
0270陽気な名無しさん2023/08/15(火) 10:51:04.930
気軽に質問しといてなんだけど、案外難しそうね
ごめんなさいね

整数mが与えられたとき
[m/p][m/q]=m[m/(pq)]
を満たす素数p,q(p≠q,pq≦m)は存在するか判定出来たりするのかしら

アタシにはちょっと難しすぎるわ…
0271Usagi2023/08/16(水) 00:58:57.410
ひゃだ!漢字まちがっちゃった☆
確かにこれじゃ「かつあい」だわ。
ていうかなんで「わりあい」の変換候補で「割愛」も出てくるのよ!(選んだアタシが悪いけど)

>>269
前半については、p ≠ q ならばその通りだと思うわ。
(例えば p = q = 2, m = 4 なら式が成り立つけど独立じゃないわ。)

後半の
>p,qが素数でもなくてよいなら
>[m/p][m/q]=m[m/lcm(p,q)]
は単なる予想で書いたの?
これが正しいかどうかは分からないけど、とりあえず書いておくと、
pが合成数の場合、Yがpで割り切れない確率が s^n みたいな単純な形にならないから上の議論は通じないの。
例えば m = 6 として
Yが4で割り切れない確率 = {(2/3)n+1}(1/2)^n
Yが6で割り切れない確率 = (1/2)^n + (2/3)^n - (1/3)^n
になるわ。

>>270
とりあえず、mが異なる素因数pとqを持つときは m = kpq と表せるから
[m/p] [m/q] = kq ⋅ kp = kpq ⋅ k = m [m/(pq)]
となって成り立つわよ。(m = p^j の形のときはわからないけど。)
もしかして、[m/p][m/q]=m[m/(pq)] が成り立つのはこの場合に限るという可能性もあるかもね。

ところでこの京大の問題、1992年の第4問の丸パクリなのね。
http://server-test.net/math/php_q.php?name=kyoto&v1=1&v2=1992&v3=1&y1=1992&n1=1&y2=1992&n2=2&y3=1992&n3=3&y4=1992&n4=4&y5=1992&n5=5&y6=1992&n6=6&y7=0000&n7=0
過去問やってた人にはボーナス問題ね。
0272陽気な名無しさん2023/08/16(水) 09:50:59.970
あ、ごめんなさい
nの回数で様子が変わるのはちょっとアタシの脳に負荷がかかりすぎるので
>[m/p][m/q]=m[m/lcm(p,q)]
も勝手にn=1だけの話にしちゃってたわスマンコ

なるほどね、素因数が2つあればそうなるわね

1992年に受験生だった人は今だいたい50歳
出題者が自分の頃の問題使ってるとか?
0273陽気な名無しさん2023/08/16(水) 09:56:43.830
しかしアタシの感覚だと30年前の過去問はやらないというか
見つけるのも難しいくらいなんだけど
東大や京大は25ヶ年とか出てるからみっちり過去問演習するのかしら
0274Usagi2023/08/17(木) 22:08:07.070
この問題、興味深いからもうちょっと考えてみたんだけど、たぶん次のことが成り立つと思うの。

1からmまでの数が等しい確率で出るサイコロをn回降って出るn個の目の積をYとする。
pとqは互いに素なmの約数で、いずれも素数の2乗を約数に持たない数とする。
このとき、pがYを割り切る事象とqがYを割り切る事象は独立である。

誰か証明してごらんなさいよ!
0275陽気な名無しさん2023/08/18(金) 10:19:04.170
いつも穏やかなうさぎが、
こんな挑発的な発言するの
珍しいわ!
0276陽気な名無しさん2023/08/19(土) 13:27:42.290
意外と難しいし、なぜか品格を感じるわ
アタシには解けないかも…
0277Usagi2023/08/19(土) 15:52:23.090
アタシも証明を書いたわけじゃないんだけど、中国の剰余定理を使えば言えるんじゃないかなって気がするの
でも証明をうまく書くのなかなか難しそうだから
いつもアタシをいぢわるな問題で苦しめてくる姐さん達におまかせして、鮮やかに証明してもらおうと思ったの☆
ま、もし間違いだったら反証してちょうだいw
0278Usagi2023/08/21(月) 21:57:31.330
>>274では、素数の2乗を約数に持たないという条件を書いたけど
よく考えたらこの条件要らないと思うわ。だからこうよ↓

mを自然数とし、pとqを互いに素なmの約数とする。
1からmまでのすべての自然数が等しい確率で出るサイコロをn回振って出るn個の目の積について、
これをpが割り切る事象とqが割り切る事象は独立である。

そしてたぶんだけどね、独立になるのはこの場合に限る気がするの。
つまり、pとqが互いに素なmの約数であることが独立になるための必要十分条件だと予想するわ。
0279Usagi2023/08/22(火) 21:52:51.670
このまま誰も解かないかもしれないし、自分で証明書いてみたわw
でも結構長くて記号や式が込み入っているから、ここに書き込むのすごく大変になるし
できたとしても、読みにくくて理解困難になると思うのよ。
だから、ちょっと恥ずかしいけどPDFにしてアップロードしたわ
これなら、もうしばらく自分で考えたい人はそのまま続けてもらえばいいしね☆
ちゃんと証明できているといいんだけど…。
https://9.gigafile.nu/1130-b5d2994370b66b6861132080b7b0d3b5e
0280陽気な名無しさん2023/08/23(水) 06:26:49.200
大作ね…
理解出来るかわかんないけど、勉強させていただくわ
0281Usagi2023/08/28(月) 22:39:17.520
ちょっとだけ修正しといたわ。
r_iが0以上p未満ってことを書き加えたのと、2頁目の式変形の下から2行目を書き加えただけだけど。
https://86.gigafile.nu/1206-db821888cb105c2b823d6343814085b9c

ていうかアタシかなり頑張って書いたのに反応薄くてちょっと寂しいわ
関係ないけどアタシ今日誕生日だったの…
0282陽気な名無しさん2023/08/28(月) 23:35:57.910
あらおめでとう
何歳になったの?
0283Usagi2023/08/29(火) 01:01:31.660
ありがとう。
先入観とか持たれるの嫌だしミステリアスでいたかったけど
アタシもうさんざん書き込んでていまさら先入観もクソもないだろうからいうと
年は、アタシが大学受験したとき加法定理を証明せよって問題が出て
何これ馬鹿なの死ぬのって思ったって言えばわかるかしら。
だってアタシ加法定理なんてもともとうろ覚えで、自信なくなったらいつも自分で証明して使ってたからね。
ちなみに他の問題は全然できなかったわw

誰か証明にコメントちょうだいよ。
なるほど、間違ってる、わからない、当たり前のこと書いていい気になるなブス!
なんでもいいからw
0284陽気な名無しさん2023/08/29(火) 08:35:59.340
あなた東大受験したのね。
それだけでレベルの高さがわかるわ。
証明も素人が余暇で気軽にコメントするには
レベルが高いわよ。
よっぽど暇でじっくり読み込める人で、
かつ結構なレベルの人でないと
この証明にコメントするのは難しいわ。

とりあえずこれはこれで反応する人を待ちつつ、
一方で普通の大学受験レベルの問題でもしないと
このスレ進まないのではないかしら。
数学オリンピックレベルの問題も、
高校の範囲内とはいえ、
スレが進みにくい高難度になるのではないかしら。

例えば加法定理を証明せよ、ってこのスレでやったら、
何通りかの証明が出てくるかもしれないわよ。
0285Usagi2023/08/29(火) 19:23:16.980NIKU
証明一見長いけど、言ってることはシンプルなの。
補題では、各A_iにmod pによる同値関係を与えることができるけど
その直積はA_1×…×A_n上の同値関係になるから、それでA_1×…×A_nを割った商集合をSとしたの。
B_iの方にも同じことをしてできる商集合をTとすると
SとTの間に、各要素のサイズの全体に対する割合とpで割った時の余りを保存する同型対応があると言ってるだけ。
本題のサイコロの話は、この補題と中国の剰余定理からの帰結よ。
同値類はもとになる集合を過不足なく分割するから、サイズを計算するときは単純な足し算になるのがポイントよ。
(高校生に理解できないかもだけど、出てきた話題を自分なりに考察してまとめたらこうなっただけだから勘弁してね)

「加法定理を証明せよ」をググってみたら、余弦定理を使ったややこしい証明ばっか出てきてびっくりしたわ。
そのやり方をする場合、余弦定理は証明しなくてもいいのかしらね?
0287Usagi2023/08/29(火) 21:51:21.410NIKU
>>286
この問題、友達が見せてきたことがあった気がするけど
こんなの解けるわけないよねwって流してた気がするわ
アタシは基本的に過去問ほとんどやらなかったの
だってどうせ同じ問題出ないだろうし
難しい問題は解き方思いつかなかったらしょうがないわよね〜て思ってたから

もし今受験勉強するなら違うでしょうけど
今はインターネットや参考書の情報が豊富で、問題のパターンや攻略法が整理されてるからね
0288陽気な名無しさん2023/08/29(火) 22:23:43.090NIKU
加法定理の証明ってアタシ高校では二点間の距離の公式使って証明習ったわ。
あと、オイラーの公式使った証明が出てくるかと思ったけど、
オイラーの公式の証明にマクローリン展開が必要なのね。
大学入試では使えないかしら。
それに複素数乗が指数定理成り立つことの証明って、
もしかして加法定理使ってたかしら?
0289Usagi2023/08/30(水) 01:40:11.890
あたしは基本的に↓みたいな方法で証明してたと思うわ
https://methodology.site/trigonometric-addition-formulas/
このベクトル使った方法だとややこしい計算不要で図形的に意味もわかりやすいの
cosとsinの式が同時に出るしね☆

オイラーの公式とかマクローリン展開は大学入試では基本ダメでしょ
もっともこの東大の問題の場合、まずsinとcosを定義せよって言って、
それからその定義にもとづいて加法定理を証明せよって言ってるの。
だからsinとcosをマクローリン級数で定義して、それを使って加法定理を証明するなら筋は通るけど
でも複素関数とか級数の収束を議論しなきゃいけなくなっちゃうわね

指数定理っていうか指数法則よね?
気になったから本で調べたら、加法定理は必要ないわ。
e^xのマクローリン級数を使うと
e^(x+y) = 1 + (x+y)/1! + (x+y)^2/2! + (x+y)^3/3! + …
だけど、この級数はxとyがどんな複素数でも絶対収束するから項の順番を変えてもよいから、yのべきで整理して
e^(x+y) = a_0(x) + a_1(x) y + a_2(x) y^2 + a_3(x) y^3 + …
となるとするわ。ここでa_i(x)はxのべき級数になっているわ。
xを固定して両辺をyについてn階微分してy = 0 とおくと
e^x = n! a_n(x)
となるから、a_n(x) = e^x/n! ね。したがって
e^(x+y) = e^x (1 + y/1! + y^2/2! + y^3/3! + … ) = e^x e^y
となるの!
0290Usagi2023/08/30(水) 07:00:38.540
「この級数は」は正しくは「これを展開した級数は」
0291陽気な名無しさん2023/08/30(水) 08:11:42.120
そうね、指数定理ではなく指数法則だったわ。
オイラーの公式は大学受験を考えなければ、
図形的な証明とは全く異なる証明として大丈夫そうね。
アタシの高校の授業では、α>βとして座標平面上に
A(cosα, sinα)、B(cosβ, sinβ)、C(cos(α-β), sin(α-β))
とするとE(1, 0)に対して三角形OABと三角形OCEが合同だから、
AB=CEが成り立つということで、
この両辺をそれぞれ二点間の距離の公式に当てはめて
等式で結んで、まずcos(α-β)の公式を証明したわ。
あとはαにπ/2-α代入したりして芋づる式に導いたわ。
普通高校の授業ってそうやってるものだと思ったけど、
違うのかしら?

オイラーの公式の証明法は基本的に大学入ってからよね。
でもドモアブル知っていれば二倍角や三倍角の公式
覚えてなくてもすぐ導けるから便利だったわ。
0292Usagi2023/08/31(木) 00:50:32.850
確かにそのやり方でもできるわね
でも2乗を計算したり後でαに別のものを代入したりで面倒だし
なによりも、結果として出る式が計算してみたらこうなったってだけで意味がわからないから
あまり良いやり方に思えないんだけど、その方法が一般的なのかしら?
アタシは高校でどう習ったとか教科書にどう書いてあったかとか覚えてないの

ベクトルのやり方は、(cos(α+β), sin(α+β)) が
e_1 = (cosα, sinα) と e_2 = (-sinα, cosα) を基底とする座標系では
(cosβ, sinβ) で表されるっていうことを用いているから、意味がよく分かるの
e_1とe_2の縦ベクトルを横に並べてできる R = (e_1 e_2) は角度αの回転を表す行列になって
(cos(α+β), sin(α+β)) = R (cosβ, sinβ) となるのよね
0293陽気な名無しさん2023/08/31(木) 06:35:52.140
一般的だとおも
0294陽気な名無しさん2023/08/31(木) 06:36:49.470
あら、途中で書き込んじゃった。
やっぱり下書きしてからじゃないとダメね。
0295陽気な名無しさん2023/08/31(木) 06:48:35.750
今度はちゃんと下書きしたわ。
改めて。

>>291 に書いた方法は、なんといっても
あの偉大な小平邦彦先生が監修している東京書籍の教科書に
載っていた方法だから、少なくとも当時は一般的だったと思うわ。
(当時っていっても、高2の数学が
代数・幾何と基礎解析に分かれていた時代だけどね)

それでベクトルのやり方って、鮮やかで簡潔ね。
驚いちゃったわ。
でも一次変換知っていないと駄目ね。
最近の高校生は一次変換やってないんでしょ。
あたしゃ代数・幾何で一次変換やったけどね。
一次変換が高校の範囲から外れたのは本当に残念よね。
0296陽気な名無しさん2023/09/01(金) 07:32:49.380
たしかに中国剰余定理を使うと当たり前のことだと思えるようになってきたわ
0297Usagi2023/09/03(日) 04:07:32.780
>>295
そうね、まあ、一次変換は表に出さなくても>>289のリンク先にみたいに書けばいいのよね
実際、説明なしで回転行列を使ったら証明としてはダメだと思うわ
あたしの時は、行列の計算方法は教えるのに、それが一次変換であるという数学的意味は教えないというクソみたいな課程だったのw
今度の新課程では行列が「数学的な表現の工夫」という項目で紹介されるけど、入試では出されない感じらしいわ
ちなみに高校で一次変換があった頃って、どのくらいのことをやってたのかしら。
3×3とか、2×2以外の行列もやったの? 固有値、階数、核とかもやったの?
0298Usagi2023/09/03(日) 04:32:16.530
>>296
そうよね?
1からmまでの中に、pで割った余りとqで割った余りのすべての組み合わせの数が同じ割合で存在しているから
サイコロを一回振って出る目について、それをpで割ってr余る確率とqで割ってs余る確率は、どのrとsについても独立よね。
だから複数回振って目の積をとっても同じことになるわよね。

逆に、pとqが互いに素なmの約数でない場合、どこかで割合が均等でなくなりそうだから
一般的には、独立でなくなると予想できると思うの。
それを証明するのはパスだけどw
0299Usagi2023/09/03(日) 04:35:43.260
上の質問は3×3とか3×2とか2×3とかも出てきたのって意味ね
0300陽気な名無しさん2023/09/03(日) 07:19:25.130
>>297
行列や一次変換やったといっても、
2×2だけよ。それより大きいのは紹介だけかな。
行列の足し算掛け算と、単位行列逆行列
くらいまでやったかしらねえ。
固有値はやってなかったと思うわ。
一次変換は座標軸が変わるイメージが
面白かったけど、大部分の生徒は、
イメージなんてなくてただ教えられた計算を
訳もわからずやっていたわ。
イメージがないと計算だけしても
面白くないと思うんだけどね。
0301陽気な名無しさん2023/09/03(日) 16:25:53.680
m,nを自然数、p,qを互いに素なmの約数とし、
M=Σ[x[1]=1~m]Σ[x[2]=1~m]…Σ[x[n]=1~m]cos(2πx[1]x[2]…x[n]/m)
P=Σ[x[1]=1~m]Σ[x[2]=1~m]…Σ[x[n]=1~m]cos(2πx[1]x[2]…x[n]/p)
Q=Σ[x[1]=1~m]Σ[x[2]=1~m]…Σ[x[n]=1~m]cos(2πx[1]x[2]…x[n]/q)
とする。
PQ/Mの値を求めよ。
0302陽気な名無しさん2023/09/03(日) 17:09:29.670
ちょっと、うさぎ!!
小平邦彦先生のウィキペディア読んでたら小平先生が「初等幾何学の重要性を主張していた。」って書いてあったわよ!
0303Usagi2023/09/04(月) 04:49:28.810
>>300
イメージできないことを訳もわからず計算させられるのは辛いわよね
大部分の生徒にとってそうだったなら、高校の課程から無くなった方が良かったとも考えられるわね
でも数学って最初イメージできないことも具体例を計算していくうちにイメージができてくるってこともあるから
一概にそういうのが悪いとも言えないところもあるわよね
あたしも群の商群なんか、具体例を自分でいろいろ作ってみてイメージができてきたから。
思ったけど、昔と違って今はPCでいろいろな関数のグラフとか一次変換の様子とかをグラフィックで簡単に見せられるんだから
そういうのを中高で活用すれば生徒がイメージしやすくなって良さそうじゃないかしら
0304Usagi2023/09/04(月) 04:57:42.670
>>301
あなたさ、これもう完全に大学入試とかのレベルじゃないでしょw いくらなんでもこれはどうなの?
そもそも問題ちゃんと成立しているのかしら。
例えば n = 1 で m ≥ 2 なら、M = Σ[x=1~m] cos(2πx/m) = 0 となるから、PQ/Mは定義されなくない?
0307Usagi2023/09/05(火) 00:07:21.230
>>302
そういえば、しばらく前に小平先生の「幾何への誘い」っていう文庫本を買ったのに
最初の方だけ読んで放置していたの思い出したから、発掘してきたわw

小平先生が初等・中等教育についても熱心だったなんて知らなかったわ。
ウィキペディアのおかげでいろいろわかったからちょっとメモ。
1957年にソ連が世界初の人工衛星打ち上げに成功して、冷戦中の米国は対抗意識で焦って
科学力増大のために高度な数学を初等・中等教育に持ち込むNew Mathという教育を始めたそうなの。
日本もその影響で、70年代の課程が高度な内容も取り入れて詰め込み教育になったらしいわ。
小平先生はこの動きに反対だったって、↓の論文の8頁目から書いてあるわ。
https://mylibrary.toho-u.ac.jp/webopac/bdyview.do?bodyid=TD35120616&elmid=Body&fname=35120616_cover.pdf
当時は小学生に集合論を、中学生にジョルダンの閉曲線定理を教えようとしていたって。
そんな定理アタシも知らなかったし、専門家にしか理解できないものをどう教えるのかしらw
でも小平先生の言葉が引用されているんだけど、僭越ながらアタシ完全には同意できないの。
歴史を辿らずに、後でできた考え方を先に学んだ方が良いこともあると思うのよね。
それに無限集合を扱わないなら集合論を教えても意味ない、は違うと思うわ。
>>265みたいに場合の数を考える時も集合で考えないと頭を整理できないし。
有限群を考える時は無限集合なんて出てこないけど、商群は集合という概念がなければ定義できないわ。

結局、70年代の課程が無理すぎたので、反動で80年代以降「ゆとり」になっていったらしいわ。
でも経緯を考えると別に「ゆとり」になったのではなく、単にふつうに戻っただけとも言えそうね。
0308陽気な名無しさん2023/09/05(火) 00:22:45.020
一時期の詰め込み教育はアタシも反対だわ。
中学校くらいで位相やったって聞いたけど、
位相なんて数学科出たアタシでも、
未だによくわかってないわ。
0309Usagi2023/09/05(火) 00:23:03.230
一次変換も、70年代に導入されたけど結局は無理もしくは意味ないと判断されて消えたってとこじゃないかしら
0310陽気な名無しさん2023/09/05(火) 00:25:55.290
一次変換は80年代後半に高校二年生でやったわ。
イヤだわ、トシがバレるわ〜
0311陽気な名無しさん2023/09/05(火) 00:28:12.670
というか、一次変換が無くなったときに、
行列自体全部無くなったんだっけ?
一次変換やらないなら行列やっても意味ないわよね?
0312Usagi2023/09/05(火) 00:54:49.910
入試が1997〜2005年の人は行列は習うのに一次変換は習わない意味不明な課程で、アタシはこれだったの。
たしか行列は、これ使って連立一次方程式解けるよね、みたいなしょうもない紹介のされ方だった気がするわ。
あと計算すると確かに成り立つよねって、ケイリー・ハミルトンの定理が教科書にあったと思うけど
まったく意味がわからなかったわねw
0313陽気な名無しさん2023/09/05(火) 07:32:39.040
ケイリーハミルトンって、固有値に関する固有多項式の話よね。
固有値って固有ベクトルとセットで一次変換がないと意味わからないのは当然よね。
固有値固有ベクトル一次変換なしでケイリーハミルトンやるって、
本当に意味が分からないわ。
0314Usagi2023/09/05(火) 18:47:25.60F
>>313
そうよね。しかも固有多項式 = 0 が重根を持たないなら固有ベクトルたちが一時独立になるから
ケイリー・ハミルトンの定理が成り立つことは簡単に理解できるけど
そうじゃない場合についてはアタシもよく知らないし、かなり高度な話よねw
0316陽気な名無しさん2023/09/07(木) 21:27:46.950
ねえ、さっきヨッシーの掲示板見たら回答が付いてたんだけど
読んでたらアタシひっどい書き間違えしてたのに気付いたわ

>M=Σ[x[1]=1~m]Σ[x[2]=1~m]…Σ[x[n]=1~m]cos(2πx[1]x[2]…x[n]/m)

これ、正しくは
M=Σ[x[1]=1~m]Σ[x[2]=1~m]…Σ[x[n]=1~m]cos(2πx[1]x[2]…x[n]/(pq))
だったわ!ごめんね!!
0317陽気な名無しさん2023/09/07(木) 23:16:33.880
あんたそっちの掲示板にずっといな。
0318Usagi2023/09/10(日) 23:56:55.350
まあそう言わずに、数学好きのゲイっていうマイノリティ中のマイノリティ同士なんだから、仲良くしていきましょう。

>>316
掲示板見たわ。この黄桃さんって方すごいわね。なるほど、
a = (x[1]…x[n-1]がpで割り切れる場合の数)
b = (x[1]…x[n-1]がqで割り切れる場合の数)
c = (x[1]…x[n-1]がpqで割り切れる場合の数)
とおくと、P = ma, Q = mb, M = mc となるのね。
ここで上で考察した確率の独立性から
c/m^{n-1} = a/m^{n-1} × b/m^{n-1}
だから、ab/c = m^{n-1}
したがって PQ = (ma)(mb)/(mc) = mab/c = m^n なのね?
よくこんなの思いついたわね。美しいと思うわ!
0319Usagi2023/09/11(月) 01:44:24.560
あ、PQじゃなくてPQ/Mね
0322Usagi2023/09/22(金) 00:48:53.420
>>321
ほとんどの人に理解できない書き方するのやめなさいよ
ふつうに言葉で書けば通じるのに。相手にしてもらえない可能性が高くなるだけよ?
それに、どういう意図の書き込みなのかもわからないわ。
あなたは答えを知っていて、それを私たちに問題として出題しているのか
それとも京大の問題を見て思いついた疑問を書いただけで、あなたもまだ答えを知らないのか
どっちなの?

ついでに言えば、|A|=∞ は記号の正しい使い方じゃないわ
そんなに記号を使いたいなら |A|=ℵ_0 と書きなさい!
0323陽気な名無しさん2023/09/22(金) 01:37:38.470
アタシも >>321 には物言いをつけたかったのよね。
でも何て言っていいか迷ってたから、うさぎが言ってくれて良かったわ。
g|fなんて書き方、高校までの範囲では出てこないわよね。
素朴な疑問だけど、アレフゼロなんて記号、スマホでも出せるのかしら?
どのみち加算無限とか連続無限なんて概念自体高校までの範囲を越えてると思うけど。

元ネタが大学入試問題だったとしても、それに手を加えて大学入試問題レベルを越えたら、
それはもうこのスレの対象範囲を越えてると思うわ。
大学入試問題そのものを出すか、手を加えるにしても大学入試問題レベルを越えない範囲にして欲しいものだわ。

大学入試問題レベルでも、ここで出されるのはハイレベルなものが多いから、
たまには難関大学ではない中堅レベル大学の入試問題程度なんかもあっていいと思うわ。
簡単な問題もたまにはないと、本当にうさぎとか一部の人しか解けないわよ。
0324陽気な名無しさん2023/09/22(金) 12:53:19.000
もう10日近く誰も書き込まないから、このまま誰も書き込まないとスレが落ちてしまって、
うさぎにもうさぎの腰巾着の>>323にも会えなくなって淋しくなるから、
以前京大の問題を解いたときにふと気になったことを、質問と出題を兼ねて書き込んだだけなのに、
なぜこんなに詰問され、叩かれなければならないの?
全くの純粋な善意から書き込んだのよ?
>>257だって同じようなスタンスの書き込みなのに、何故アタシだけ論難されなければならないのかしら?
0325Usagi2023/09/23(土) 05:00:28.530
>>324
スレあげてくれてありがとうね。傷ついたならごめんなさいね。
でも人のことを腰巾着だとか言うのは良くないわよ。仲良くしてくれているだけなんだから。
別にあなたをハブろうとして書いたわけじゃないの。
>>257は誰が読んでも理解できるように書かれているじゃない?
それに、自分でもまだ答えを出していない疑問だというスタンスもわかるわ。
>>321は違うのよ。言葉が足りなすぎるの。ていうか言葉がないw
これじゃ数学科に行ったような人にしか理解できないでしょ。それって人口の何%かしら?
大学で数学科以外の理系専攻だった人も分からない人が大部分だと思うわよ。
あたしだって前スレで ℝ[x] みたいな記号を知ったから分かったけど、もともと知らなかったし。
前スレを見ていなくて今のスレから見る人だっているでしょう?
だから、ふつうの言葉で書いておけば理解して疑問に答えてくれるかもしれない人でも
説明なしの記号を使っているせいでそうしてくれない、という可能性があるのよ。
それって、あなたにとっても損でしかないと思うけど、どうかしら?
確かにこのスレによく書き込んでいるのは数人だろうけど
その人たちだけに通じればいいって考えは排他的すぎるわよ。
こんな書き方が当然になったら、新しく参加する人がいなくなってしまうわ。
それに、あたしだけかもしれないけど、言葉がないと人間と会話してる感じがしないから
あまり相手をする気にならないのよ。

そして、まだ自分でも答えを出していない疑問なのか、それとも答えを知っている問題を出題しているのか
ここのスタンスをはっきりさせるのも重要よ。
前者の場合なら、まとまりきっていなくても思ったことを>>266みたいに気軽に書き込めるわ。
けれど後者の場合なら、見た目からして難しそうだし、あたしとしては
これまでの流れからして、超絶難問でテストされているように感じてしまうのよ。
そう思うと居心地よくなし、間違ったり解けなかったりすると意地悪コメント来るかもしれないと
身構えちゃって、完璧に解かないだめと思って気軽に書き込めないのよ。

第一あなたの問題、京大の問題を一般化してるのはわかるけど、一気に二段階一般化してて難しすぎるわ。
つーか、元の京大の問題がすでに難しいでしょw ネットで調べると最難レベルという評判よ。
そこから二段階難しくするってw
とりあえず一段階だけ一般化した問題については考えてみたから、書いておくわ。
0326Usagi2023/09/23(土) 05:06:02.400
問題
f(x)とg(x)は実数係数のxの整式で、すべての正の整数nに対してf(n)/g(n)は整数である。
このとき、f(x)はg(x)で割り切れることを示せ。

解答
g(x) = 定数の場合はf(x)はg(x)で割り切れるから、g(x)の次数が1以上の場合を考えるわね。
f(x)をg(x)で割った商をq(x)、余りをr(x)とすると f(x) = q(x)g(x) + r(x) となるわ。
q(x)の次数に関する帰納法で r(x) = 0 を示すわ。
ただし便宜上、q(x) = 0 の場合はq(x)の次数を -1 と考えることにするわね。

q(x)の次数が -1 の場合は、f(x)/g(x) = r(x)/g(x) となるわ。
r(x)の次数をmとすると、これはg(x)の次数より小さいから
r(x)/g(x)の分子と分母をそれぞれx^mで割ったものを考えると
n → ∞のとき、分子 → 定数、分母 → ∞ だから、r(x)/g(x) → 0 となるわ。
したがって、もしr(x) ≠ 0 なら十分大きい整数Nに対して 0 < |r(N)/g(N)| < 1 となって
f(N)/g(N) = r(N)/g(N) が整数であることに矛盾するわ。したがって r(x) = 0 よ。

では命題が次数kの場合まで成り立つするわ。
f(x)とg(x)の次数をそれぞれ s, t とすると q(x) の次数は s-t だから
これが k+1 となる場合、つまり s-t = k+1 の場合を考えるわ。
F(x) = f(x+1)g(x) - f(x)g(x+1)
G(x) = g(x)g(x+1)
とおくわ。f(x) = ax^s + …, g(x) = bx^t + … の形だから、
F(x) = {a(x+1)^s + … }(bx^t + … ) - (ax^s + …){b(x+1)^t + … }
は x^{s+t} の項が消えて、次数は s+t-1 以下になるわ。G(x)の次数は 2t だから
F(x)をG(x)で割った商の次数 ≤ (s+t-1) - 2t = s-t-1 = k
となるわね。そして、すべての正の整数nに対してf(n)/g(n)が整数だから
F(x)/G(x) = {f(x+1)g(x) - f(x)g(x+1)} / {g(x)g(x+1)} = f(x+1)/g(x+1) - f(x)/g(x)
もすべての正の整数xに対して整数になるの。
したがって帰納法の仮定から、F(x)はG(x)で割り切れるわ。
F(x) = f(x+1)g(x) - f(x)g(x+1)
= {q(x+1)g(x+1) + r(x+1)} g(x) - {q(x)g(x) + r(x)} g(x+1)
= {q(x+1)-q(x)} g(x)g(x+1) + r(x+1)g(x) - r(x)g(x+1)
だから、これが G(x) = g(x)g(x+1) で割り切れるということは
r(x+1)g(x) - r(x)g(x+1) が g(x)g(x+1) で割り切れることになるけれど
r(x)の次数がg(x)の次数より小さいので
r(x+1)g(x) - r(x)g(x+1) の次数は g(x)g(x+1) の次数より小さくなるから
r(x+1)g(x) - r(x)g(x+1) = 0
が結論されるわ。仮定から、正の整数nに対してg(n) ≠ 0 だから
r(2) = {g(2)/g(1)} r(1)
r(3) = {g(3)/g(2)} r(2) = {g(3)/g(2)} {g(2)/g(1)} r(1) = {g(3)/g(1)} r(1)
などとなって、結局すべての正の整数nについて
r(n) = {g(n)/g(1)} r(1)
となるわ。もし r(1) ≠ 0 なら、xのt次方程式 {g(x)/g(1)} r(1) - r(x) = 0 が
t個より多くの解を持つことになって矛盾するから、r(1) = 0よ。
するとr(x)はすべての正の整数nに対して r(n) = 0 となるけれど
r(x)はt次未満の整式だから、これが成り立つのは r(x) = 0 の時に限るわ。(終)

>>321の問題だと、もしAが無限に続く等差数列を含むなら同様に解けると思うけれど
そうでない場合はちょっと分からないわ。
ちなみにこれを書いたのは、あなたがふと疑問に思ったことだって書いたからよ。
そう書いてくれなければ、もとの問題は解けていないからダメねと思って書き込まなかったわ。
0327Usagi2023/09/23(土) 05:11:20.520
>>323
あたしPCだけどℵをどうやって出すのか分からなかったから
Wikipediaのaleph numberのページに行って、そこからコピペしたわ
ちなみに加算無限じゃなくて可算無限よね
0328陽気な名無しさん2023/09/23(土) 06:49:17.840
>>327
打ち間違えたわ<可算無限
0329陽気な名無しさん2023/09/23(土) 08:48:39.360
なるほどねえ
上手く解けるものね
これなら全然大学入試にも使えるじゃない
0330陽気な名無しさん2023/09/23(土) 22:06:00.220
ていうか、これ前に似たような問題やらなかったっけ
その時もうさぎが日本語書けええええええってブチ切れてたような
同じ出題者かしら?
うさぎがキレ散らかしながらもどこかから見つけてきた解き方を覚えてると
そんなに難しくないような気がするわ
0331Usagi2023/09/24(日) 04:59:30.200
>>321について、ちょっと思ったことを厳密じゃないけど書いてみるわ。
f(x) = a_s x^s + … + a_1 x + a_0
g(x) = b_t x^t + … + b_1 x + b_0
として、s+t+2 個の整数 n_1, …, n_{s+t+2} に対して f(n_i)/g(n_i) = m_i が整数だとするわ。
すると s+t+2 個の式 f(n_i) - m_i g(n_i) = 0 ができるけど
これは s+t+2 個の未知数 a_s, …, a_0, b_t, …, b_0 についての連立方程式になるわよね。
すべて = 0 だから値は決まらないけれど、うまくいけば a_j/b_t や b_k/b_t の比率がすべて決まって
その場合、連立方程式の係数がすべて整数だから、これらの比率は有理数になるんじゃないかしら。
これが解けるかどうかについては線型代数の話になってくると思うんだけど
あたしはこれについてはちょっとわからないから、誰か教えてくれたら嬉しいわ。

でももしこれができるとすれば、F(x) = f(x)/b_t と G(x) = g(x)/b_t の係数はすべて有理数になって
f(x)/g(x) = F(x)/G(x) = Q(x) + R(x)/G(x)
に現れる商Q(x)と余りR(x)の係数もすべて有理数になるわ。
ここで R(x) ≠ 0 と仮定するわ。
Q(x)に現れる係数をすべて規約分数で表した時、その分母の最小公倍数をdとするわ。
|x| → ∞ のとき R(x)/G(x) → 0 だから、ある正の整数Nがあって |x| ≥ N なら 0 < |R(x)/G(x)| < 1/d となるわ。
すると |n| ≥ N である任意の整数nについて、
Q(n) が整数なら f(n)/g(n) = Q(n) + R(n)/G(n) は整数ではないし、
Q(n) が整数でない場合は、Q(n) は一番近い整数から 1/d 以上離れているから
f(n)/g(n) = Q(n) + R(n)/G(n) はやはり整数じゃないわ。
すると f(n)/g(n) が整数となるnは2N個未満しかないことになって、無限にあるという仮定に反するわ。
だからR(x) = 0、つまりf(x)がg(x)で割り切れることになると思うの。
0332陽気な名無しさん2023/09/24(日) 09:37:19.190
素晴らしいわ
正解だと思うわ

> Q(n) が整数でない場合は、Q(n) は一番近い整数から 1/d 以上離れているから

これ、f(ℝ-ℚ)⊂ℝ-ℚとなるf∈ℝ[X]の時に出てきたわよね?
03333212023/09/24(日) 13:33:17.030
やっぱり凄いわね、うさぎ
口の達者さもさることながら数学の実力も一流ね
この問題(質問)は過去にいくつかの掲示板で出してきたけど解いたのはうさぎがはじめてよ🐇🐰
0334Usagi2023/09/24(日) 22:20:12.390
やだ、あたしも既約分数を規約分数とか書いてたわw

>>332
前スレ見てみたけど、こういう議論は出てなかったと思うわ。
前スレの問題を解くのにも使えるのか知らないけど。

大筋では解けたかもしれないけど、>>331の前半部分がいいかげんだから
アタシの中では完全にスッキリとはしてないのよね
a_s, …, a_0, b_t, …, b_0 を縦に並べたベクトルをvとすると
上の連立方程式は n_i や m_i を使って表される (s+t+2)次の正方行列Aを用いて
Av = O と表されるわよね(Oは零ベクトル)
b_t ≠ 0 だから v ≠ O ね。したがって、vはAの核空間に属するOでないベクトルになるわ。
核空間の次元が1なら v/b_t は一意的に決まるけど
次元が2以上ならそういかなくて、その場合、a_j/b_t や b_k/b_t が有理数である必要性がない気がするの。
そういうことが起こらないのかどうか、アタシちょっとわからないから
ここのところをだれか説明して補完していただけないかしら?
0335陽気な名無しさん2023/09/25(月) 01:20:35.970
変なところで悩むのね
Ax=Oの零ではない解で成分が全て有理数のものどれかひとつを
(a'[s],…,a'[0],b'[t],…,b'[0])
とし、
F(x)=a'[s]x^s+…+a'[0]
G(x)=b'[t]x^t+…+b'[0]
とするとf(n)G(n)-F(n)g(n)=0が無数のnで成り立つのでfG-Fgは多項式として0
Gは多項式として0ではないので任意のxで
f/g=F/G
が成り立つ
初めからF/Gだと思えばいい
0336Usagi2023/09/26(火) 21:35:37.42F
なるほど、ありがとう。

>Ax=Oの零ではない解で成分が全て有理数のものどれかひとつを
なぜそういう解が存在するかというと、連立方程式を解いていくと
s+t+2個の未知数のうちいくつかをパラメータとして
残りの未知数がそれらのパラメータの有理数係数の一次結合で表されるから
パラメータの未知数に0以外の有理数を入れればそういう解が得られるのね。

>f(n)G(n)-F(n)g(n)=0が無数のnで成り立つので
無数ではなくて、n_1, …, n_{s+t+2} のs+t+2個よね。
でもfG-Fgは高々s+t次だから、fG-Fg = 0 となるのはその通りね。
ってことは、s+t+2個じゃなくてs+t+1個の格子点だけ考えれば十分だったのね。

>Gは多項式として0ではないので任意のxでf/g=F/Gが成り立つ
ここは不正確で、gとGの零点以外の任意のxで、よね。でもそれらは高々有限個しかないから
F/Gが無数の整数に対して整数の値をとるという性質は保たれるのね。

で考えてみたけれど、Aの核空間の次元(dim ker A)って、実は t+1 なのね。(tはg(x)の次数)
(s+t+1個の格子点で議論を始めるならAは (s+t+1)×(s+t+2)行列だけど、どっちでも同じ)
問題の条件が満たされる場合 f/g = Q となる整式Q(x)があるわけだけど
任意のs次以下のF(x)とt次以下のG(x)に対して、それらの係数のつくるベクトルvが Av = O を満たせば
上の議論から F = (f/g)G = QG となってFの係数はGの係数が決まれば決まるから
vは実質Gのt+1個の係数 b_t, …, b_0 で決まるので、dim ker A ≤ t+1 ね。
逆に、任意のt次式G(x)に対して F = QG とおけば
F(n_i) - m_i G(n_i) = F(n_i) - Q(n_i) G(n_i) = 0
となるから、FとGの係数のつくるベクトルvは Av = O を満たすわ。
このことから、dim ker A ≥ t+1 がわかるわ。
したがって dim ker A = t+1 ね。
ついでに rank A = (s+t+2) - dim ker A = s+1 になるわ。
0337陽気な名無しさん2023/09/26(火) 23:56:17.650
なんか急に不安に駆られたわ
勝手に s+t+2 個の整数 n_1, …, n_{s+t+2} をとってきて s+t+2 個の式 f(n_i) - m_i g(n_i) = 0 ができるのはいいんだけど、
これをAv=OとかいたときrankA=s+1となる根拠ってどこにあるんだっけ
つまり、なんで勝手に s+t+2 個の整数 n_1, …, n_{s+t+2} をとってきた時点で f(n_i) - m_i g(n_i) = 0 のうちのs+1個が独立な方程式だと言い切れるの?
0338Usagi2023/09/27(水) 10:39:12.850
それは、線型写像 T: V → W に対して
rank T + dim ker T = dim V
であるという定理を使っただけなんだけど。

結局G(x)の係数 b_t, …, b_0 がパラメータになるのよ。
これらを未知数じゃなくて定数だと思うと
もとの連立方程式は、 a_s, …, a_0 のs+1個を未知数として持つ連立方程式とみなせるの。
これが解けるということは、s+1個独立な式があるということで
解くとa_s, …, a_0 が b_t, …, b_0 の有理数係数の一次結合で表されるわ。
逆にs+1個独立な式がないと、a_s, …, a_0 が完全に決まらないことになるわ。

でもs+t+1個以上のn_iについて
f(n_i) - m_i g(n_i) = 0
に加えて
F(n_i) - m_i G(n_i) = 0
が成り立てば
(第1式) × G(n_i) - (第2式) × g(n_i)
から
f(n_i)G(n_i) - F(n_i)g(n_i) = 0
で、これは高々s+t次だから整式として f(x)G(x) - F(x)g(x) = 0 となり
したがって F(x) = {f(x)/g(x)} G(x) = Q(x)G(x) となって
F(x)の係数a_s, …, a_0はG(x)の係数b_t, …, b_0の有理数係数の一次結合として完全に決まるわ
だからどれでもいいからs+t+1個の情報があれば十分ってことじゃない?
0339陽気な名無しさん2023/09/27(水) 11:38:55.270
s≧2とする。
s+t+2 個の整数 n_1, …, n_{s+t+2} を、
s+t+2 個の式 f(n_i) - m_i g(n_i) = 0 が、
この中のどれか2つだけが独立で、他の式は独立な2式の線型和となるようにとる。
(もしかしたらどう頑張ってもこのようには n_1, …, n_{s+t+2} をとれないかもしれないんだけど、もしとれないのだとしたらその理由は?)
このとき、s+t+2 個の式 f(n_i) - m_i g(n_i) = 0 を Av=O とかくと rankA=2 である。

しかし、ここから>>335-336の議論を続けると、あれよあれよというまに
rankA=s+1>2
となり、最初に rankA=2 にとったのはなんだったのかしら?
とならない?

本当は s+t+2 個の方程式 f(n_i) - m_i g(n_i) = 0 をどうとってもこの中に十分豊富に独立な式が入っていることを先に示さないといけないのに、独立な式がたくさんあると仮定して最後結局やっぱり独立な式がs+1個ありましたね、
ということになってない?
0340Usagi2023/09/28(木) 02:05:30.180
(1) f/g が無限個の整数に対して整数の値を取る
⬇︎
(2) f/g が s+t+1個の整数に対して整数の値を取る
⬇︎
(3) ある F, G ∈ ℚ[x] があって高々有限個の点を除くすべてのxに対して f(x)/g(x) = F(x)/G(x)
⬇︎
(4) GはFを割り切る
⬇︎
(5) f/g = F/G = Q となる Q ∈ ℚ[x] が存在する
⬇︎
(6) dim ker A = t+1

こういう流れよ。
(2)の時点では dim ker A ≥ 1 はわかっているけど dim ker A = t+1 であることは判明していないわ。
でも(2)から(3)を出すのにそのことは使ってないから問題ないわ。
もし独立な式の数が少ないなら、それは制約がより少なくて自由なんだからより簡単なだけ。
その場合パラメータの数が増えるけど、全部が0にならないように有理数を適当に設定しとけばいいわ。
(5)の時点で>>321の問題が解けるわけだけど
(6)はそれに付け加えとして、(5)で出たQ(x)を使って dim ker A = t+1 を導いたのよ。

だから実際問題として

>この中のどれか2つだけが独立で、他の式は独立な2式の線型和となるようにとる。
>(もしかしたらどう頑張ってもこのようには n_1, …, n_{s+t+2} をとれないかもしれないんだけどもしとれないのだとしたらその理由は?)

これはどう頑張ってもできないのよ。理由は(6)によるわ。
0341陽気な名無しさん2023/09/28(木) 12:43:43.620
あ、つまり、
P,Q,R∈ℝ[x], degP=s, degQ=t, P=QR,
∀x[k](k=1,2,...,s+t+2)∈ℝ, x[i]≠x[j](i≠j),
y[k] : =R(x[k])
としたときにs+t+2個の(P,Qの係数に関する)方程式
P(x[i])-y[k]Q(x[k])=0 (k=1,2,...,s+t+2)
のなかには必ずs+1個独立なものがある、
という定理が、(この問題にまったく無関係とまではいわなくても)やや脱線気味に、豆知識的に書いてある、
と思えばいいのかしら?
0342陽気な名無しさん2023/09/28(木) 13:02:53.080
アタシを1番不安にさせたのは>>336
> 無数ではなくて、n_1, …, n_{s+t+2} のs+t+2個よね。
このツッコミだわ

ここ無数でいいはずなのに、なんでいきなり細かいこと言い出すのよ、って思ったんだけど、
よく読むとうさぎの中では(2),(3)で
f/g∈ℝ(X)(有理関数体)がdegf+degg+2個の格子点を通る⇒f/g∈ℚ(X)
という元の問題より少し緩やかな仮定で上記命題を示したい、
という意志がわりと強めだったってことなのかしら

アタシの中では(2)が全く存在しなかったから急に行列とかs+t+2個の整数とか持ち出されて、もしかして精密に考えると何かすごいことが言えるのかしらとか混乱してしまったわ
そもそも>>335もAx=Oなんて考えてなくて、無数にある有理数係数の連立方程式のゼロでない有理数解が存在するのでそれを〜
くらいしか念頭になかったの
0344Usagi2023/09/29(金) 21:43:19.990NIKU
>>341
そういう感じよ。より正確にはs+t+1個以上の式があれば独立な式の数は常にs+1個となるわ。

>>342
>ここ無数でいいはずなのに、なんでいきなり細かいこと言い出すのよ、って思ったんだけど、
無数でいいはずないわ。
あくまでAv = Oの解を見ているのだから、Aを作る時に使ったs+t+2個に関する条件しか成立することは保証されていないわ。
もしs+t個以下の方程式で始めていたら、「無数の〜」は言えないし、(3)の結論は得られないでしょ。

>そもそも>>335もAx=Oなんて考えてなくて、無数にある有理数係数の連立方程式のゼロでない有理数解が存在するのでそれを~
なんでそういうものがあるってすぐわかったのかしら?
>>336の最初に書いたことを頭の中で考えたの?
あたし考え直してみて今はっきり認識したんだけど、これを言うためには
ker A ≠ {O}(したがって dim ker A ≥ 1)であることをはっきりと言う必要があったわね。
このことは問題のfとgの係数で作るベクトルがker Aに属することからわかるわ。

あなたって数学的直感が素晴らしいけど、無限の扱い方がちょっといいかげんな気がするの。
無限は数じゃないから、ふつうの数と同じように扱うことはできないわ。
(|A|=∞ とか書いちゃってることとも無関係でない気もするわ)
証明は有限の長さでなければならないから、無限にあるもの全てを扱うっていうのは基本的にできないのよ。
無限に式があっても、基本的にはそのうち何個かを取り出したものを解くことしかできないわ。
その個数はいくら大きくても、一億個でもいいけれどね。
そして連立一次方程式を解くというのは線型代数の話になるのだから、行列の話になるのは自然な流れでしょ。
無限に多くの式は、行列を使って同じように検討することもできないし。
「無限に式があるんだから、情報が不足したら考慮する式をどんどん増やしていけばうまくいきそうよね」
とかつい思っちゃうけれど、それだけでは証明にはならなくて、うまくいくことを検証する必要があるわ。
だって、情報を増やしていく時に「ハズレ」ばっか引いてしまったら永遠に証明が終わらないからね。
この問題の場合は、式がs+t個以下では足りないけれど、どれでもいいからs+t+1個以上あれば十分なのよね。
そこまではっきりさせてはじめて証明になるし、数学的意味もわかってくるじゃない?

あなたって恐らく前スレであたしが解けなかった「f(ℝ-ℚ)⊂ℝ-ℚならf∈ℚ[X]」の問題を出した姐さんよね?
(もし人違いだったらごめんなさいね)
あの時あなたは
「n次多項式f(x)が無数の有理数に対してfが有理数の値をとるなら、f(x) ∈ ℚ[x]である」
を帰納法で示してくれたけど、あたしがそれを見て
「n次多項式f(x)がn+1個の有理数に対して有理数の値をとるなら、f(x) ∈ ℚ[x]である」
に書き直してみたわよね。無限ってむやみやたらに使うものじゃないと思うの。
あたしは問題となっていることを示すのに必要最低限の仮定だけを使って示したいし
そうすることで数学的理解も深まると思うの。
0345Usagi2023/09/29(金) 21:47:49.160NIKU
関係ないけど、無限次元の行列といえば、ヒルベルト空間上の線型作用素も
はじめの頃は無限行列とか呼ばれてたらしいわ。今でもそう言うことあるのかしら?
あと「線型」と「線形」のどっちが正しいのか、いまだに分からなくて困るわw
0346陽気な名無しさん2023/09/29(金) 22:32:34.690NIKU
うーん、やっぱりぜんぜん納得できないわ
うさぎが最初にs+t+2個の整数を選んだのって、なにか選び方があるの?
それとも適当に絶対値の小さいほうからs+t+2個選ぶ、とかでいいのかしら

アタシには後者のように適当に選んでるようにしか読めないからそれで考えるけど、
かりにdim kerA=1であるようにs+t+2個を選んでしまった場合、有理数解を
(a'[s],…,a'[0],b'[t],…,b'[0])
とし、
F(x)=a'[s]x^s+…+a'[0]
G(x)=b'[t]x^t+…+b'[0]
とするとf(n)G(n)-F(n)g(n)=0がs+t+2個のnで成り立つ、って言えないのでは?
だって、dim kerA=1ということは見かけ上f(n_i) - m_i g(n_i) = 0がs+t+2個あるように見えても本質的にはs+t+1個はあるどれかの式の定数倍ということであって、
ある(n_1,m_1)を通るということしか情報はないんじゃないのかしら

>あたし考え直してみて今はっきり認識したんだけど、これを言うためには
>ker A ≠ {O}(したがって dim ker A ≥ 1)であることをはっきりと言う必要があったわね。

こんなことは当たり前中の当たり前のことであって、もっと別のdim kerAについての議論が必要な気がするわ


>この問題の場合は、式がs+t個以下では足りないけれど、どれでもいいからs+t+1個以上あれば十分なのよね。

アタシ、これ大間違いだと思う
0347陽気な名無しさん2023/09/29(金) 22:43:04.700NIKU
アタシ迂闊にもうさぎに釣られて>>335でAなんて書いたけど
ほんとうは余計なこと書かず
この無数の連立方程式を解き、(係数が有理数だから)有理数解が存在するので、それを〜〜
だけのほうがよかったんだわ
0348Usagi2023/09/30(土) 02:19:12.920
>>346
>うさぎが最初にs+t+2個の整数を選んだのって、なにか選び方があるの?
>それとも適当に絶対値の小さいほうからs+t+2個選ぶ、とかでいいのかしら
あたしも>>331書いた時には、そういうの気にしなきゃいけないのかと思ってわかってなかったの。
けれど今はわかったの。どれを選んでも良いの。そしてs+t+1個でいいの。

>だって、dim kerA=1ということは見かけ上f(n_i) - m_i g(n_i) = 0がs+t+2個あるように見えても本質的にはs+t+1個はあるどれかの式の定数倍ということであって、
>ある(n_1,m_1)を通るということしか情報はないんじゃないのかしら
これはいろいろ間違っているわ。
dim ker A = 1 ということは rank A = s+t+1 だから本質的に独立な式がs+t+1個あることになるわよ。
本質的に独立な式が1個だけになるのは dim ker A = s+t+1 の場合でしょ?
そしてここで独立な式だとかいうのは、あくまでこれらの式たちを
a’[s], …, a’[0], b’[t], …, b’[0] を変数とする方程式として見た時の話よ。

次の段階でf(n)G(n)-F(n)g(n)=0がs+t+1個のnで成り立つという話の時は
今度は a’[s], …, a’[0], b’[t], …, b’[0] の方を定数と見て
h(x) = f(x)G(x) - F(x)g(x) という、xを変数とする整式を考えているのよ。
h(x) = 0 が x = n_1, …, n_{s+t+1} というs+t+1個の根を持つことから
h(x) は因数定理により (x-n_1)…(x-n_{s+t+1}) で割り切れ
h(x)が高々s+t次であることから h(x) = 0 がわかるのよ。

>>347
>この無数の連立方程式を解き、(係数が有理数だから)有理数解が存在するので、それを~~
無数の〜ってそんなこと有限のステップでできるかどうかわからないし
きちんとした数学の証明とはいえないと思うわ
0349陽気な名無しさん2023/09/30(土) 08:56:58.140
>本質的に独立な式が1個だけになるのは dim ker A = s+t+1 の場合でしょ?
ごめん、書き間違えたわ

>そしてここで独立な式だとかいうのは、あくまでこれらの式たちを
>a’[s], …, a’[0], b’[t], …, b’[0] を変数とする方程式として見た時の話よ。
わかってる

>次の段階でf(n)G(n)-F(n)g(n)=0がs+t+1個のnで成り立つという話の時は
>今度は a’[s], …, a’[0], b’[t], …, b’[0] の方を定数と見て
>h(x) = f(x)G(x) - F(x)g(x) という、xを変数とする整式を考えているのよ。
これも当然わかってるんだけど…

やっぱりアタシうさぎの理屈では
なぜ無数の(s+t+2個の)xに対してF(x)/G(x)がf(x)/g(x)と同じ値をとるのか
がいまいち判然としないわ
s+t+2とかs+t+1とか出てきて具体的で現実的で精緻な議論のはずなのに、むしろ全然わかんない…

連立方程式を解くっていっても
f(n_i) - m_i g(n_i) = 0
という方程式は無数にあるが、独立なものは有限個(f,gの係数の個数未満)
これら独立な方程式全てをもってきて、それらの解が作るベクトル空間をVとする
Vには成分が全て有理数の元が存在する(というか、Vの基底は全てが有理数のものをとれる)
が、それはいまの議論にはまだ関係なくて、とにかく、
Vの元でゼロではなくかつf,gの係数でもないものをとりF,Gとする (*)
F(x)/G(x)は無数のxにたいしてf(x)/g(x)と同じ値をとる (*)
したがってfG-Fgは多項式として0 …(以下略)
ということなんだけど、
うさぎの議論でなんで(*)が不要なのかアタシにはわかんないのよね
F,Gの係数がVの元だということはかなり決定的なことだと思うんだけど
でも、うさぎの主張だとそもそも(*)が不要ってことなのよね?n_iを適当にとるということは
わからん…
まあ、もう少し考えてみるわ

というか考えてたら別の方向にいっちゃったんだけど
係数をハメル基底で表すのも悪くなさそうよね
0350陽気な名無しさん2023/09/30(土) 14:01:49.060
あ、ごめんわかったわ
なるほどね、勝手にs+t+2(s+t+1)個選べばいいのね
0351Usagi2023/09/30(土) 16:15:56.630
そうよ、どれでもいいからs+t+1個以上選べばいいのよ。
何がわからなかったのかよくわからなかったけど納得されたのかしら。

>これら独立な方程式全てをもってきて、それらの解が作るベクトル空間をVとする
そのVってあたしの ker A そのものよね?

>Vの元でゼロではなくかつf,gの係数でもないものをとりF,Gとする (*)
このF, Gはゼロでないものでなければいけないけれど、F = f, G = g でもいいわ。
その場合、f, gがもともと有理数係数だったってだけの話よ。

つーかこれ、もともとアタシがよくわからないって書いてた疑問を
あなたが>>335で鮮やかに解決してくれたと思ったら
なぜか今度はアタシがそれをあなたに説明することになってワケわかんなくて
アタシおかしいこと言ってるのかしら?って不安になってきてたわw
0352陽気な名無しさん2023/09/30(土) 17:30:38.070
この問題考えた時にまず>>349で解いたのよ
つまり、無数の方程式から独立なもの全てを選び、これが全部でr個だとする(r<s+t+2)
この連立方程式をr×s+t+2行列BでBx=Oと表す(dimkerB=s+t+2-r)
で、kerBの有理数解をひとつ選ぶ、と

でもうさぎの言うことをよく読むと、行列Bじゃなくて適当に選んだ行列Aで議論してるじゃない?
んもう、いくらなんでもアンタ適当に選んじゃダメよってずっと思ってたんだけど
今日ゆっくり考えたら大丈夫だったわ

議論をBで始めるのとAで始めるのとはうさぎには同じかもしれないけどアタシの中ではかなり違う最初の一歩だからうさぎのやろうとしてることとなかなか折り合いがつかなかったわ
無数でいいのになんでいちいちsとかtとか出すのよ!んもう!!ってしっかり読もうとしなかったり
自分の考えが抽象的でなかなか良いものだと自信があったのもあって行列もs+t+2ももうやめて!!ってなってたところもあるかもしれない

悪かったわね
0353陽気な名無しさん2023/10/15(日) 08:06:50.44ID:Tsb2pvOj0
移動して復活したのね。
皆さん戻ってくるかしら?
0354陽気な名無しさん2023/10/15(日) 23:42:36.46ID:38gfmdz+0
もしかしたら簡単かも

ℤ1[X]⊂ℤ[X]はモニックの集合、P∈ℤ1[X]とする
∀f∈ℤ[X], ∃F∈ℤ1[X] s.t. F⚪︎f/P∈ℤ[X] か?
0355陽気な名無しさん2023/10/16(月) 17:16:00.96ID:A6mSabhj0
F⚪︎fの意味が分からないわ。
F(f(x))のこと?それともf(F(x))のこと?
もしかしてF(x)f(x)のこと?
F(x)f(x)なら簡単よね。F=Pとすればいいんだから。
0356陽気な名無しさん2023/10/17(火) 20:11:28.34ID:Dqo2CTbA0
P(x)=x
f(x)=1
0357陽気な名無しさん2023/10/18(水) 11:00:47.98ID:A4dHkKYj0
Pは∀なの?∃なの?問題からして∀Pを一つ固定した時に、ってことよね?
何なら「∀f∈ℤ[X], ∃F∈ℤ1[X] s.t. F⚪︎f/P∈ℤ[X]となるPの条件は?」という問いにしてもいいのでは?
例えばP=定数とかにしたらFをPの任意の倍元にすればいいから必ず成り立つわけだし。
もし任意のPに対して成り立つなら>>354の答えはyesになるわけだし。
出題者はyesになることをわかっていて出題しているのかな。
0358Usagi2023/10/22(日) 02:55:55.13ID:qIszYqZg0
>>354
あなたって321の人? もしそうなら失望したわ
違う人だとしても、アタシもうこういうの相手にするの嫌になったわ
このスレで解いているのほぼアタシだから、アタシへの嫌がらせみたいにも感じてきたわ
モニックって最高次係数が1の多項式のことよね? そんな言葉、一般人は知らないの
そしてℤ1[X]っていうのは一般的な記号なのかしら? 誰か教えてほしいわ

まず言いたいのは、s.t. は such that だろうけど、こんなの一般的な日本人に通じるはずないわ
省略記号というのは、何度も登場するものに対して使うもので、しかも必ず最初に断ってから使うものよ
しかも英語で書いているならまだしも、日本語の文章中に英単語の略をいきなり使うなんてありえないわ
これ↓のk170さんの解答を読みなさいよ
https://math.stackexchange.com/questions/934108/what-does-s-t-mean
英語でも基本的には s. t. なんて使うべきではないって書かれているわ
自分用のノートなら好きなだけ書いていいわよ?
でも他人に理解してもらうための文章を書く時に、説明もなくこういう書き方をするのは独り善がりすぎるわ

そもそも、こういうインチキ論理式もどきみたいなのって、アタシ軽蔑するくらい嫌いなの
これ↓のhmakholm left over Monicaさんの解答を読みなさいよ
ちょうどアタシが言いたいことが丁寧に説明されているから
(321にある (∀a∈A)なんて、まさにこの方が批判している書き方ね)
https://math.stackexchange.com/questions/2933370/do-we-need-such-that-after-qualifiers
日本語の文章中でs.t.とか書くくらいだから、英語は読めるんでしょ?
これ読んだらこういう書き方はやめなきゃいけないとわかるはずよ
もし理解できないとしても、そういう書き方を続けていると
記号の使い方に対する無理解をさらすだけだからやめた方がいいわよ

そしてF⚪︎fだっていきなり使ってるから、実際に通じていないじゃないの
理解してもらえなくて損するのはあなただって、何度も言っているでしょ?
もちろん関数合成に⚪︎を使うのは一般的なことだけれど
何の説明もないから、アタシも何を表しているのか推測しなきゃわからなかったわ
>>356は F⚪︎f が f(F(x)) の意味ではないということを示す反例として書いたんだろうけど
なんで日本語で説明しないで、読者に推測させようとするのかしら?
ここは数学の問題を解くスレであって、日本語が不自由な人の書き込みの意味を推測するスレではないわ
ホント何様のつもりなの?

あなたみたいな書き方は間違っていると思うけれど、やるなら同サロじゃなくて数学板行きなさいよ
こんなとこよりも通じる人が多いでしょ?
0359Usagi2023/10/22(日) 03:15:16.37ID:qIszYqZg0
変なコメントが来たら嫌だから最後に一応問題を解いておくわ
でももう二度としないわよ

アタシが理解した問題を書き直してみるとこうよ↓
(こういうふうに書かれていたら文句言わなかったわ)

ℤ[X]で整数係数のXの多項式の集合を表し、ℤ1[X] = { f(X) ∈ ℤ[X] | fの最高次係数が1 } とする。
P(X) ∈ ℤ1[X] かつ f(X) ∈ ℤ[X]とする。
このとき、ある F(X) ∈ ℤ1[X] があって F(f(X))/P(X) ∈ ℤ[X] となることを示せ。

解答
P(X)が0次式の場合 P(X) = 1 だから、これは明らかに成立するわ。
だからP(X)がn次式として、n ≥ 1 の場合を考えるわ。
P(X)のn個の根をα_1, …, α_n とすると、 P(X) ∈ ℤ1[X] だから
根と係数の関係からα_1, …, α_n を変数とする基本対称式はすべて整数となるわ。
F(X) = (X - f(α_1)) … (X - f(α_n))
とおくと、これを展開した時の各 X^i の係数は f(α_1), …, f(α_n) を変数とする基本対称式かその -1倍になるけれど、
それぞれをα_1, …, α_n を変数とする式として展開した時の各項の係数は、f(X) ∈ ℤ[X] だから、整数になるわ。
そして、これらはα_1, …, α_n の任意の置換に対して不変だから
対称式の基本定理からα_1, …, α_n の基本対称式を変数とする整数係数の多項式で表せるわ。
以上からF(X)の係数は整数となって、したがって F(X) ∈ ℤ1[X] となるわ。
さて、P(X) が (X-α)^m を因数に持つとすると、
F(f(X)) = (f(X) - f(α))^m × (Xの多項式) だから F(f(α)) = 0、
そして m ≥ 2 なら任意の k < m について
F(f(X))のk階微分 (d/dX)^k F(f(X)) は (f(X) - f(α)) を因数に持つから X = α で値0を取るわ。
このことから F(f(X)) は (X-α)^m で割り切れ、したがってP(X)で割り切れるの。
F(f(X)) ∈ ℤ[X]であって、P(X) の最高次係数が1だから、割り算のやり方を考えれば
商の係数が全て整数となることがわかるから F(f(X))/P(X) ∈ ℤ[X] となるわ。
0360陽気な名無しさん2023/10/22(日) 09:15:50.56ID:kukMdiDj0
>>うさぎ
あなた戻ってきたのね、良かったわ。
>>354 はたぶんいつもの、おそらく専門家かそれに近い人よ。
monicは業界内では常識の言葉だから、わざとか無意識でか知らないけどここでも使ったのね。
s.t.はアタシが受けた授業の、とある教授は当たり前に使っていたわ。
業界内では当たり前に使う人もいるってことね。
ℤ1[X]に関しては一般的な記号ではないと思うけど、
『ℤ1[X]⊂ℤ[X]はモニックの集合』と書いているから、まあ、まあまあ。
繰り返しになるけど、全体的に業界内では普通に使う(人もいる)表現で書いてあるから、
わざとか無意識でか知らないけど・・・って、アタシはこの人
わざと業界人っぽさをひけらかしているんだと思うわ。
合成関数の表し方については、F⚪︎fと書いたら、
F(f(x))のつもりで書く人、f(F(x))のつもりで書く人、どっちもいるみたいよ。
だからこれは業界内でも同じ流儀の身内たちだけの中でならいいけど、
そうでないならどっちの意味で書いているのか明記しないとダメよ。
でも合成関数の記号のマルは、もっと小さいマルで書いていた記憶があるから、
これ見た時はちょっと違和感を覚えたわ。

解法については、何となくアタシも考えていたんだけど、
大筋同じような解法考えていたわ。
ただ、『さて、P(X) が (X-α)^m を因数に持つとすると、』から先、
微分とかまで考える必要あるかしら?
P(X) はモニックだから P(X)=(X - α_1) … (X - α_n) よね。
それで、F(f(X)) = (f(X) - f(α_1)) … (f(X) - f(α_n)) よね。
各 i=1, …, n に対して F(f(X)) の因数 (f(X) - f(α_i)) の X に α_i を代入すると0になるから、
この因数 (f(X) - f(α_i)) は (X-α_i) を因数に持つ。
だから F(f(X)) 全体では (X - α_1) … (X - α_n) = P(X) を因数に持つ。
これは重解を持とうと持つまいと(複数の α_i が等しかったりしたとしても)関係なく成り立つわ。
だから F(f(X)) は P(X) で割り切れる。
あと、F(f(X)) のその他の因数、つまり F(f(X))/P(X) が整数係数であることは、
『割り算のやり方を考えれば』でいいと思うわ。

これで良くない?
0361陽気な名無しさん2023/10/22(日) 09:35:04.20ID:4P24DPsi0
このことを端的に言うとなんだろうか?
学部生が習うような代数の言葉で表すと?
0362陽気な名無しさん2023/10/22(日) 10:01:27.26ID:PquMP1Mh0
みんな戻ってきたようで安心したわ。
0363Usagi2023/10/22(日) 12:33:06.06ID:qIszYqZg0
>>360
>s.t.はアタシが受けた授業の、とある教授は当たり前に使っていたわ。
授業なら先生が説明するだろうし、板書するの大変だから省略するのはわかるわ。
それに、もし授業が英語で行われるような場合なら、書くときはs.t.でも
口で言うときは such that て言うわけだからふつうにわかるだろうし。
でもこういうところでいきなり s.t. はないわ。

>全体的に業界内では普通に使う(人もいる)表現で書いてあるから、
そうだろうけど、上の二番めのリンクの方は、業界に蔓延している、
論理記号と一緒にコンマや s.t. を書いたり量化記号を後に書いたりする習慣を批判しているわ。
この例の場合
for all f∈ℤ[X], there is an F∈ℤ1[X] such that F⚪︎f/P∈ℤ[X]
という英文の一部の省略のために論理記号を使っているけれど
それは英語と論理という異なる言語をごっちゃにしていて言語道断ってこと。

>でも合成関数の記号のマルは、もっと小さいマルで書いていた記憶があるから、
正しい記号は f∘g かしら?
こういう問題もあるから、やはり記号の説明は必要なのよね。

解法についての指摘はその通りね!
(f(X) - f(α_i)) それぞれに因数定理を使えばいいのね。
アタシはつい F(f(X)) 全体に因数定理を使うことを考えちゃったから
重根があったらわからないわって思っちゃったの。

ていうか、このスレ5ch全体で見ても屈指の健全スレだと思うんだけど
なんでPINKに追いやられなきゃいけないのかしら
18禁よね? 本来の対象者である大学受験生の多くが立ち入り禁止になるわ?
0364Usagi2023/10/22(日) 12:34:12.31ID:qIszYqZg0
>>361
なんなのよ。思わせぶりに書かないで教えなさいよ。
0365陽気な名無しさん2023/10/23(月) 05:23:23.71ID:zV9Mewbs0
この問題を一言で表すと
ℤ[X]/(P)はℤ上整である
ということだ
だからもう少し環論的な証明があるかもしれない
0367陽気な名無しさん2023/10/23(月) 09:20:36.71ID:aUn6DPEh0
>>354 = >>361 = >>365 = >>366 かしら?
学部生が習うようなとか、環論的とか、加群とか、さすがにスレチよ。
数学板でやってちょうだい。
>>354 は、>>359 のように書かれていれば
大学入試レベルで解けるからスレチではないかもしれないけど、
書き方がダメよね。
0368陽気な名無しさん2023/10/23(月) 09:38:28.35ID:aUn6DPEh0
大学入試レベルで解けるってつい書いちゃったけど、本当かしら?
代数学の基本定理を当然のように使っているけど、
大学入試レベルで代数学の基本定理やってたかしら?
それとも代数学の基本定理使わないで解けるかしら?
0369Usagi2023/10/23(月) 16:55:36.90ID:cuLGepjl0
>>368
高校まででは代数学の基本定理も習わないし対称式の基本定理も習わないから、高校レベルでは解けないわよ
これらの定理を聞きかじったことのある高校生はそれなりにいるとは思うけど
証明まで読んだことのある高校生なんてほとんどいないでしょうし
それにfが整数係数だから書き換えた式も整数係数になるとか、環を意識しなきゃいけないポイントもあるわ
0370Usagi2023/10/23(月) 17:16:28.71ID:cuLGepjl0
>>365
興味深そうな話をしているのはわかるけど、同サロでやる意味がわからないわ
そういうことを知らないアタシみたいな人たちに説明してくれるならいいけどさ
別に義務ではないけれど、同サロって基本的にオネエ言葉での会話を楽しむ場所でしょ?
言葉が全くないか、あってもオネエじゃないから、言葉のやりとりを楽しむ目的ではなさそうね
かと言って数学的に専門的な話をしたいなら、こんなとこじゃなくて数学板の方が100倍適しているわ
(それとも書き込んでいないだけで数学科釜がけっこういて見てたりするのかしらw)
0371陽気な名無しさん2023/10/25(水) 07:08:38.81ID:3krOhf1x0
>>369
やっぱり高校レベルでは無理よね。
代数学の基本定理の証明なんて、アタシだって覚えていないわ。
というかちゃんと証明を理解して納得した記憶もないわ。

>>370
>>365 に関してなんだけど、
>可換環論において、可換環 B とその部分環 A について、B の元 b が A 係数のモニック多項式の根であるとき、b は A 上整である(integral over A)という。
B のすべての元が A 上整であるとき、B は A 上整である、または、B は A の整拡大(integral extension)であるという。(Wikipedia)
ということなんだけど、ℤ[X]/(P)ってℚ[X]/(P)つまり、ℚにPの根を添加した体の部分環でしょ。
ということはPはℤ係数のモニックなんだからこれがℤ上整なのは当たり前じゃない。
>>365 は当たり前なことしか言ってないと思うわよ。
でも多項式環を考える時、元である多項式を関数と見て合成関数を考えるなんてしたことないから、
このことが>>354とどう関係するのかはよくわからないわ。
0372陽気な名無しさん2023/10/25(水) 07:19:36.49ID:3krOhf1x0
数学板にも高校レベルのスレだってあるんだし、
純粋に数学要素だけを求めるなら数学板行けばいいのよ。
ここはやっぱり同サロ要素があってナンボだと思うわ。
それともスレチ連発するいやらしさが同サロっぽいとでもいうのかしら?
そういう同サロっぽさはいやだわ。
0374陽気な名無しさん2023/10/26(木) 13:50:53.17ID:YQsdlhAf0
>>373 は、アタシとうさぎ以外の釜にチャレンジしてもらいたいわね。
このスレにちょうどいい問題だと思うわ。
0375陽気な名無しさん2023/10/26(木) 17:54:08.30ID:9gTB3v3B0
厚かましいわね
アンタうさぎほど解いてないでしょう?
アンタが解きなさいよ
0376陽気な名無しさん2023/10/26(木) 18:10:57.70ID:YQsdlhAf0
厚かましいとかそういう問題ではなくて、
せっかく高校レベルの問題なんだから、
大学数学やってない人に解いてもらいたいのよ。
このスレが大学数学やった人だけが解くのは
残念なことだもの。
(うさぎも大学数学独学でかじってるわ)
もっと多くの釜に参加してもらいたいのよ。
0377Usagi2023/10/26(木) 21:57:37.65ID:2b5Q++bK0
>>371
レスありがとう。
ℤ[X]/(P)という記法がそもそもわからなかったから、ちょっと調べてみたんだけど
まず (P) はPによって生成されるイデアルで、Pの倍数の集合、つまり
(P) = { gP | g ∈ ℤ[X] } ということよね?
ℤ[X]/(P)はℤ[X]を(P)で割ってできる剰余環、つまり
ℤ[X]の要素の中で、その差が(P)の要素であるものを同一視して得られる環ってことよね?
アタシまだ環論勉強できていないから色々おかしなこと書いてたらごめんなさいね。
でもそうなら>>365は確かに>>354の書き換えなんだと思うわ。
Wikipediaの定義の可換環Bにあたるのが、この場合ℤ[X]/(P)でしょ。
f ∈ ℤ[X] に対して、ℤ[X]/(P)の要素である同値類でfが属するものを [f] と書くことにすると
[f] = { f + g | g ∈ (P) } よね?
F(f)がPで割り切れる、つまり F(f) ≡ 0 (mod P) なら、F([f]) = [0] となるから
ℤ[X]/(P)で考えると、F(Y) = 0 というℤ係数モニック方程式の根が Y = [f] であるということでしょう。
ただし、厳密にはℤ自体はℤ[X]/(P)の部分環じゃなくて、Wikipediaの定義の部分環Aにあたるのは
{ [z] | z ∈ ℤ } になるんじゃないかしら? でもこの集合をℤと同一視してℤと呼んでいいなら
任意の [f] ∈ ℤ[X]/(P) がℤ上整、つまり ℤ[X]/(P) はℤ上整、と言えるんじゃないかしら。

でもあなたはもしかして違う話をしているの? それとも実は同じなの?
>ℤ[X]/(P)ってℚ[X]/(P)つまり、ℚにPの根を添加した体の部分環でしょ。
ここがよくわからないわ。Wikipediaの「剰余環」を読むと、確かに
「剰余環は体の拡大を構成する」と書かれていて、例えば剰余環ℝ[X]/(X^2+1)は複素数体ℂと同型、とあるわ。
アタシなりの理解では、X^2+1 = 0 の根のどちらか、例えばiをXで表すことにすれば
ℂとℝ[X]/(X^2+1)の間に a + bi ↔︎ [a + bX] という同型対応が作れるってことかしら?
一般に体Kに対して、K/(P)はKにPの根を添加した体と同型になるの?
上の例みたいに Pが2次式で P(X) = X^2 - cX + d なら、その根のひとつをXで表せば
もうひとつは根と係数の関係から c-X と表せるから、このことが言えそうなのはわかるけれど
Pが3次以上でも同様のことが言えるのか、未勉強のアタシにはわからないわ。

もうひとつ気になった点は、ℚ[X]/(P)がℚにPの根を添加した体であると書かれているけれど
ℚ[X]/(P)は、ℚにPの根を添加した体に同型なだけで、イコールではないのよね?
上の例だとℂとℝ[X]/(X^2+1)は同型だけど ℂ = ℝ[X]/(X^2+1) ではないのよね?
「ℚにPの根を添加した体の部分環」というのは、数の集合よね。
一方、ℤ[X]/(P) はXの多項式の集合の集合よね。
>ℤ[X]/(P)ってℚ[X]/(P)つまり、ℚにPの根を添加した体の部分環でしょ。
この言い方は、これらを同型対応を通して同一視しているの?
そしてそうだとすると、「ℤ上整なのは当たり前」なの?
>>365>>354の単なる書き換えだから、365が当たり前ということは
アタシが>>359で証明したことは証明するまでもなかったってことになるわ。
当たり前というのは勘違い? それとももっと深い意味で当たり前なの?
ああっ、もういろいろわかんない!
ごめんなさい、狂ったように書き散らしてしまって…
0378陽気な名無しさん2023/10/27(金) 07:36:53.04ID:W+xQRGGA0
話がよく見えんが、とりあえずQ[X]/(P)は体になるとは限らない
0379陽気な名無しさん2023/10/28(土) 08:43:34.74ID:u669qhtc0
>>377

あなたの学習能力は凄いわね。尊敬するわ。

アタシゃ昔やったうろ覚えの記憶で書いてるから、

アタシの方こそ色々おかしなこと書いてるような気がするわ。

だいたい元々Pはモニックの前提だったのに、

いつの間にPは既約(しかもある代数的数の最少多項式)のイメージで書いてしまってたようだし。

(>>378 が指摘してるのはたぶんここではないかしら)

あなたの説明で、>>365>>354の書き換えであることが分かった気がしたわ。

ありがとうね。



ただね、

>ただし、厳密にはℤ自体はℤ[X]/(P)の部分環じゃなくて、Wikipediaの定義の部分環Aにあたるのは

{ [z] | z ∈ ℤ } になるんじゃないかしら?

この部分は、{ [z] | z ∈ ℤ } = ℤ になると思うわよ。

なぜなら P はモニックなんだから degP ≧1としていいわよね。そうすると(P)の要素は0以外はすべて deg ≧1でしょ。

そうすると

>[f] = { f + g | g ∈ (P) }

なんだから [z] (∀z ∈ ℤ) は z と異なるあらゆる z' ∈ ℤに対して [z'] とは同じ剰余類にはなりえないでしょ。

この段階で、

> >>365>>354の単なる書き換えだから、365が当たり前ということは

アタシが>>359で証明したことは証明するまでもなかったってことになるわ。

これは言えてるのではないかしら。



そこから先の体の拡大の話、

>一般に体Kに対して、K/(P)はKにPの根を添加した体と同型になるの?

これはPがK上代数的な元の最少多項式なら成り立ったと記憶しているわ。

もしそうならならPの一つの根をαとすれば、1,α,・・・,α^(degP-1)

を基底とするK上 degP 次の拡大体になったはず。

もちろん同型対応を通して同一視している表現よ。
0380陽気な名無しさん2023/10/28(土) 08:44:51.68ID:u669qhtc0
ヤフーメールで下書きして、そのままコピペしたら一行おきになってたわ。
ごめんなさいね。
0381陽気な名無しさん2023/10/28(土) 09:18:19.07ID:u669qhtc0
間違えちゃいけないと、ちょっと慎重な表現してたけど、
モニックが既約であることと、ある代数的数の最少多項式であることは同値みたいね。

ttps://math-notes.info/wp-content/uploads/2021/08/field-3.pdf
0382陽気な名無しさん2023/10/28(土) 09:37:44.64ID:u669qhtc0
>>378
納得したわ。
Pが既約でない時、P=fgとすると、
ℚ[X]/(P)∋f,g はどちらも0ではないが、fg=0 になるわね。
零因子を持つから、体どころか整域ですらないわね。
既約なら体になることは>>379>>381 で大丈夫ね。
0383陽気な名無しさん2023/10/28(土) 10:18:45.54ID:9+wDBC/80
>>379の話はよく分からんが
たしかにQ[α]=Q(α)になることを理解するだけでもそれなりの代数学の素養が必要
0384陽気な名無しさん2023/10/28(土) 10:40:45.65ID:u669qhtc0
>>383
そうね。
すくなくともうさぎが独学でスムーズに理解できるようにキーワードを書いておくわ。

(P)は素イデアル
ℚ[x] はP.I.D.
P.I.D.では素イデアル⇔極大イデアル
極大イデアルによる商環は体

これくらい書いておけば十分でしょう。
0385Usagi2023/10/28(土) 22:27:31.79ID:Fa3w/DfT0
>>379
いろいろ補足ありがとう。
> [z] (∀z ∈ ℤ) は z と異なるあらゆる z' ∈ ℤに対して [z'] とは同じ剰余類にはなりえないでしょ。
これはわかってたんだけど、{ [z] | z ∈ ℤ } と ℤ は同型なだけで、厳密には異なるものでしょ?
前者は多項式の同値類(つまり多項式の集合)の集合で、後者は数の集合だもの。
代数では、同型なものを = としちゃうのが一般的な習慣なの?
なんか気持ち悪く感じちゃうんだけど…

>> >>365>>354の単なる書き換えだから、365が当たり前ということは
>アタシが>>359で証明したことは証明するまでもなかったってことになるわ。
>これは言えてるのではないかしら。
えーと、任意の ℤ[X]/(P) の元fは ℚ[X]/(P) の元でもあり
つまりℚにPの根を添加した体の要素であるってことよね?
でもこのことからfがℤ係数のモニック多項式の根と言えるのかしら?
もちろんfがPの根の場合は、それはℤ係数のモニック多項式の根だけど
例えば f = 1/2 なら、これはℤ係数のモニック多項式の根じゃないんじゃないの?
あとこれって、あなたが、Pが既約だと思い込んでいて
その結果ℚ[X]/(P)をℚにPの根を添加した体だと思ったから言ったことなのよね?
そうでない場合は、別の議論が必要なんじゃないのかしら?

>これはPがK上代数的な元の最少多項式なら成り立ったと記憶しているわ。
>もしそうならならPの一つの根をαとすれば、1,α,・・・,α^(degP-1)
>を基底とするK上 degP 次の拡大体になったはず。
これはとても興味深いわ。じゃあK上既約なn(>1)次方程式の根のひとつをαとすると
残りのn-1個の根はすべてαのK上の(n-1)次以下の式で表すことができるのね?
いずれにせよ、このこと自体が当たり前とは言えないレベルの話だから
Pが既約な時あなたの議論が正しいとしても>>354が当たり前と言っていいのか微妙な気がするけど…
0386陽気な名無しさん2023/10/29(日) 11:27:56.88ID:LnxNFhA30
てすと
0388陽気な名無しさん2023/10/29(日) 13:20:54.53ID:8rEB/JGo0
>>387
>>373 が先よ
0389陽気な名無しさん2023/10/31(火) 08:40:00.81ID:FVOckjyx0
>>385
>同型なだけで、厳密には異なるものでしょ?
これね、その疑問はよくわかるわ。
代数で同型なものっていろいろあるけど、
いろいろ証明して同型だ、って判明するものもあれば、
自然に同一視できる自明な同型(canonicalな同型とかいうわね)
もあって、自明な同型は場合によて同一視することもあるのよ。
同一視した方が便利だったり理論がわかりやすくすっきりすることもあるし。
例えば線形代数の「双対空間」って知ってるかしら。
双対空間の双対空間は、最初の空間に同型になるんだけど、
これは自然な同型になるから普通同一視するわ。
今回の場合、{ [z] | z ∈ ℤ } と ℤ は自然な同型でしょ。
同一視して差し支えない例だと思うわ。

>> >>365>>354の単なる書き換えだから、365が当たり前ということは
>アタシが>>359で証明したことは証明するまでもなかったってことになるわ。
>これは言えてるのではないかしら。
これに関しては、あなた >>377
> でもこの集合をℤと同一視してℤと呼んでいいなら
>任意の [f] ∈ ℤ[X]/(P) がℤ上整、つまり ℤ[X]/(P) はℤ上整、と言えるんじゃないかしら。
って言ってたでしょ。その通りだと思うし、
上記の自然な同型でこの話は終わっていると思うわ。
体の部分環とかいう話はここでは必要ないと思うわ。

>これはPがK上代数的な元の最少多項式なら成り立ったと記憶しているわ。
>もしそうならならPの一つの根をαとすれば、1,α,・・・,α^(degP-1)
>を基底とするK上 degP 次の拡大体になったはず。
これはね、K上既約なn(>1)次方程式の根のひとつをαとして、
他の根の一つをβとしたときに、K(α)とK(β)は同型なのは間違いないのだけれど、
同一になるとは限らないのよね。
だから、
>K上既約なn(>1)次方程式の根のひとつをαとすると
>残りのn-1個の根はすべてαのK上の(n-1)次以下の式で表すことができるのね?
これは成り立たないわ。
例えばℚ上で考えるとして2の4乗根の最少多項式は x^4-2 になるけど、
これの4っつの解は ±(2の4乗根), ±(2の4乗根)i になるわよね。
ℚ(2の4乗根)=ℚ(ー2の4乗根)ではあるけれど
ℚ(2の4乗根)≠ ℚ((2の4乗根)i )よ。
(上は実数体ℝの部分体だけど下は違う)だから、
この場合、ℚ(2の4乗根)はℚ上4次だけど、最少多項式 x^4-2
の根全てを添加した体(ℚ上の x^4-2 の分解体って言うんだけれど)
ℚ(2の4乗根, i) はℚ(2の4乗根)上2次、
ℚ上だと8次の体になるわ。
ちなみに余談だけど分解体の、元の体上の自己同型群を考えた時、
その部分群たちの一つ一つとその分解体の中間体の一つ一つが、
その部分群による固定体(その部分群によって移り合わない元全体からなる体)
という関係で完全に1:1対応する
(部分群の指数と体の拡大次数の関係も完全に一致する)
というのが体のガロア理論よ
0390陽気な名無しさん2023/10/31(火) 12:13:38.34ID:pLlvhAYD0
この人、アタシたちとは整の定義が違うのかしら
0391陽気な名無しさん2023/10/31(火) 15:17:56.07ID:FVOckjyx0
>>390
整の定義はずっと >>371 にあるwikiの定義でやってきてるわよ。
あなたたちの「たち」がどんな人たちを含むのかは知らないけど、
あなたたちはwikiの定義と違う定義でやっているのかしら?
それは一体どんな定義なのかしら?
0392Usagi2023/11/03(金) 00:19:21.12ID:Xc1Wtdwf0
>>389
なるほどね、自然な同型の場合は同一視するのね。

>上記の自然な同型でこの話は終わっていると思うわ。
>体の部分環とかいう話はここでは必要ないと思うわ。
ちょっとよくわからないわ。
あなたは、>>365>>354の書き換えであることを気づかずに
(☆)  ℤ[X]/(P)はℚ[X]/(P)の部分環だからℤ上整である
と書いたと思うのよ。
もちろん、>>359でアタシが書いた証明を前提にすれば、365は354の書き換えだから当たり前と言えるけれど
でもあなたは、>>379で359の証明は必要なかったって書いたと思うの。
結局359は必要なの?
アタシがわからないのは、直接(☆)が当たり前のものとして理解できるのかしら、ということよ。
例えば、もしℚ[X]/(P)がℤ上整なのだとしたら(☆)が言えるんだろうけど、そうは思えなかったの。

最後の部分については納得したし、勉強意欲が湧いてきたわ。ありがとう。
あなたが>>371で「ℚにPの根を添加した体」って書いた時、
「ℚにPの根のひとつを添加した体」の意味で書いていたのね。
でもアタシは「ℚにPの根のすべてを添加した体」の意味だと思ってしまったのよ。
日本語はこういうところが難しいわね。
0393陽気な名無しさん2023/11/03(金) 10:50:17.06ID:gfK4fzRt0
いやにネチっこい書き方するわね
はっきり言ってやりなさいよアンタはアホだって
0394Usagi2023/11/03(金) 11:06:03.09ID:Xc1Wtdwf0
>>393
消えなさい
実生活でストレス溜まってるのか知らんが、匿名掲示板で他人をそしるとか
他人のことをとやかく言う前に、アンタはまず自分を見つめ直しなさい
仲良く交流する気がないなら来ないで
0395陽気な名無しさん2023/11/03(金) 11:15:31.28ID:gfK4fzRt0
アタシャこのやりとりにうんざりしてんのよ
当たり前と主張する人は一向に証明しないしそれをうさぎがいやらしい言い回しでネチネチとせめるのも
そろそろいい加減にしなさいよ
0396Usagi2023/11/03(金) 11:58:54.49ID:Xc1Wtdwf0
あなたちょっと捻くれてないかしら
もっと物事を単純にとらえたら?
アタシはせめてるわけじゃないわ
人の書き込みで理解できなかった部分、共通見解に達していないと思われる部分について問うているだけよ
よくわからない部分をそのままにするのは嫌だからよ
それに、そうではないのに表面的にすべて納得したように振る舞うのは誠実じゃないから
(これって対面だと結構難しいから、ある意味掲示板のメリットだと思うわ)
だから、どこですれ違っているのかはっきりするように丁寧に書いているだけよ
まあ、丁寧に書いているのはいつもだけど。
それは、アタシの考えがなるべく正確に読み手に伝わって欲しいからよ
それをいやらしいだとか言われたら、じゃあどう書いて欲しいのよ?
0397Usagi2023/11/03(金) 12:08:32.28ID:Xc1Wtdwf0
もしかして>>390もあなた?
390みたいに具体的な問題点を指摘しないものは、ただのいいがかりと変わらないのよ
0398陽気な名無しさん2023/11/03(金) 12:25:25.08ID:gfK4fzRt0
はい?
あまりにも話が噛み合わないからアンタの使ってる整の定義は何?と問い糺すのが言いがかりなの?
アンタ、ちょっと捻くれすぎてない?
0399Usagi2023/11/03(金) 13:40:14.11ID:Xc1Wtdwf0
なんでそんなに喧嘩腰なのかしら。
定義はWikipediaを引用しているじゃないの
あなたの定義がそれと異なるなら、「私の知っている定義はこうなんだけど…」と具体的に述べればいいわ
定義は同じだけれど、そのあとの議論でおかしいと思う部分があるのなら、
「…の理由で、議論がおかしいと思うんだけど」と、やはり具体的に述べればいいのよ
(あたしが>>392で書いたのはそういうものよ)
それはいやらしいとかいう問題ではなく、学問的に至極当然の態度よ
具体的な批判を述べずに嫌味や罵倒を言うのは、学問的態度の対極にあるのよ
小学生の口喧嘩と変わらないわ
0400陽気な名無しさん2023/11/03(金) 13:54:05.69ID:fMv6n82v0
それならあとは>>389が弁明するだけね
どう説明してくれるのか楽しみにしてるわ
0401陽気な名無しさん2023/11/03(金) 14:08:20.48ID:L4C0hZuG0
それならアタシは>>391の学問的態度を非難したいわ
ここに少なくとも2人(Usagiの自演でなければだけど)の混乱したオンナがいるんだからさっさと説明しなさいよ
煙に巻くようなことばかり言って証明しようとしない態度こそ真摯な学問的態度とはかけ離れていると言えるわ
0402陽気な名無しさん2023/11/03(金) 14:21:25.90ID:gfK4fzRt0
何よりもまずおかしいと指摘できるくらいの議論を見せてほしいわw
0403陽気な名無しさん2023/11/04(土) 09:55:19.16ID:2/nTeCmd0
>>うさぎ
>>400
ごめんなさいね。
あと一週間くらい忙しくて、しばらく回答できそうにないわ。
一週間ずっと殺人的に忙しいわけではないんだけれど、
キチンと回答するためには、>>392 やそれ以前のやり取りを
しっかり読み返して論点整理して、じっくり考え直して、
さらに回答の文章を丁寧に書かなきゃいけないでしょう。
それにはまとまった時間が必要そうだから、しばらくはごめんね。

ところでアタシはうさぎがねちっこいとは思ったこともないわよ。
うさぎの指摘は具体的で、挑発的な発言もないし、
いつも真摯なレスをくれて感謝しているわよ。
このスレが良スレならば、それはひとえにうさぎのおかげだと思っているわ。
それに引きかえ>>400 はこれまでのイヤミな人と同一人物だと思うけど、
指摘するにしても具体的ではないことが多く、挑発的な発言が多くて嫌になるわ。

ちなみにアタシがあまり問題を解かないのは、
パッと見てすぐにわかるような問題ならば、他の誰かが解くべきだと思うし、
じっくり考えないとならないような問題ならば、なかなかその時間が取れないからよ。
まあ最近は>>400 らしき人が出す、完全にスレチな問題に、
うさぎが頑張って解くってパターンが多いようだけれども。
それより珍しく>>373 に、このスレにふさわしい問題が出されたから、
それを高校数学までしかやっていない釜に解いてほしいと思っているわ。
それがこのスレの本来の姿だと思うから。
0404陽気な名無しさん2023/11/05(日) 06:14:04.44ID:R5/eDDzn0
問題解けない人が
この問題はまだ解くな、あっちが先だ
とかいちいち指図しないでほしいわ
お門違いも甚だしいのよ
どの問題を解くかは問題を解ける有能な人に任せておけばいいと思うの
0405陽気な名無しさん2023/11/05(日) 07:14:12.87ID:d5pWgaGy0
スレタイ読めない人が書き込みしないでほしいわ
0406陽気な名無しさん2023/11/05(日) 07:40:55.47ID:l6LzFpCq0
解きもしないのに異様にスレタイに拘泥るのやめてほしいわ
0407陽気な名無しさん2023/11/05(日) 12:43:07.75ID:PzH8Zoxc0
>>405
たしかに…
解くゲイのスレなのに何で解かない人が出しゃばってくるのかしら
0408Usagi2023/11/05(日) 23:17:41.86ID:qOvk+zjX0
403=ものぐささんとして話を進めるけど、
ものぐささんは指図ではなくて希望のつもりで書いたんだろうとは思うわ。
でも反発する人が出るのもわかる。
でもね、アタシはみんなに仲良くしてほしいの。

問題解かない人が、って非難しているけど、じゃあみんなも解いてよ!
このスレで問題解いているの、大半がアタシだけど
アタシ以外が解いたもののいくつかは、ものぐささんによるものだと思うわよ。

このスレ、同サロがPINKに来る前はIDなしだったじゃない?
だからどれくらい人がいるのかさっぱりわからなかったの。
最悪、アタシとものぐささんと他一人で3人しかいない可能性すらあると思ってたのよ。
でもIDありになって、今日の書き込みから、アタシとものぐささん以外にも
4人はいることがわかって、すごく嬉しかったわ。
書き込まないで読んでいる人もいるだろうから、案外多くの人が見ていることがわかったわ。

実をいうと、アタシかなり孤独に感じてて辛かったのよ。
もしかして、3人しかいないスレで、ある一人が出し続ける難しい問題を
アタシが解き続けているだけだったらどうしよう、って時々頭をよぎってw
まあアタシが解き続けるから問題がどんどん難しくなってきた節もあるけど
でもそれで余計にアタシが解かないと誰も解きそうにない感じになってきて…
本当は、いろんな人の解き方を見たり、それについて話し合ったりしたいの…

もうスレタイと実態が全然合っていないけど
個人的には大学入試レベルにこだわる必要はないと思っているわ。
高校で何を習うかは、文科省が勝手に決めているだけで数学的に何も意味はないし
それに、まだ解けていない問題は高校レベルで解けるかどうか分からないものね。
実際、京大の入試問題を考察してたら高校レベルを超えた話になったりしたじゃない?
でもそれは自然なことだし、むしろ興味深いことだわ。

ただね、問題を出す人は、高校レベルの数学の知識で理解できるように書いて欲しいと強く思っているの。
要するに、大学数学を勉強した人でないと理解できない表記はやめて欲しいの。
(それがやりたいなら数学板にいくべきだわ)
アタシとものぐささんしかスレにいないなら実際上あまり問題なかったかもしれないけれど
他にも多くの人が見ていることがはっきりした以上、これは守って欲しいわ。
そもそも大学数学で使う記号って、本や分野が違えば違う意味になったりするわけで
実際、数学書でも記号は巻頭か巻末にリストになっていることが多いわよね。
高校までで習う記号は一般常識と考えられるからいいけれど、それ以外のものは必ず説明を付すべきね。
0409陽気な名無しさん2023/11/09(木) 15:32:00.02ID:jEJpfjjc0
>>うさぎ
とりあえず時間みつけて読み返してみたわ。
まず分かったのは、アタシは>>377 を勘違いしていたわ。
うさぎはこの一段目でで、>>365>>354 が書き換えであることを示しただけだったのね。
てっきり、365を証明しようとしていて、{ [z] | z ∈ ℤ }がℤと同一視できるかどうかが
証明の障壁になってるものだと勘違いしたのよ。
だから>>379 では同一視できることを説明して、それでOKだと思い込んでいたの。
アタシがきちんと読んでいなかったようだわ。ごめんなさいね。

それで、>>385 の2段目の
>例えば f = 1/2 なら、
これは
>ℤ[X]/(P) の元f
の仮定に反するわよね。
もしℚ[X]/(P)が体ならば既約でモニックであるPの根αを添加したものだから、
ℤ[X]/(P) の元fは整数係数のαのn-1次以下の式で表されるわ。
それで、モニックの根となる数のことを代数的整数って言うんだけど、
代数的整数同士加法減法乗法したものはやはり代数的整数であることが知られているの。
このことのきちんとした証明は、代数的数(有理数係数の多項式の根)が
加減乗除で閉じていることの証明と同じようにするのだけれど、
詳しくは、例えば高木貞治先生の「代数的整数論」なんかにきちんと載っているわ。
だからℚ[X]/(P)が体ならばℤ[X]/(P)はℤ上整であることが言えるわ。

でも、
>あとこれって、あなたが、Pが既約だと思い込んでいて
その結果ℚ[X]/(P)をℚにPの根を添加した体だと思ったから言ったことなのよね?
そうでない場合は、別の議論が必要なんじゃないのかしら?
これは全くその通りだわ。
Pが既約でない時の、環論的な証明は今のところ思い浮かばないから、
やっぱりその場合は>>359 がわかりやすいと思うわ。
そもそも個人的に整域でない環は扱うの苦手だわ。
0410陽気な名無しさん2023/11/09(木) 15:40:12.26ID:jEJpfjjc0
>>408
それから、同じ日付でIDが異なるからって、別人だとは限らないわよ。
やっぱり書き込み内容や口調で判断する方がまだ妥当ではないかしら。
0411Usagi2023/11/09(木) 23:06:52.43ID:ECaIrZJc0
>>409
アタシが分からないのは↓なの。

(☆)  ℤ[X]/(P)はℚ[X]/(P)の部分環だからℤ上整である

なぜ f = 1/2 という例を出したかというと、はじめに(☆)を見た時、

(★) ℚ[X]/(P)はℤ上整であり、ℤ[X]/(P) ⊆ ℚ[X]/(P) なのでℤ[X]/(P)はℤ上整である

と言いたいのかと思ったのよ。 1/2 ∈ ℚ だから 1/2 ∈ ℚ[X]/(P) よね?
けれど 1/2 はℤ上整でないからℚ[X]/(P)はℤ上整ではないので、(★)は間違っているわよね。
じゃあ(★)の意味でないのなら、(☆)はどういう根拠なのかしら、とききたかったの。

>ℤ[X]/(P) の元fは整数係数のαのn-1次以下の式で表されるわ。
↑はOKよ。でも↑と↓がどう関係があるのか、分からないわ。
>代数的整数同士加法減法乗法したものはやはり代数的整数であることが知られているの。
これはℤ上整なもの同士で加法減法乗法してできるものは、ℤ上整という意味よね?
でもそこからなぜ↓が結論されるのかアタシには理解できないわ。
>だからℚ[X]/(P)が体ならばℤ[X]/(P)はℤ上整であることが言えるわ。

f∈ℤ[X]/(P) が整数係数のαのn-1次以下の式で表されるのはいいんだけれど、
今示したいことって、そういうfがℤ上整、つまり整数係数のモニックの根であることよね?
Pの根をα_1, …, α_nとして、fがn-1次以下の整数係数のgを用いて f = g(α_1) と表されるとして
fがℤ上整であることを示すには、例えば (X-g(α_1)) … (X-g(α_n)) が整数係数になるとか
そういうことを言わなければいけないんじゃないのかしら?
0412Usagi2023/11/10(金) 01:11:07.53ID:Q9WevY+D0
ていうか f ≡ g (mod P) よね
だからこれって結局>>359に書いたものと同じことよね…
0413陽気な名無しさん2023/11/12(日) 09:04:00.44ID:0xDWsMhK0
>>411
とりあえず(★)は間違っているわ。
>じゃあ(★)の意味でないのなら、(☆)はどういう根拠なのかしら、とききたかった
これを説明すればいいのね。そのためには
>代数的整数同士加法減法乗法したものはやはり代数的整数であることが知られている・・・(a)
これが必要なので、これは認めてね。又は自分で調べてね。(wikiなんかでも参考になるかも)

ではまず、
「αをPの根の一つとしたとき、ℚ[X]/(P)=ℚ(α) が体ならば、Pはモニックなので、αは代数的整数」
これは代数的整数の定義「モニックの根となる数のことを代数的整数と言う」より、大丈夫よね。
それで、
>ℤ[X]/(P) の元fは整数係数のαのn-1次以下の式で表されるわ。
↑はOKよ。
と書いてくれたので、これを使うわ。
fを表している整数係数のαのn-1次以下の式の各項についてみると、
係数は整数だからこれは代数的整数、あとはαのべきだからやはり代数的整数の積。
よって各項は(a)によって代数的整数であることがわかる。
するとfを表している整数係数のαのn-1次以下の式は各項の和又は差だから、
やはりf自体も(a)によって代数的整数であることがわかる。
以上より、ℤ[X]/(P) の元fは必ず代数的整数、つまりあるモニックの根であるので、
ℤ[X]/(P) はℤ上整であることが言える。

かなり丁寧に説明したつもりだけど、これでどうかしら。
ただ、これだと一見、「ℚ[X]/(P)=ℚ(α) が体ならば」という条件はどこで必要なの?
という疑問が出てくるかもしれないわね。

これについては、
ℚ[X]/(P)=ℚ(α) が体ならば、ℚ[X]/(P)⊃ℤ[X]/(P)=ℤ[α] であることが言えるので、
うさぎが認めてくれた
>ℤ[X]/(P) の元fは整数係数のαのn-1次以下の式で表されるわ。
↑はOKよ。
これが確かにOKなの。
でも、ℚ[X]/(P)が体でないならば、Pが既約でないから、
ℚ[X]/(P)にも、ℤ[X]/(P)にも零因子が存在するので、
(P=P’・P''としたとき、[P']と[P'']は共にℚ[X]/(P)やℤ[X]/(P)の元で
[0]とは異なるが、[P']・[P'']=[P]=[0]になるわ)
αをPの根の一つとしたときに、整数係数のαのn-1次以下の式で表されるものは零因子ではないから、
(これはαの最少多項式でありPの既約因数であるP’に対してℚ[X]/(P’)が体になり、
ℚ[X]/(P’)=ℚ(α)や ℤ[X]/(P’)=ℤ[α] に対しては前半の議論が成り立つわ)
ℤ[X]/(P) の元fの中には整数係数のαのn-1次以下の式で表されないものも存在してしまうのよ。
だから「ℚ[X]/(P)=ℚ(α) が体ならば」という条件はどうしても必要なの。
0414陽気な名無しさん2023/11/12(日) 12:53:21.48ID:LlqaUPnL0
やっとこのアホの言うことが分かってきたかも…
0415Usagi2023/11/13(月) 04:38:17.28ID:m/ukN7k50
>>413
>「αをPの根の一つとしたとき、ℚ[X]/(P)=ℚ(α) が体ならば、Pはモニックなので、αは代数的整数」
>これは代数的整数の定義「モニックの根となる数のことを代数的整数と言う」より、大丈夫よね。
ちょっと文章がおかしくなっている気がするんだけど、きっと
・仮定からPは整数係数のモニック多項式なので、αをPの根の一つとすると、αは代数的整数
・ℚ[X]/(P)が体ならば、ℚ[X]/(P)=ℚ(α) となり、ℤ[X]/(P) の元fは整数係数のαのn-1次以下の式で表される
とおっしゃりたいのよね?
そして代数的整数は単なる「モニック多項式の根」ではなくて「整数係数のモニック多項式の根」よね?
そうなら、前半は理解できたわ。ありがとう。
>>409を読んだ時は、αが代数的整数だということがアタシの頭から抜け落ちていたから分からなかったの。

高木本は持ってないけれど、前スレで紹介していただいた草場(著)「ガロワと方程式」のp. 80に
「2個の代数的数a, bに対してa+b, a-b, ab, a/bも代数的数である」
という定理はあったわ。これの証明を代数的整数に限定してa/bを無視したら、そのまま
「代数的整数同士加法減法乗法したものはやはり代数的整数である」の証明になりそうだわ。
ある行列の行列式 = 0 の話に帰着させている証明なんだけど。
(でもこの本の証明、p. 82に出てくるcたちの添字が間違っている気がするわ。
 本を持ってる人にしか分からないこと書くけど、kが変わればcたちも変わると思うから。)

>αをPの根の一つとしたときに、整数係数のαのn-1次以下の式で表されるものは零因子ではないから、
ここでアタシ混乱しているんだけど、「零因子」って「ℤ[α]の 零因子」という意味かしら?
ちょっと基本的なこと確認させていただきたいのだけれど
ℤ[α]というのは、あくまでαを文字と見た多項式の集合なの?
それとも、そういう多項式のαに特定の値を入れてできる数の集合なの?
例えば P(X) = (X^2-2)(X^2+1) なら根のひとつとしてα=√2を考えることができるけれど
α^2-2 ∈ ℤ[α] はαを文字とする多項式? それとも α^2-2 = 0 なの?
ℤ[X]/(P)=ℤ[α]と書かれているから、ℤ[α]の要素としての (α^2-2)(α^2+1) を 0 と同一視しているのかしら?
てことは数として見ていて α^2-2 = 0 ってこと? でもそれならα^2-2 は零因子よね?
それとも、多項式として見ているってことかしら?
でもそういう解釈ならば、ℤ[α]はn-1次以下に限らず零多項式以外に零因子はないことになるから
「n-1次以下」とか限定して書かれているのは変よね…?
(そもそも、まず零多項式もn-1次以下の多項式で零因子だと理解しているんだけれど、これは除外して考えられているの?)

ところでPが既約でない場合の話だけれど
P = P_1^{n_1}…P_k^{n_k} とℤ上で既約な式の積に分解しておけば
任意の f ∈ ℤ[X] について、あなたの説明から、各 P_i に対してあるモニックの F_i ∈ ℤ[X] があって、
F_i(f) ≡ 0 (mod P_i) となることがすぐわかるのよね。
それならば F = F_1^{n_1}…F_k^{n_k} とおけば、F(f) ≡ 0 (mod P) となるんじゃないかしら?
このことからℤ[X]/(P)はℤ上整であると言えるんじゃない?
0416陽気な名無しさん2023/11/13(月) 10:54:51.05ID:IZkEuANU0
>>415
うさぎ、あなた何ちゅー時間に書き込んでるの?
いつ寝てるのかしら?
まあ、それはそうと、順番に回答していくわね。

文章おかしかったかしら?
でもまあうさぎの解釈でOKだからいいわ。
もちろんモニックといえば整数係数モニックのつもりで書いているわ。
有理数係数ならわざわざモニックって書く意味がないから
整数係数は省略してもわかると思って省略しちゃったわ。

「代数的整数同士加法減法乗法したものはやはり代数的整数である」
の証明については、高木本でも行列式使っていたみたいだから、
多分うさぎの考え通りだと思うわ。
係数を整数に限定して同様にやればいいらしいわ。
アタシは以前この証明をきちんと追った記憶もあるような気がするけど、
今はもう面倒で追う気になれないわ。

「零因子」という言葉について、まずアタシはこの言葉を
「零元以外の者同士の積が零元になるもの」という意味で使っているから、
アタシは零元は零因子とは呼んでいないわ。
>「零因子」って「ℤ[α]の 零因子」という意味かしら?
「ℚ[X]/(P)やℤ[X]/(P)に零因子があるかないか」の話で、
Pが既約な時はℚ[X]/(P)は体で=ℚ(α)と書けるし、ℤ[X]/(P)も=ℤ[α]と書けるし、
このときℤ[α]には、もちろんℚ(α)にも零因子はない、だけど
Pが既約でない時はℚ[X]/(P)やℤ[X]/(P)に零因子がある、
ということが言いたかったのだけれど、
そうね、そもそもPが既約でない時は、αなんて持ち出すのはナンセンスだから、
>αをPの根の一つとしたときに、整数係数のαのn-1次以下の式で表されるものは零因子ではないから、
この表現はまずい、というかそもそも
>ℤ[X]/(P) の元fは整数係数のαのn-1次以下の式で表されるわ。
ということ自体が言えないので、>>413 の前半の議論が成り立たない、
というべきだったかもしれないわね。
「そもそもPが既約でない時は、αなんて持ち出すのはナンセンスだから」については
後でまた書くわね。
長くなるから一度切ろうかしら。
0417陽気な名無しさん2023/11/13(月) 10:56:51.32ID:IZkEuANU0
つづき
>ℤ[α]というのは、あくまでαを文字と見た多項式の集合なの?
それとも、そういう多項式のαに特定の値を入れてできる数の集合なの?
この疑問、言われてみて、これは非常に重要な、
ハッキリさせておかなければならないことだと気づいたわ。
基本的にℤ[α]でもℤ[x]でもなんでも、ℤ[(何か文字)]と書けば、
(何か文字)の多項式環なのだけれども、
ℚ[X]/(P)やℤ[X]/(P)においてはdegP以上の次数の多項式は
P=0という関係式でdegP未満の次数の多項式と等しいことになるから、
degP未満の次数の多項式からなる環、ということになるわね。
さらにPが既約であれば、この場合だけは『特別』で、
Pの任意の根αを一つ持ってくれば
ℚ[X]/(P)やℤ[X]/(P)はℚ(α)やℤ[α](特定の値を入れたもの)
と「自然に同型」になるから、これらを同一視しても構わない、ということ。
Pが既約でない場合、同型が成り立たないから、
ℚ(α)やℤ[α]と同一視することは出来ないわ。
「そもそもPが既約でない時は、αなんて持ち出すのはナンセンスだから」
というのは、そういうこと。

うさぎの出してくれた例で言えば、
P(X) = (X^2-2)(X^2+1) = X^4-X^2-2を考えると、
Pの根は±√2と±i の四つよね。
ℚ[X]/(X^2-2)はℚ(√2)ともℚ(-√2)とも自然に同型、
ℤ[X]/(X^2-2)はℤ[√2]ともℤ[-√2]とも自然に同型になるから、
例えばℚ[X]/(X^2-2) = ℚ(√2)、ℤ[X]/(X^2-2) = ℤ[√2]
と書いてしまっても(勿論√2でなくて-√2でも構わない)代数的には差し支えないけれど、
ℚ[X]/(X^4-X^2-2) や ℤ[X]/(X^4-X^2-2) は、X^4-X^2-2と0を同一視はしているけれど、
(X^2-2)と(X^2+1)のどちらか一方を0と同一視しているわけではなく、
単なるこの環の元である多項式に過ぎないから、
例えばℚ(√2)やℤ[√2]、またはℚ(i)やℤ[i]なんてものを考えても、
それはℚ[X]/(X^4-X^2-2) や ℤ[X]/(X^4-X^2-2) とは全く別物、同型ですらないわ。
ℚ(√2)とℚ(i)、ℤ[√2]とℤ[i]もそれぞれ同型じゃないしね。
それでこの場合、[X^2-2]と[X^2+1]はどちらも零元ではないが、
零因子になるわね。だからℚ[X]/(P) や ℤ[X]/(P) は整域ですらない。

こんな感じで疑問は解消されてくれるかしら。
ちなみに次の段の
>あなたの説明から、各 P_i に対してあるモニックの F_i ∈ ℤ[X] があって、
F_i(f) ≡ 0 (mod P_i) となることがすぐわかる
って、アタシのどの説明から、どうしてそれが言えるのかよくわからないわ。
そもそも多項式環て、多項式を一つの元としてしか見たことなくて、
多項式の変数に他の多項式を代入するなんて、
今回の問題に出会うまで考えたこともなかったわ。
ここ、もう少し丁寧な説明お願いできるかしら。
それからまた一週間くらい書き込み難しくなるかもしれないけど、
時間に余裕ができれば書き込めるかもだけど、そこんとこよろしくね。
0418陽気な名無しさん2023/11/14(火) 18:35:17.12ID:sP28vZPN0
この人の言うことをアタシなりに翻案してみると、以下の通りとなる

ℤ[X]からℤ[α]への準同型をX→αで定める
準同型定理からℤ[X]/(P)≅ℤ[α]
ℤ[α]はℤ上整なのでℤ[X]/(P)もℤ上整


疑問:
どこにℚ[X]/(P)やらℚ(α)が必要なんだ?
0419陽気な名無しさん2023/11/15(水) 09:32:43.84ID:YRgVjec30
今日はそこそこ忙しいんだけれど、
割とすぐにレスできそうだからしちゃうね。

>>418
準同型定理使うとこんなにシンプルにできるんだ?
って一瞬思ったけど、いやいやそんなに簡単な話ではないと思い、
ちょっとちゃんと読み返してみたら、あなたちょっと雑よ。
あなた長文があまり好きではなさそうだから、
できるだけシンプルに問題点を書こうと思うけど、できるかしら。
問題点を書いてる中に、あなたの疑問に対する回答も出てくる予定よ。

問題点は、ざっくり2つ。
1.Pは何?
2.ℤ[α]はℤ上整はなぜ言える?

まず1.Pは何?だけど、これまでの流れででてきたPは2種類あるのよ。
一つは元々の問題で出てきた、「整数係数のモニック」。
もう一つは「整数係数のモニックで、かつ『既約』なもの」。
この2つは決定的に重要な違いがあって、
二つ目の定義ならあなたの説明は真だけど、
一つ目の定義だと偽になってしまうわ。
αはPの任意の根を一つ選んで決めたものだと思うけど、
それだとあなたの準同型写像、全射はまあ自明と言っていいと思うけど、
Kerはαの最少多項式で生成されるイデアルよ。
もしPが『既約』ならこれは(P)で正しいけれど、
そうでないならこれは(P)ではないので、あなたの説明は成り立たないわ。

次に2.ℤ[α]はℤ上整はなぜ言える?について、
これは正しいのだけれど、自明なことではないのよ。
αがℤ上整なのは定義より自明なんだけれど、
だからといってℤ[α]のすべての元がℤ上整だというのは少し手間がかかるの。
これまでにやってきている
「代数的整数同士の和、差、積は代数的整数である」
という定理を利用することによって、
やっとℤ[α]のすべての元がℤ上整だと言えるのよ。
この定理を証明するには代数体の整数環の話になるから、
それでℚ[X]/(P)やらℚ(α)が顔を出したのよ。
Pが『既約』ならばℚ[X]/(P)はℚ(α)と同型で同一視できる。
だからℤ[X]/(P)は代数体ℚ(α)の整数環ℤ[α]と同一視できる。
といった感じにね。
まあ定理の証明自体は本にお任せで省略させてもらったけれども。

以上で回答としてはOKかしら。
これまでうさぎにいろいろレスしてきたから
この問題に関しては結構頭の中が整理されていて、
それで割とすぐ返答出来たわ。良かったわ。
0420陽気な名無しさん2023/11/15(水) 09:57:47.32ID:yEvQ+Ztj0
>>419
アンタちょっとオツムが弱いんじゃない?
0421陽気な名無しさん2023/11/15(水) 12:38:13.43ID:K+HQeVZv0
このババア、なんでアタシが間違ってるような感じを醸し出してくるのかしら
ほとんど至る所稠密に誤解と脱線が散りばめられたアンタの取り止めもない与太話から辛うじて論理の胎動が認められる箇所をより抜いて翻訳要約再構成したらこうなるだろ?って話よ?
だから、
>もしPが『既約』ならこれは(P)で正しいけれど、
>そうでないならこれは(P)ではないので、あなたの説明は成り立たないわ。
アンタにこんなこと言われる筋合い毛頭ないわ
Pを既約だと思い込んでたのはアンタでしょうよ

Pは整係数既約モニック、αは根
ℤ[X]からℤ[α]への準同型をX→αで定める
準同型定理からℤ[X]/(P)≅ℤ[α]
ℤ[α]はℤ上整(証明は成書に委ねる)なのでℤ[X]/(P)もℤ上整

アンタの言ってることってこうじゃないの?
0422陽気な名無しさん2023/11/15(水) 21:23:22.47ID:5u42BBv50
頭が良い人って使う言葉が面白いわねw
面白いというか興味深いというか
他のスレでは絶対に見られないような単語
0424陽気な名無しさん2023/11/18(土) 12:59:31.80ID:8mLhZAjn0
このスレどうしたらいいんだろうね
せっかく問題作って出してもうさぎとその金魚の糞は意味不明なスト起こしてるし
うさぎが孤独を感じないように何か話でも聞いてやるスレにでもしたらいいのかしら

うさぎは小さい頃習い事何かしてたの?
初恋はいつ?どんな人だった?
0425Usagi2023/11/19(日) 02:15:55.68ID:Fy2g79nZ0
>>416
丁寧にどうもありがとう。そしてお返事遅くてごめんなさいね。
>文章おかしかったかしら?
そうね…
>「αをPの根の一つとしたとき、ℚ[X]/(P)=ℚ(α) が体ならば、Pはモニックなので、αは代数的整数」
だと「ℚ[X]/(P)=ℚ(α) が体であること」が
「Pはモニックであること」の理由となっている、または
「(Pはモニックなので)αは代数的整数であること」の理由となっている、
のどちらかを意味しているように見えるわ。
でもおっしゃりたかったのは、そのどちらでもなく、「ℚ[X]/(P)が体であること」は
「ℤ[X]/(P) の元は整数係数のαのn-1次以下の式で表される」の理由となっている、ってことでしょう?
それから、ℚ(α) はℚにαを添加して得られる体だから、定義上、体に決まっているけれど
ℚ[X]/(P)は体とは限らないわけだから、「ℚ[X]/(P)=ℚ(α) が体ならば」という言い方もおかしいわよね。
「ℚ[X]/(P)が体ならば、ℚ[X]/(P)=ℚ(α) となるので」ならわかるけれど。

>有理数係数ならわざわざモニックって書く意味がないから
>整数係数は省略してもわかると思って省略しちゃったわ。
たしかにそう言われるとそうね。

>アタシは零元は零因子とは呼んでいないわ。
Wikipediaを調べたら、それをいう場合は「0でない零因子」または「非自明な零因子」と呼ばれる、とあるわ。

すんごい細かいことをごちゃごちゃ書いてしまって申し訳ないけれど
おそらく、あなたみたいに代数を専門的に勉強した人同士なら
こういう細かい部分を自然に脳内補完しあって話が通じるんだと思うけど
アタシは部分的に齧ってるだけでちゃんと勉強していないから
よく知らない用語をネットで調べながら字義通りに理解しようすることしかできないのよ。
それでなんか変と思っても、それが自分の理解が足りないせいなのかわからなくて混乱しちゃうのよね。

でもおかげで理解したわ。
0426Usagi2023/11/19(日) 02:17:55.99ID:Fy2g79nZ0
>>417
結局、ℤ[α]は多項式(の同値類)の集合じゃなくて数の集合ってことになるわけね?
ℤ[√2]という書き方さえするわけでしょ。
√2は文字じゃないから、これはどう考えても多項式(の同値類)の集合じゃないわよね。
ℤ[X]/(X^2-2) ≅ ℤ[√2]
というのは、つまり (√2)^2-2 と 0 が ℤ[√2] の要素として区別されないということだものね。
要するに、ℤ[α]のαは単に√2とかのある具体的な数を表しているだけで
ℤ[X]のX(こちらは多項式の関数の引数を表す)とは別物ってことね。

数学って時々こういうところがいい加減で嫌ね。
たとえば x^2 + y^2 という表現は、ある具体的なひとつの数を表しているとも読めるし
xの関数とも読めるし、yの関数とも読めるわよね。
アタシ論理学を少し齧ってるんだけど、論理学では x^2 + y^2 がある具体的なひとつの数を表すのに対し
xの関数としてなら λx. x^2 + y^2、yの関数としてなら λy. x^2 + y^2 というふうに
ラムダ項というもので書き分けるのよね。
関数型プログラミングとかされている人はよくご存知でしょうけれど。
でもふつうの数学の本でこういう書き分けしてるのって見たことないのよね。
上の例で言えば、ℤ[λX. X]とℤ[α]みたいに書きわければ混乱しないのに〜とか思ったわ。

>>あなたの説明から、各 P_i に対してあるモニックの F_i ∈ ℤ[X] があって、
>F_i(f) ≡ 0 (mod P_i) となることがすぐわかる
>って、アタシのどの説明から、どうしてそれが言えるのかよくわからないわ。
これは>>365>>354の書き換えであるということに戻っただけよ。
あなたはP(X)が既約の場合、ℤ[X]/(P(X)) ≅ ℤ[α] がℤ上整であることが
(代数的整数の集合が環となることを前提とすれば)すぐわかると言っているわけでしょう。
つまり、P(X)が既約の場合、任意の [f(X)] ∈ ℤ[X]/(P(X)) ≅ ℤ[α] について
あるモニックの F(Y) ∈ ℤ[Y] があって F([f(X)]) = (P(X))、つまり
F(f(X)) ≡ 0 (mod P(X)) であることがすぐわかるということでしょう。
アタシも多項式の変数に他の多項式を代入するなんて考えたことなかったけれど、
多項式の変数に ℤ[α] の元を代入することはふつうに考えるわけで、ℤ[X]/(P(X)) ≅ ℤ[α] なんだから、
多項式の変数に ℤ[X]/(P(X)) の元を代入しても結果的に同じことになるわよね。
0427Usagi2023/11/19(日) 02:46:11.25ID:Fy2g79nZ0
>>418
あなたの「疑問」は、修辞疑問文(反語)なんだけど、ものぐささんは本当の疑問文だと思っちゃったのよね。
でも、あなたの言いたいことはわかったし、ものぐささんの説明と合わせて理解が深まったわ。
Pが既約のとき ℚ[X]/(P) ≅ ℚ(α) となることも、X→αにより定まる体の準同型と準同型定理から来ていたわけね。
それでℤ[α]がℚ(α)の部分環だから、ℤ[X]/(P)もℚ[X]/(P)の部分環なのね。
でも確かに言われると、わざわざ体のℚ[X]/(P)とℚ(α)を介さずに、直接、環の準同型の話をすれば済むわね。
「代数的整数同士の和、差、積は代数的整数である」という話も、体を出さずに環の話だけで完結すると思うわ。
https://math.stackexchange.com/questions/2133205/set-of-algebraic-integer-form-a-ring
0428Usagi2023/11/19(日) 02:54:11.91ID:Fy2g79nZ0
>>424
ストって何? >>373を無視していることを言ってるの?
アタシは>>365の話が完結するまで次に行きたくなかっただけよ。
そもそも時間を見つけて趣味でここに書き込んでいるだけで
ここの問題を解くことはアタシの仕事でもなんでもないの。
なんでアタシが解くことが前提になってて、すぐに解かないとストだとか批判されなきゃいけないの?

>うさぎが孤独を感じないように何か話でも聞いてやるスレにでもしたらいいのかしら
>
>うさぎは小さい頃習い事何かしてたの?
>初恋はいつ?どんな人だった?
これは何よ。アタシに対する嫌味のつもり?
だったらアタシもう書き込むのやめるわ
0429陽気な名無しさん2023/11/21(火) 11:39:22.28ID:0DHQcTR10
>>421
あなたの口が悪いのはいいとして、
っていうか口が悪いからって一々気分を害していたら、
このスレではうさぎとしか会話できなくなってしまうから諦めたわ。
ご指摘の通りアタシは最初Pを既約だと勘違いしていたわ。
それでも勘違いに気づいて以降は既約かそうでないか、
その都度ごとにわかるように明記してきたつもりよ。
その流れの中で、どちらかを明記しないのはやっぱり良くないと思うわ。
それからあなたはアタシの言っていることを要約してくれたそうだけど、
とても簡潔にまとめてくれたのは本当にありがたいんだけど、
アタシの言っていたこととはちょっと違うのよね。
アタシは準同型定理を使うことは、あなたが書き込みしてくれるまで
全く頭の中になかったのよ。
ℚ(α)の整数環だからℤ[α]という考え方しかしていなかったの。
もちろんうさぎが
>Pが既約のとき ℚ[X]/(P) ≅ ℚ(α) となることも、X→αにより定まる体の準同型と準同型定理から来ていた
って言ってくれて思い出したんだけど、
そういえばℚ[X]/(P) ≅ ℚ(α) を当たり前に使っていたけど、
ここにも準同型定理使われていたのをすっかり忘れていたわ。

>>うさぎ
文章については、感覚で文章書くとおかしな文章になるのね。
「αをPの根の一つとしたとき、ℚ[X]/(P)=ℚ(α) が体ならば、Pはモニックなので、αは代数的整数」
というのは、アタシの無意識の感覚で
「αをPの根の一つとしたとき、ℚ[X]/(P)=ℚ(α) が成り立つ、すなわち体になるとする」
という、議論を始める大前提と、
「Pはモニックなので、αは代数的整数」
という議論をスタートさせた最初の部分をいっしょくたにしてしまったということみたいだわ。

零因子はもちろん厳密にいえばwikiの言う通りなんだけど、
言葉のニュアンスとして「零でないのに積が零になる不思議なもの」みたいな、
なにか普通のものではないイメージがあるので、
アタシは零因子というときは非自明なものをイメージしていたのよ。
「零因子はない」「零因子はある」というときに「零以外の」を頭につければいいんだけど、
面倒だし、「不思議なもの」のあるなしの話だから通じるわよね?と思ってしまったのよ。

ℤ[α]は、αが数なら数の集合だし、αが文字なら多項式の集合よ。
論理学の書き分け方については初めて聞いたわ。なるほどね。

>各 P_i に対してあるモニックの F_i ∈ ℤ[X] があって、F_i(f) ≡ 0 (mod P_i) となることがすぐわかる
について、納得したわ。
Pが既約でなくても>>415 の論法でOKっぽいわね。
0430陽気な名無しさん2023/11/21(火) 11:40:53.19ID:0DHQcTR10
>>427
「代数的整数同士の和、差、積は代数的整数である」という話が、
体を出さずに環の話だけで完結するのはそのとおりなのよ。
だけどアタシが勉強した時に、環論やって体論やって、
それでガロア理論やって、って順番で勉強して、具体例として代数体が出てきて、
その後に代数的整数いろいろやり始めたから、
代数的整数といえばある代数体の整数環の元、ってイメージがアタシにとっては強いのよ。
だから代数的整数の説明をするときに、つい代数体経由で説明してしまうのよね。
代数体経由で説明するのは、そういう順番で勉強したアタシ流の説明であって、
体を使わずに説明する人がいてもそれを否定するつもりは全くないわ。
その方が簡潔だからその方が優れた説明だと言われればその通りかも知れないし。

>>428
>>424 は完全にスルーすべき書き込みだと思うわよ。
このスレは口が悪い人だけではなく根っから意地悪な人もいるから、
いちいち相手にする必要ないわよ。
そもそもこのスレで問題解いているのは大部分がうさぎだし、
残りの少しアタシも以前解いたこともあったけど、
それよりもっと昔解いてくれていたリサ姐さん達も今はいないみたいだし、
このスレが辛うじて良スレだと思えるのはうさぎのおかげよ。
問題出すより解く方が大変なのに、
出しておいて解答解説しないのも無責任な出題だと思うから、
基本的に問題の出し逃げみたいな問題はスルーして構わないと思うわよ。
もちろん解けそうなら解いてもいいと思うけど。
0431陽気な名無しさん2023/11/21(火) 20:10:10.20ID:HfcHPnuV0
それは違うんじゃないかしら

そりゃ>>150みたいなつまらん問題出すだけなら簡単だろうけど、この問題や>>321のように議論が盛り上がり、そして参加した人が皆それぞれに勉強になるような問題を出すのは、実は難しいことだと思うわ
入試問題を見て一般化に挑戦したり、代数学の教科書の演習問題をできるだけ易しい方法で解けないか試行錯誤したり、そういう地道な努力があってはじめてこのような出題が可能となる気がするの
出題する側にこそ学力が必要なのよ

また、出題の仕方にも丹精しているのでは、と気付かされたわ
スレに居る人の特徴をよく観察して、その人たちに興味を持ってもらえるように問題を表現する、そうすることで活発な議論が巻き起こり、スレが活性化するの
例えば、うさぎの神経逆撫でするようなスパイスの効いた問題文も天晴れよね
うさぎがペダントリイに堕した性格なのを知り抜いてるからこそ、あえてうさぎが突っ込みたくなる問題文にして、うさぎの学識誇示欲を刺激し、ここを余スレをもって代えがたい豊穣なスレにしている
洞察力の賜物よ

>>150>>225みたいなコクものどごしもない、なんの味わい深さあったもんじゃないゴミを並べていれば、いずれスレは死ぬわ
このスレを辛うじて良スレならしめているのは、なんとかして質の高い問題を出そうと努力してくれている人のおかげなのよ
少しでも想像力があれば問題出すより解く方が大変なんて口が腐っても言えないはずよ
0432陽気な名無しさん2023/11/21(火) 21:06:00.54ID:0DHQcTR10
>>431
それならある程度盛り上がったら、出題者から想定していた解法や、
盛り上がりに対する何らかのコメント等があってもいいはずよね。
偶然盛り上がったのではなく、努力して盛り上がるような問題を作ったのなら、
ぜひその盛り上がりのまとめのネタばらしまで出題者自身がやるべきだと思うわ。
問題の出し逃げはダメよ。

その前にそもそもスレタイは完全無視なのね。
>>150>>225みたいな問題こそ、正にスレタイそのものなのにゴミ呼ばわりですものね。

そもそもあなたの言う「盛り上がり」って、うさぎを含む、
大学数学を少しでもかじった人でないとついていけないのよ。
このスレは大学数学はやっていないけど大学受験までは理系だった、
って人が本来の対象のスレだったはずよ。
>代数学の教科書の演習問題をできるだけ易しい方法で解けないか
これは完全にスレ違いの発想だと思うわ。

うさぎが頑張るからスレ違いの発想が暴走してしまっているけど、
数年前はもっとたくさんの人が解けるような問題を出して、
もっとたくさんの人が解いていたと記憶しているわ。
それこそがこのスレの本来の姿だと思うわ。
そのためにも>>150>>225みたいな問題がいいんだと思うわ。
他にも例えば、中学入試の問題だけど、
大学入試を理系でやった人でもちょっと戸惑うような問題なんかもいいかもしれない。
話の流れで大学数学の話にまで話が膨らむのはまだいいとしても、
最初から大学数学レベル狙いで出題するのはやめてほしいわ。
0433Usagi2023/11/22(水) 22:39:47.86ID:u4RMVQ+J0
>>429
零因子という用語についてもうちょっと調べてみたの。
零因子に0を含めるWikipediaの定義は、ブルバキによるものみたいだわ。
零因子の英語はzero divisorだから「零を割るもの」「零の約数」って意味なのね。
そして、0でない零因子は nonzero zero divisor、零因子でない元は non-zero-divisor、と非常に紛らわしいわ。

で、本棚の肥やしになってた MacLane & Birkhoff (AMS) を調べてみたら、0を含まないように零因子が定義されていたわ。
気になったから、とあるサイトでいろいろな代数の本(Lang, Hungerford, Dummit & Foote, Herstein)を
ダウンロードして調べたら、すべて0を除外する定義だったわ!
だからきっとブルバキ原理主義者以外には、あなたの用語の使い方の方が標準的なんだと思うわ。

別の本棚の肥やし Michael Artin著 “Algebra” 第1版 (1991) はブルバキと同じで0を含めていて、
そこに興味深いコメント(これもおそらくブルバキのぱくり)があったわ。

The term “zero divisor” is traditional, but it has been poorly chosen, because actually every a∈R divides zero: 0 = a0.
(zero divisor という用語は伝統的だが、お粗末に選ばれた用語だ。なぜなら、どの a∈R も0を割るからだ。)

でもネットでArtinの第2版 (2010) をダウンロードししたら、0を除外する定義に変わってて、上のコメントも無くなってたの!
確かにいつも nonzero zero divisor とか早口言葉みたいなもの言ってられないものね。
同じ数学用語でも、本や人によって意味が微妙に変わる場合もあるという例ね。

354 = 365 の問題は、二通りの全然違う解き方ができて
ものぐささんのおかげでアタシは環論にも少し触れることができて勉強になって良かったわ。
だけど、365の人は >>366 ですでにひとつの答えを出していて、これも気になってたのよね。
https://ja.wikipedia.org/wiki/有限生成加群
を見ると、
・Aを有限生成R加群である
・AはR上有限生成環かつRの整拡大である
のふたつが同値である、と書かれてて、366のリンク先はこれを利用しているのよね。
で、Wikipediaで引用されている Kaplansky をダウンロードしてちょっと見てみたの。
おそらくだけど、ℤ[X]/(P) がℤ上整である証明は、ℤ[α]がℤ上整であることの証明と同様にできるっぽいわ。
ℤ[α]がℤ上整であることを示すには、1, α, …, α^{n-1} が基底となる(有限次元になる)ことを使うわけで
上の議論では、ℤ[X]/(P(X)) ≅ ℤ[α] であることを言うために、Pが既約であることを要求したのよね。
でもℤ[X]/(P) の元はすべて n-1次以下の多項式と同一視できるから、1, x, …, x^{n-1} がℤ[X]/(P)の基底となって
あとはℤ[α]のときの証明と同様にできるっぽいわ。この場合、Pが既約かどうかは関係ないのよ。
0434Usagi2023/11/22(水) 22:53:20.61ID:u4RMVQ+J0
>>431
あんたの言葉使いのセンスは面白いわね。そこは認めるわw
で、褒められてるんだが貶されているんだかわからないけれど

>うさぎの神経逆撫でするようなスパイスの効いた問題文も天晴れよね
>あえてうさぎが突っ込みたくなる問題文にして

これはおかしいわ。深読みしすぎだし捻くれてて倫理観が崩壊しているわ。
アタシは、>>321とか>>354みたいなのは、単純に、
数学板ではないここでは大多数の人に理解できない書き方だからやめてほしいと言っているだけなの。
こういう書き方への苦情は、アタシとものぐささん以外からはほとんどないように見えるかもしれないけれど
それは、理解できない人が書き込みにくい雰囲気にこのスレがなってしまったせいだと思うわ。
こういう書き方のおかげでアタシの書き込み意欲が増すということは全くないの。
むしろ正反対よ。
本当は無視したいんだけど、そしたら問題の意味がわからない人がわからないままになるから
ふつうの日本語に翻訳してあげなきゃ、と思って書き込んでいるところもあるわ。
でも、こういう書き込みをやめてほしい、という怒りの説明文を書くのもすごく大変でストレスを感じるし
日本語への翻訳も二度手間でアタシが疲れるだけだし、本当に嫌なの。
だけど、苦情を書くだけで解かないと
「逃げた」とか「解けないくせに」とかコメントが来てまた嫌な思いをしそうだし
いずれにせよアタシが解かないと誰も解きそうもないし、そしたらスレが廃れて落ちるだけだし
とか思って、スレ民の信用を得る目的もあって解いているのよ。
0435Usagi2023/11/22(水) 22:53:47.80ID:u4RMVQ+J0
>>321も相手にしないつもりだったけれど、>>324で日本語を話せる人だとわかったから、相手してあげることにしたのよ。
>>365も流そうと思ってたけれど、>>371でものぐささんが書き込んでくれたから、アタシはそれに反応しただけなの。
>>354の問題だって、書き込まれてすぐにアタシは>>359の解き方を思いついたわ。
でも一週間も経ってから書き込んだのは、相手にするのが嫌だったし>>358みたいなものを書くのが嫌だったからよ。
だからくれぐれも勘違いしないでほしいわ。
最初から誰にでもわかるような日本語で書かれていたら、アタシはいろいろ嫌な思いをしないで済んで
純粋な気持ちで取り組めて、書き込み意欲もずっと高くなるわよ。
上にも書いたけど、今後はもうこういうのは相手にしないわ。

洞察力の賜物、ってアタシは実験観察対象の動物扱いなの?
うさぎはモルモットじゃないの!
アタシは、321や354が、故意にアタシをイラつかせようとしてそういう書き方をしたとは考えたくないわ。
その考えは捻くれすぎよ。
それに、故意に特定の個人に嫌な思いをさせても、自分が楽しめる or 全体の利益となるなら良いと考えて、
そういうやり方(実際にはしていないと思うけど)を賞賛するなんて倫理観が崩壊しているわ。

疲れた…
0436陽気な名無しさん2023/11/23(木) 17:36:38.94ID:GRjP6KSQ0
>>433
あらまあわざわざ調べてくれたのね、ありがとう。
ブルバキだったのね、なるほど納得だわ。
ブルバキといえば自然数の定義にも0を入れてたわよね。
確かに自然数は0を入れないのは加法に関する半群で、
入れたのは単位的半群になるから
入れたほうが数学的により美しいのは確かにわかるわ。
でも、自然数って、人間が一番最初に素朴に持った数の概念でしょ。
だったら0は入れないで1からとすべきよね。
だって0は1,2,3,・・・の概念が出来てから
かな〜り経ってから発明?発見?されたものでしょ。
歴史的にはやっぱり自然数に0は入れるべきではないと思うわ。

有限生成加群についてなんだけど、
アタシ何度か言っているように、群、環、体ガロの順に勉強して、
加群は飛ばしちゃったから、ちょっと苦手意識があるのよね。
でもせっかくうさぎがいろいろ調べてくれて、
なんかわかりそうな気になったから、アタシも調べてみたわ。
wikiの整拡大のところを見ると(https://ja.wikipedia.org/wiki/整拡大)
B を環とし、A をその部分環とする。このとき B の元 b について次は同値。
b は A 上整
部分環 A[b] ⊂ B は A-加群として有限生成
A[b] は有限生成 A-加群である部分環 C ⊂ B に含まれる
(以下略)
なんてのもあったわ。
まあうさぎが調べてくれたのと同じような内容で、
加群メインで書いてあるのと、環メインでかいてあるのが違うくらいかしら。
あと他に、https://detail.chiebukuro.yahoo.co.jp/qa/question_detail/q12239745397
こんなところに「Bの元xはA上整 ⇔ A[x] はA上整」の証明があったわ。
この証明がOKならば、ℤ[X]/(P) の元XはモニックであるPの解なんだから、
既約かどうか関係なくB=ℤ[X]/(P) とみなせばXはℤ上整であり、B=A[x]となって、
証明したかったことが完全に証明されてるのではないかしら。
この証明、行列式=0とか使っていて、代数的整数同士の和、差、積が
代数的整数になることの証明と似てるわよね。
その証明(二人目の補題2)のチェックは面倒だから
というか、忘れてる線型代数見直さないと、tって何?状態なので、
チェックはお任せしてよろしいかしら?
0437Usagi2023/11/23(木) 18:19:14.32ID:YgW66qYv0
まだ詳しく読んでいないけれど、^t は転置を表しているものだと思うわよ
(a_1,...,a_n) は本当は縦ベクトルなんだけど、ネットだとそのまま書けないから
横ベクトルで書いて、その前に ^t をつけることで本当は縦ベクトルって言いたいだけだと思うわ
0438Usagi2023/11/24(金) 12:48:55.32ID:q2UzmVkH0
知恵袋読んだわ。ていうか知恵袋にこんなものまであるなんてビックリね!

これって、FullHouseさんが「整閉包が環をなす」ことを用いて問題に答えたから
それを受けて、りっきぃさんが「整閉包が環をなす」ことを証明したって流れなのかしらね?
でも、実はりっきぃさんみたい書いてくれるなら、この【定理】まで言わなくても
【補題2】まででこの問題に答えるのには十分よね?
環RとしてAを、そして環SとしてA[x]をとればいいから。

で、あたしも余因子行列とか詳しくないから調べたけど、要するに
Mを正方行列、単位行列をEとすると、(Mの余因子行列) × M = (det M) E となるらしいわね。
でもこんなややこしいやり方をしなくても、草場本の証明を参考にすると
零ベクトルでない縦ベクトル (a_1, …, a_n) に行列 aE-A を作用させたら零ベクトルになるのだから
det(aE-A) = 0 ってすぐ言えるんじゃないのかしら?

あなたのおっしゃる通りで、これって本質的に
「代数的整数同士の和、差、積が代数的整数になる」つまり「代数的整数の集合が環をなす」
ことの証明と同じだと思うわ。
https://math.stackexchange.com/questions/2133205/set-of-algebraic-integer-form-a-ring
上にも出したリンクだけれど、行列の具体例が見やすいから参考になると思うわ。
これも最後の部分でまた違うやり方をしていて、アタシも線型代数詳しくないからよくわかってないけど
行列Mの固有多項式をχとするとχ(M) = O(零行列)となる、というのはケイリー・ハミルトンの定理よね。
で、この場合、MB = pB から M^i B = p^i B なので、零ベクトル = OB = χ(M)B =χ(p)B となるから
χ(p) = 0 がわかるってことかしら?
ともかくχ(p) = 0 なら det(pE-M) = 0 となるわね。

高木本はどういうやり方なのかしら?

一般的に、α, βを環R上整、そしてαとβのR上の最小多項式の次数をそれぞれ n, m とすると
R[α, β]は有限集合 { α^i β^k | 0 ≤ i < n, 0 ≤ j < m } を生成集合とするR上の加群となるわけよね?
それで、知恵袋の【補題2】の {a_1, …, a_n} を { α^i β^j | 0 ≤ i < n, 0 ≤ j < m } にすれば直接【定理】が証明されるわ。
草場本や上のリンクはそういう流れよね。
細かいことを書くと草場本では { α^i β^k | 0 ≤ i < n, 0 ≤ j < m } を直接使うんじゃなくて
高々有限次元だからその基底を u_1, …, u_l として、それらが作るベクトルに行列をかけているんだけど
もし { α^i β^j | 0 ≤ i < n, 0 ≤ j < m } が基底になっていない、
つまり一次独立でなかったとしても特に議論に差し障りはないわよね?
{ α^i β^j | 0 ≤ i < n, 0 ≤ j < m } の元がすべて 0 なわけでないから問題ないと思うんだけど…

話が戻るけど、上にも書いたように R[x]が環となることを言うには、整閉包の話まで一般化する必要はなくて
【補題2】の {a_1, …, a_n} を { 1, x, …, x^{n-1} } とすればもうそれで十分だと思うわ。
0439Usagi2023/11/24(金) 13:27:32.51ID:q2UzmVkH0
>>436
>ブルバキといえば自然数の定義にも0を入れてたわよね。
それは知らなかったけれど、自然数に0を入れるのは数学基礎論(論理学や集合論)では一般的よ。
数学のすべてを集合で表現しようって考えだから、自然数とかの数もすべて集合で定義するの。
自然数の集合論定義はいくつかあるらしいけど、一番有名なのはフォン・ノイマンによるもので
0 := ∅
n+1 := n ∪ {n}
と帰納的に定義するの。すると
1 = {∅}
2 = {∅, {∅}}
3 = {∅, {∅}, {∅, {∅}}}
4 = {∅, {∅}, {∅, {∅}}, {∅, {∅}, {∅, {∅}}}}
てな感じになるわ。
すると、|n| = n (集合nの要素の個数はn個)となるのよ。
あと n < m ⇔ n ⊊ m ⇔ n ∈ m なんて関係も成り立って便利なのよね。
もし自然数を1から始めたら、自然数の集合論的定義がきわめて不自然なものになってしまって
理論全体が大変なことになってしまうと思うわ。

0の発見っていうのも、10進法とかで数を表記する時に空の位に書くものを発明した、という意味じゃないかしら?
0がないと、数字が無限に必要になってしまうから。
数としての0の概念自体は、人間の中に普通にあるもののような気もするわ。
実際、正の自然数と同様、0は自然言語で表現可能なことじゃないかしら。

うさぎはいちごをひとつ食べた。
うさぎはいちごを(まったく)食べなかった。

そこへいくと、負の数は自然言語では表現しようがないわ。
0440陽気な名無しさん2023/12/03(日) 11:49:19.54ID:kFj7X8sB0
>>うさぎ
お待たせしたわね。
最近忙しくて、今日やっと時間が取れたわ。

>>437
そうね、^t は確かに転置ね。
それですっと意味が通ったわ。

>>438
そうそう、補題2までで十分よ。
det(aE-A) = 0 がすぐ言えて、だからaは
モニック多項式 det(xE-A)∈ R[x] の根、でいいわよね?
「代数的整数同士の和、差、積が代数的整数になる」の高木本の証明は、
見た感じあなたの書いた草場本と同じような感じみたいね。
{ α^i β^k | 0 ≤ i < n, 0 ≤ j < m } や u_1, …, u_l のことを、
基底なんて一言も書いていなくて、
基底かどうか関係なく、式変形を追って行けば導かれるっぽいわよ。
0441陽気な名無しさん2023/12/03(日) 11:50:00.87ID:kFj7X8sB0
>>439
自然数については、wikiに説明があるわ。(https://ja.wikipedia.org/wiki/自然数)
これ0入れる考え方も入れない考え方もちゃんと説明されてて結構いいわ。
アタシは自然数の形式的定義はペアノの公理でやったから
0は入れない1からの定義が自然だと感じるし、wikiにもあるように
歴史的にも1〜は古代からあったけど0の概念は628年だそうよ。
集合論、論理学で主流らしい0を入れる定義も載っていて、
それはあなたが書いたのと同じようなものだけど、
それは後世になっていろんな数の概念が出来てから定型化したもので、
歴史的なことは関係のない、理論的整合性に重きを置いた定義だと思うわ。
理論的整合性で考えれば0を入れるのが当然ですもの。
もちろんペアノの公理も後世になって作られたものだけど、
こちらはきっと自然数の概念は歴史的にこのように発生したのでは、
ということに重きを置いたものだと思われるわ。
うさぎの言う、自然言語で表現可能かどうか、という考え方って、
後世になっていろんな数の概念が出来てから考えられたものであり、
628年より前は「無い」という概念はあっただろうけど、
数の概念と結びついていなかったものと思われるわ。

本当かどうか知らないけど、アタシのきいた話では、
原始時代数の概念が最初にできた時は、「1,2,3,たくさん」
だったとかいう話よ。4以上は認識されなかった時代の話ね。
勿論この時代に0の概念なんてなかったと思われるわ。

それで、0を入れたものを非負整数と呼んだり、
0を入れないものを正整数と呼んだりするけど、
自然数という言葉は、自然発生的に、
歴史的に最初に認識された数、と解釈するのが自然だと思うから、
アタシはやっぱり自然数には0は入れない方を支持するわ。
実際使うときは自然数と呼ばずに、必要に応じて
正整数とか非負整数と呼べばいいのよ。
もちろん自然数をどっちと考えるかは人それぞれだから、
学問的に議論することではないと思うわ。
0442Usagi2023/12/07(木) 22:27:01.84ID:pxoB+5/s0
>>441
Wiki読んだわ。いろいろ複雑ね。
よく読むと、10進法とかの空の位を表す数字としての0は1世紀頃からあったけれど
数としての0は628年からって書いているわね。
つまり0という数「字」はあっても、それを単独で数として使うと言う発想がなかったってことなのかしら。
つまり 1 + 0 = 1 みたいな計算は考えなかったって意味かしらね。

他にも古代ギリシアでは数は2からで1も数じゃなかったとか書いてあって面白いわね。

ペアノの公理についてもアタシは論理学の本で読んだから0から始まるものと思ってたわ。
https://ja.wikipedia.org/wiki/ペアノの公理
これ↑見ると、実際0も入っているんだけど
「歴史」のところを読むと、ペアノ自身が考えたときは1から始めてたみたいね。

アタシ思うんだけど、自然数は0からか1からか、はたまた古代ギリシア風に2からか、とかって
最初にどう習うかの影響が大きい気がするのよね。
中学高校で自然数は1からって刷り込まれるから、そのイメージを壊すのが難しいだけの気がするのよね。
実際アタシも最初に自然数が0から始まっている本を読んだ時すごく抵抗あったんだけど
なんで抵抗あるのか考えてみたら、中学高校でそう習ったって以外に理由が見つからなかったの。
結局、数学は理論的に一番綺麗であるべきだと思うから、今はアタシの中では自然数は0からになってるのw

自然言語で表現できるかというアタシの考察はちょっと適当過ぎたかもしれないわ。
実際「ひとつ」「ふたつ」「みっつ」と、1以上の数を表す単語は昔からあるのに
0個を表す単語についてはそうではないものね。
確かに「何もない」ことを表現できることと、数としての0の概念をもっていることは別のことかもしれないわね。

>原始時代数の概念が最初にできた時は、「1,2,3,たくさん」
>だったとかいう話よ。4以上は認識されなかった時代の話ね。
>勿論この時代に0の概念なんてなかったと思われるわ。

原始時代と言わず、現代でもブラジルのアマゾンに住んでいる「ピダハン族」の言語では
「少ない」「多い」のような表現はあるけれど、数を表す言葉がないらしいの。
そして、この部族の人に数をマッチングさせる課題をやらせると
要素の数が4以上になるとパフォーマンスが低下するらしいわ。
これを説明している面白い動画があったから貼っておくわね。
https://www.youtube.com/watch?v=Xv-3HB8b1as
動画によると、数を表す言葉を覚える前の赤ちゃんも4以上の数を把握できないらしいわ。
でもこれを見て思ったのだけれど、おそらく赤ちゃんに
1 - 1 = 0 を実演しても驚かないと思うし、2 - 1 = 0 をトリックで実演したら驚きそうな気がするのよ。
少なくともピダハン族の大人はそういう反応をすると予想するわ。
とすると、こういった実験の結果を動画が説明するように
数の概念を理解しているかどうかとして解釈できるならば
0は誰もが概念として持っていると言えるんじゃないかしらとも思ったのよ。
もしこの考え方で「自然」数を考えるなら、(0,) 1, 2, 3 だけが自然数になりそうね。

取り留めのないことつらつらと書いたけれど、一言で 0 と言ってもいろいろな理解の仕方があるわけね。
数「字」としてなのか数(概念)としてなのか、
概念といっても、言葉で表せるという意味か、引き算の実演で間違いに驚くと言う意味か、
はたまた加法の単位元としてなのか、
結論も何もなくてごめんなさいねw
0443Usagi2023/12/08(金) 00:03:51.05ID:yeo6heGD0
ちなみに、この「ピダハン」という呼称は誤りなの。
これはポルトガル語で Pirahã だから「ピラハン」の方が正しいわ。
(ã は鼻母音だから最後の音節は正確には異なるが「ハン」に近い)
この言語の研究は、ほぼすべてがダニエル・エヴェレットというアメリカ人の現地調査に基づくもので
この人が一般向けに書いた本が10年程前に日本語に翻訳されたのよ。
(『ピダハン「言語本能」を超える文化と世界観』)
その際に(失礼ながら)無能な翻訳者が「ピダハン」としたために、この呼称で知られるようになってしまったの。
どうしてこうなったかと言うと、エヴェレットの原著では
アメリカ人読者に Pirahã の発音の仕方が分かるように pee-da-HAN と表記してあるの。
ポルトガル語の Pirahã の r は日本語のラ行と同じで、舌先で歯茎をはじく「はじき音」という子音で発音されるの。
でも英語の r は、はじき音ではないわよね。
そのかわり、アメリカ英語ではアクセントのない音節の頭の d がはじき音に変化するから、
アメリカ英語の話者が pee-da-HAN を読むと、自然にこの da はちょうどブラジル人の ra = 日本人の「ラ」の発音になるのよ。
つまり、エヴェレットはあくまでアメリカ英語話者が読むとポルトガル語の発音に似るように書いただけなの。
ところが、この本の翻訳者はそんな基本的なことも理解できなかったので
pee-da-HAN をそのまま日本語のカタカナにして「ピダハン」としてしまったの。
この本はかなり話題になったから、日本では完全に「ピダハン」で定着してしまったわ。
アタシ、こういうのってかなり罪深いと思うのよ。
こういうふうに誤りが世の中に広がっていくのね、って思ったわ。
日本でまだよく知られていないことの翻訳を出す時って、それなりの責任が伴うと思うの。
きちんとできない人がやると、誤りを広めたりして迷惑だとアタシは思ってしまうの。
ていうか、アタシ気になったから、今回この翻訳書を図書館で借りてきたの。
そしたら訳者あとがきにこう↓書かれていたわ。

 ちなみに、「Pirahã」については「ピラハ」「ピラァ」といった表記を見るが、本書の説明によると、
 あえてカタカナに直すとすれば「ピーダハーン(ハーンの部分に強勢がくる)」のような音になるらしく、
 迷った末に「ピダハン」と表記することに落ち着いた。カタカナ表記については日本でも研究が進み、
 適確な表記法が工夫されることを切に願う。

本当に無能で無責任で驚き呆れるわ。
Pirahãというのはブラジル人がこの部族を呼ぶ時の名称だから単純にポルトガル語の発音の問題であって
Pirahã語の発音の問題じゃないから、研究の余地もクソもないのよね。
Pirahãの人たち自身は自分たちのことを Hiáitihí と呼んで、自分たちの言語を Apáitisí と呼ぶそうよ。

別の話だけど、アタシだいぶ前に Steve Awodey の Category Theory を
少数人数の読書会で読んだことがあって(結局最初の1章くらいしかできなかったけど…)
アタシは英語読めるから原著を読んでたけど、他の人は日本語の翻訳書を使ってたの。
それを見せてもらったら誤訳だらけで意味不明だったの!
例えば the と書いたら「唯一の」と言いたいわけだけど、「まさにその」だか何だか意味不明な訳になってたり
とにかく翻訳者が明らかに全然英語を読めていなくて、数学の内容を理解しているのかも疑問に思ったわ。
特に当時は日本語での圏論の情報がすごく少なかったのに
せっかく見つけた本がデタラメ翻訳書だったら笑えないし本当に迷惑よね、と思ったわ。
0444Usagi2023/12/09(土) 03:23:39.15ID:OSxWAmwv0
最後にスレの趣旨に合わせて>>354の問題を大学入試問題風にしたら例えばこんな感じかしら

a, b, c, d, e は整数で、d^2 < 4e とする。
この時、ある整数 p, q があって、関数 f(x) = x^2+px+q が
「すべての自然数nに対して f(an^2+bn+c)/(n^2+dn+e) が整数である」
という条件を満たすことを示せ。

なんだかすごく難しそうね
ほとんどの受験生に解けなさそう…

ちなみにfの中身を1次式にして
f(an+b)/(n^2+dn+e) の形の問題だと (nの2次式)/(nの2次式) だから
これがいつも整数ということは = a^2 ってばれちゃうから割と解けそう
0445Usagi2023/12/09(土) 03:47:48.18ID:OSxWAmwv0
だいぶ遅くなったけど>>373解いてみるわ
(1) 0 < arg z < π/4 なので 0 < arg z^2 < π/2 つまり z^2 ∈ { a + bi | a > 0, b > 0 }
したがって w^2 = 1 - z^2 ∈ { 1 - (a + bi) | a > 0, b > 0 } = { c + di | c < 1, d < 0 }
w = x + yi と表すと w^2 = x^2 - y^2 + 2xyi なので
x^2 - y^2 < 1 であり、さらに 2xy < 0 つまり (x < 0 かつ y > 0) または (x > 0 かつ y < 0)
を満たす点の集合でいいかしら。

(2) w = x + yi とすると z^2 = 1 - w^2 = (1-x^2+y^2) - 2xyi
|z| = 1 ⇔ |z^2| = 1 ⇔ |(1-x^2+y^2) - 2xyi| = 1 ⇔ (1-x^2+y^2)^2 + (2xy)^2 = 1
これを整理すると
x^4 + 2x^2y^2 + y^4 - 2x^2 + 2y^2 = 0
となるんだけど、これを図示するのって難しいわよね

だからおそらく w = r(cosθ + i sinθ) と極形式を使うのよね
すると w^2 = r^2(cos2θ + i sin2θ) だから
|z^2| = 1 ⇔ |1 - w^2| = 1 ⇔ (1 - r^2 cos2θ)^2 + (r^2 sin2θ)^2 = 1
これを整理すると r^2(r^2 - 2cos 2θ) = 0
したがって r = 0 または (r ≠ 0 かつ r^2 = 2cos2θ) となるわ
では r ≠ 0 の場合を考えることにするわ
r^2 = 2cos2θから 0 < r^2 ≤ 2 ね
そして cos 2θ > 0 なので
0 ≤ θ < π/4 または 3π/4 < θ < 5π/4 または 7π/4 < θ < 2π ね
さて、x = r cosθ だから
r^4 = 2r^2 cos2θ = 2r^2(2cos^2θ -1) = 4x^2 - 2r^2
したがって x^2 = (1/4)r^4 + (1/2)r^2 で
これは r^2 = 2 のとき最大値 2 をとるわ
xの最大値は √2 で、r = √2から、y = 0、つまり w = √2, θ = 0 の時
(xの最小値は -√2 で、r = √2から、y = 0、つまり w = -√2, θ = π の時)
また y = r sinθ だから
r^4 = 2r^2 cos2θ = 2r^2(1 - 2sin^2) = 2r^2 - 4y^2
したがって y^2 = -(1/4)r^4 + (1/2)r^2 = -(1/4)(r^2 - 1)^2 + 1/4 で
これは r^2 = 1 のとき最大値 1/4 をとるわ
yの最大値は 1/2 で、r = 1から、x = ±√{1^2-(1/2)^2} = ±√3/2
つまり w = √3/2 + (1/2)i, θ = π/6 または w = -√3/2 + (1/2)i, θ = 5π/6 の時
(yの最小値は -1/2 で、r = 1から、x = ±√{1^2-(-1/2)^2} = ±√3/2
 つまり w = √3/2 - (1/2)i, θ = 11π/6 または w = -√3/2 - (1/2)i, θ = 7π/6 の時)

偏角が 0 ≤ θ < π/4, 3π/4 < θ < 5π/4, 7π/4 < θ < 2π の範囲で動いて
θがπ/4, 3π/4, 5π/4, 7π/4に近づく時は r^2 = 2cos2θだから r は 0 に近づく
r = 0 で原点を通る
あと上で調べた最大値最小値のポイントを通るように描いていくと ∞ みたいになるかしら
こんな適当な解き方でいいのかしら?

微分とかするのかしら、とも思ったけど
x^4 + 2x^2y^2 + y^4 - 2x^2 + 2y^2 = 0
みたいな陰関数っていうの?を微分する方法アタシ分からないわ
と思ってWiki見てたら、2変数のものは陰伏曲線というらしくて、この問題の例が出てたわ!
カッシーニの卵形線(これはその特殊な場合でレムニスケート)ていうらしいわね
ところでWikiに陰伏の読み方が「いんふく」って書いてあるんだけど「いんぷく」よね??
0446陽気な名無しさん2023/12/13(水) 17:51:38.10ID:V9XS6v6M0
>>442
見直したら、アタシがやったテキストにあるペアノの公理も、0から始まっていたわ。
でもよくわからない、というかこの著者何を考えているのかしら、と思うのが、
同じ著者、同じシリーズのテキストで、一方は自然数に0を入れていて、
もう一方は自然数に0を入れていないのよ。

具体的に言うわ。松坂和夫という先生の、岩波書店のシリーズで、
集合・位相入門という本では、自然数には0は入れていないの。
P.3に、「自然数全体の集合ℕは {1,2,3,4,・・・,n,・・・} と表わされる.」とあるわ。
でも同じ松坂和夫先生の、岩波書店のシリーズの、代数系入門には
やっぱりP.3に、「正の整数と0とを合わせて自然数とよび、」とあって、
巻末にペアノの公理の説明があって、そこでも0が入っているの。
松坂先生にとっての自然数の解釈はどっちなのかしら?
それに、wikiの自然数の項には
>19世紀、自然数の集合論的な定義がなされた。この定義によれば零を自然数に含める方がより便利である。
>集合論、論理学などの分野ではこの流儀に従うことが多い一方、
>数論などの分野では 0 を自然数には含めない流儀が好まれることが多い。
とあるから、異なる定義をするなら逆じゃないの?
集合・位相入門の方0入れて、代数系入門の方0入れないべきじゃないのじゃないの?
松坂先生が何を考えているのかわからないわ。

ピダハン族の話は面白いわね。動画も拝見したわ。
数の広がった順は、
@まず3まで
A指を見て5,10,20まで
B指の本数を一つのまとまりとしてさらに大きな自然数まで
AかBくらいまでは原始時代にすでにあったものだと思うわ。
その後628年にインドで
C0の発明
といった感じではないかしら。
自然数に対するアタシの感覚は、この順のどこまでの過程を自然数に含めるか、
という感覚で考えていることが改めて分かった気がするわ。
原始時代にすでにあったものを「自然」数と呼ぶのが妥当だと思うから、
アタシは自然数に0を入れない派なんだわ。
あなたのトリック実演の話では、「無い」ということに関して驚くか驚かないか、
というこに関する話だと思うけど、
「無い」という概念があることと、「無い」ことを「0」という数の概念として持っていることは、
別ではないかしら。

まあアタシとしては、個人的には自然数には0は含めないけど、含めると考える人もいるし、
どちらが正しくてどちらが間違っているということではないと思うわ。
コミュニケーションや著作等においては、0を含めようが含めまいが全く支障ないならば、
特に自然数に0を含めるかどうか前置きしなくても構わないけど、
必要がある場合は、どちらの立場で自然数という言葉を使うかを前置きする、
又は自然数という言葉を使わず、非負整数、正整数などの言葉を使えばいいと思うわ。
0447陽気な名無しさん2023/12/13(水) 17:52:04.57ID:V9XS6v6M0
>>443
このほとんどはアタシは個人的にあまり興味がないわ。
ただ最後のところ、数学書の原著と日本語訳についてはアタシも覚えがあるわ。
個人的に英語のテキスト使っていた時があって、でもアタシ英語苦手だから苦労して、
ふと日本語訳版の本があることに気づいて入手したの。
でもその時は「数学の勉強=英語本の翻訳作業」みたいになっていたから、
日本語版の本は自分で訳したのとの比較参考に使っていたの。
そうしたら日本語版も、英語版も、それぞれにめちゃくちゃなところがたくさんあって笑ったわ。
でも突き合せたおかげで、著者が言いたいことがわかったわ。一冊だけならわからなかったかもしれない。
著者はフランス人だから多分原著はフランス語なんでしょ。
それで英語版に訳した時も誤訳やミスプリがいっぱいあって、
日本語版はたぶんフランス語版の原著から訳したのね(英語版と全然違ったりしてたから)、
それも誤訳やミスプリがあって、それで突き合わせるとめちゃくちゃになっていたんだと思うわ。
最近の翻訳ではそういうことはたぶん少なくなってきてはいると思うけど、
それでもやっぱり数学の勉強は出来れば原著で、
それが出来なければ複数の翻訳版を突き合せないと結構信用できなかったりするわ。
0448陽気な名無しさん2023/12/13(水) 17:56:09.48ID:V9XS6v6M0
>>445
>>373 のリンク先、もう見れなくなっているわ。
どんな問題だったかしら。
さほど難しくない問題だったような記憶がおぼろげに、状態だわ。
0449Usagi2023/12/14(木) 20:38:00.49ID:rfcoGIyP0
>>446
>同じ著者、同じシリーズのテキストで、一方は自然数に0を入れていて、
>もう一方は自然数に0を入れていないのよ。
あら、それは不思議ね。けっこうテキトーなのかしら。
アタシはそれらの本は持ってないけれど、松坂和夫先生著の「数学読本」っていう
中高生向けのシリーズの最初の1巻くらいを中学の時に読んだ気がするわ。
内容がしっかりしていて、ちゃんと勉強したら中高の数学がばっちりになりそうな雰囲気の本だったわ。
おそらくだけど、その「集合・位相入門」の本では
自然数を集合として定義していないんじゃないかしら。
アタシの理解では、自然数を例えば>>439みたいに集合として定義するなら0から始めるのがほぼ必然となるけれど
自然数を他のもので定義されない原始的な存在として扱うなら
0から始まっていなくても別に困らないんだと思うわ。
集合論や論理学で自然数が0を含むことが多いのは、
単に自然数を集合で定義することの結果にすぎないと思うわ。
アタシは逆にWikiの「数論では0を入れない方が好まれる」っていうのが
単なる伝統なのかそれとも数学的な理由があるのかが気になるわね。

>>447
スレチで他の人たちが興味ないこといっぱい書いちゃってごめんなさいね。
Pirahãの話題を出したかったけど
誤った呼称が広まるのをアタシが手伝うことになるのが嫌だったからw
説明書いてたら例によって長くなっちゃったの。

あなたの読んでた本がどうだったのかはわからないけれど
一般的にはフランス語→英語の翻訳だと言語構造が近いから、誤訳ってそんなに起きないと思うのよね。
ほぼ単語変換で意味が通じることが多そうだし。
だから英語版がおかしいなら、フランス語の原著がすでにおかしかった可能性もあったかもしれないわね。
やはりヨーロッパ語→日本語みたいに、全然似てない言語間の翻訳の場合の方が
誤訳が生じる可能性は圧倒的に高いと思うわ。
それに、例えば日本語話者が仏語に熟達するのは
英語話者が仏語に熟達するより遥かに難しいから
きちんとした翻訳をできる人の絶対数って
言語の組み合わせごとにかなり違うと思うのよね。

でも翻訳書の方が、原著の誤りまで訂正していて
原著よりもむしろ良いとかもたまにあるらしいから
翻訳書の出来って、ほんと翻訳者次第で天と地なんだと思うわ。
0450Usagi2023/12/14(木) 20:40:21.80ID:rfcoGIyP0
>>448
あら消えちゃったのね。こういう問題だったの↓

複素数 z, w は z^2 + w^2 = 1 を満たすとする。

(1) z ≠ 0 であり, z の偏角をαとするとき,
z は 0 < α < π/4 を満たして動くとする。
w がとりうる範囲を複素数平面上に図示せよ。

(2) z は |z| = 1 を満たして動くとする。
w の実部, 虚部のとりうる値の最大値をそれぞれ求めよ。
また, w がとりうる範囲を複素数平面上に図示せよ。
0451陽気な名無しさん2023/12/19(火) 11:16:53.12ID:9V4IuOIs0
>>449
「数論では0を入れない方が好まれる」というのは、
例えば幾つかの(正の)自然数の最小公倍数を考える時、
自然数に0が入っていたら最小公倍数は常に0になるじゃない。
数論だと最小公倍数や最大公約数なんかをよく考えるから、
そういう時に自然数に0が入っていると邪魔なんだと思うわ。
0が入っていなくても、そういう時に特に支障を来すこともないし。
だからではないかしら。

翻訳については、フランス語→英語では、言語的な誤訳というより、
記号や添え字などのミスプリ?転写ミス?みたいのがやたらあったわ。
英語版はGTMっていう、やたら沢山数学書を出しているシリーズなんだけど、
やたら沢山出している分やっつけ仕事的な部分も多少あったのかもしれないわね。
原著の誤りまで訂正している優れた翻訳書は、日本語訳に多い印象だわ。
少しの事例しか見ていないのだけれど。

>>450
ありがとう。
見た感じ解答は>>445 でよさそうね。
陰伏曲線という言葉は初めて聞いたわ。陰関数ならよく聞いたけど。
要するに陰関数のグラフのことよね。
とりあえず陰伏曲線のwikiに接線の方程式が出てたけど、
xの最大値を求めたいときは、接線の傾きが無限大(分母F_yが0)の時、
yの最大値を求めたいときは、接線の傾きが0(分子F_xが0)の時を考えればいいんでしょ。

アタシも解析は苦手だったから詳しくはわからないんだけど、
とりあえず偏微分F_xは、yを定数とみなしてxの関数として微分する、
偏微分F_yは、xを定数とみなしてyの関数として微分する、
で良かったと記憶しているわ。

これで計算すると
F_yが0の時は、F_y=4x^2y+4y^3+4y=0つまりy(x^2+y^2+1)=0で、
これを満たすのはy=0のときのみ。
このときx^2(x^2-2)=0だからx=0またはx=±√2となり、xの最大値√2,最小値-√2がわかるわ。
F_xが0の時は、F_x=4x^3+4y^2x-4x=0 つまりx(x^2+y^2-1)=0で、
これを満たすのはx=0またはx^2+y^2=1のとき。
ここから最初の4次式と連立させて解くんだろうけど面倒だし、
とりあえずレムニスケートと単位円の交点であることがわかって、
うさぎの解答もこの条件を満たしているからうさぎの解答と同じ結果になるんだと思うわ。
0452Usagi2023/12/24(日) 02:02:20.42ID:+4htNgis0
>>451
>数論だと最小公倍数や最大公約数なんかをよく考えるから、
>そういう時に自然数に0が入っていると邪魔なんだと思うわ。
なるほど〜
小中学校で習うことって数の計算が主だから、それを考慮すると自然数に0を含まないと習うのも納得だわ。
あなたのレスを読んで、以前、順序集合についての本で読んだことを思い出したわ。
非負整数の集合に
 m ≼ n ⟺ km = n となる非負整数kが存在する
で二項関係 ≼ を定義すると、これは半順序になるわ。
任意の元 a, b に対して sup{a, b} と inf{a, b} が存在する半順序集合を束というわ。
上の例では sup{a, b} = lcm{a, b} そして inf{a, b} = gcd{a, b} となるから、これは束なの。
束の最大元や最小元がある時、それぞれをoneとzeroと呼ぶんだけど
するとこの例ではなんと、oneが0で、zeroが1となるの!

>記号や添え字などのミスプリ?転写ミス?みたいのがやたらあったわ。
それは誰でもチェックできる単純ミスだから酷すぎるわね!
GTMのSpringerは数学系に関しては一番信頼できる出版社だと勝手に思ってたから驚いたわ。

微分使ったやり方教えてくれてありがとう。
なるほどうまく計算できるのね。
Wikiも見たけどアタシ解析関係も勉強不足だから
なぜ接線の傾きが -F_x/F_y となるのか分からないので消化不良だけど…
0453陽気な名無しさん2023/12/30(土) 11:34:41.16ID:FhKxFjlC0
最後の書き込みから一週間近くなってくると、
このスレ落ちないかしら、って心配になってくるわね。

supとかinfとかって、上限下限の意味だったと思うけど、
上限下限って概念、勝手に全順序集合でのイメージでわかった気になっていたわ。
束ってググってみてやっとわかったけど、こんな概念あったのね。知らなかったわ。
って、集合・位相のテキスト見返してみたら、
上限下限は集合の部分に説明があったけど、
「完備束」とかいうものの説明が位相の方にあったわ。
それであとがきで完備束より緩い概念で束ってのがあるけど、
これの説明をすると集合・位相の入門書の範囲を超えるから、とか何とか。
アタシ集合の方はちゃんと読んだけど、位相の方は読んでなかったのよね。
いろいろしらべてうさぎの出した例がやっと違和感なく思えるようになったけど、「
>するとこの例ではなんと、oneが0で、zeroが1となるの!
こんなことが起こらないように(なのかな?)、テキストの例では、
集合{2,3,4,5,・・・}に対してうさぎの定義した順序を入れてたわ。

>Springerは数学系に関しては一番信頼できる
これはたぶんその通りだと思うんだけど、
それは数学書の大部分が英語で執筆されているからではないかしら。
Springerではないけど、洋書の日本語訳て結構丁寧なものが多い、
何なら原著にないことまで説明してくれていたりするイメージがあるわ。
だからラクに勉強しようと思ったら、日本語訳を読んで、
よくわからないところだけ英語版や、読めるなら原著を読むのが多分ラクよね。
勉強の範囲を超えて研究のレベルになったら原著でないと、ってことになるだろうけど。

>なぜ接線の傾きが -F_x/F_y となるのか
これについて、「うさぎでもわかる解析」とかいうシリーズがネット上にあったわ。
https://www.momoyama-usagi.com/entry/math-analysis18
このページがPart18 偏微分を用いた陰関数微分・陰関数定理ってなていて、
ちょうど問題となっている、正にその内容だと思うけど、
これの説明がちょっと形式的で、やっぱり不完全燃焼だわ。
だって例題、解説として載っているのが、変数分離タイプで、
xの式とyの式の積になっているような項があったらどうすんのよ、と思ったけど、
適当な例を考えて積の微分法で(高校で習った解き方)でやってみたら、
やっぱり-F_x/F_y になって、一応疑問は解消したわ。
うさぎ、あなた「うさぎでもわかるシリーズ」なんてあるのって、単なる偶然かしら?
かなり面白い偶然ね。
0454Usagi2023/12/31(日) 13:15:56.42ID:inxPlQ/W0
アタシが出した例がWikipediaにあったわ
https://upload.wikimedia.org/wikipedia/commons/thumb/e/e6/Infinite_lattice_of_divisors.svg/1280px-Infinite_lattice_of_divisors.svg.png
こういうのハッセ図っていうんだけど視覚的でとてもわかりやすいのよね

束では、任意の2つの要素からなる集合が上限と下限を持つことが保証されているけれど
このことから帰納法を使えば任意の有限集合が上限と下限を持つことが示せるわ
けれど、無限集合に対しても上限や下限が存在する保証はないのよ
有限集合だけじゃなく任意の無限集合も上限と下限を持つことが保証されているのが完備束よ

ふつう数学で上限・下限が初登場するのは実数の集合の話だと思うけど
実数の間の大小関係は全順序であって
全順序集合では、有限集合の上限と下限が単にその集合の最大限と最小限になるから
上限・下限の意味がかえって理解しにくいの
全順序集合で上限・下限が最大限・最小限と異なる場合を考えるには
無限集合を考える必要があって、
これが実数の集合の場合に考えられるのは実数の集合が完備束だからなのよね
そういうわけで一般的な数学のカリキュラムって
基本概念である有限集合の上限・下限の意義がいまいち納得できない状態で
無限集合の上限・下限のことを考えさせられることになるから
とても理不尽で理解しにくいと思うのよ
上限・下限の意義は半順序集合で考えてこそ理解できるものだと思うわ
その特殊な場合として全順序集合の場合があるだけだから

でもWikipediaにorder theoryの日本語版のページもないくらいだから
順序集合の理論って相当人気ないみたいよねw
アタシもともとこれを勉強してたら例として正規部分群が出てきて
それであなたに正規部分群のことを聞いたのが去年(2022年)だったのよ
具体的にいうと、ある群Gの正規部分群全体のつくる集合は
集合の包含関係⊆によって半順序集合となるけど
実はこれは束になっていて、Gの任意の正規部分群H, K に対して
sup{H, K} = { hk | h ∈ H かつ k ∈ K }
inf{H, K} = H ∩ K
となるわ(これらが正規部分群であることは簡単に確認できるわ)

群Gの部分群全体のつくる集合に⊆を入れたものも束になるけど、その場合は
sup{H, K} = (H ∪ K が生成するGの部分群)
inf{H, K} = H ∩ K
となるわ
アタシ去年は位数12以下の群に対して
その部分群がつくる束をハッセ図に描いて楽しんでたのよw
この子はどんなハッセ図になるのかしらってワクワクしながらやってたわ
0455Usagi2024/01/01(月) 01:44:10.85ID:AIEEN8X10
あけおめ!

束と完備束の間の概念としてσ完備束 (σ-complete lattice) っていうのもあるわ
これは任意の可算集合が上限と下限を持つ束のことよ
つまり非可算集合に対しては上限や下限が存在する保証がないってことね

測度論で可測空間って出てくるじゃない?
可測空間の定義や用語ってちょっとわかりにくことになっていると思うんだけど
英語だとある集合Xとその上のσ-algebraのペアとして定義していることが多いと思うの
σ-algebraのalgebraというのは、実はBoolean algebra(ブール代数 = ブール束)の意味なの
X上のσ-algebraは、Xの冪集合の部分集合で、Xを元に持ち、補集合をとる演算と
可算個の集合に対して和集合や共通部分をとる演算について閉じているものになるわ
和集合が上限、共通部分が下限になるから、これはσ完備束なの

別の流儀だと可測空間は集合Xとその上のσ-ringのペアとして定義されるんだけど
σ-ringはXを元として持つことを要求されないところがσ-algebraとの違いらしいわ

で、どうやらσ-completeのσはこのσ-ring、σ-algebraから来ているみたいで
これはFσ集合のσと同じでフランス語の somme(和)を表していて、
σ-ring、σ-algebraが可算個の集合の和集合をとる演算について閉じている、
ということから来ているらしいわ

またわけわからん長文書いちゃったけど
シグマの由来が気になって調べたことの備忘録だから気にしないでw

ネットで調べてみると日本語では可測空間がXとX上の「完全加法族」のペアとして
定義されていることが結構あるみたいね。
「完全加法族」は completely additive class の訳らしいんだけど
これはどうやら σ-ring の(おそらく古くさくてマイナーな)別名みたいよ
https://encyclopediaofmath.org/wiki/Additive_class_of_sets
個人的には適切な用語とは思えないわね
「完全」じゃ「可算」までの話だってことがわからないし
加法性も、この集合族自体の話ではなく、それに対して定義される測度の話だと思うから


「うさぎでもわかるシリーズ」、実はアタシもちょっと前に偶然見つけて
一瞬「え、アタシ煽られてる?!」って衝撃受けたんだけど
よく考えたらそんなわけないのよねw
見たけどやっぱり証明がよくわからないわ
陰関数定理でググるとヤコビアンがどうのとか出てきて
アタシそこまで勉強してないから、ちゃんと勉強しなきゃと思ったわ
この用語も衒学的なアテクシは気になったから調べてみたんだけど
Jacobian matrixの訳語でヤコビ行列とも言うみたいね
Jacobiはドイツ人だから「ヤコビ行例」は良いけど、
Jacobian は英語の形容詞だから、そんなに英語使いたいならジャコウビアンって言ってほしいわね
ドイツ語ではJacobischeらしいからヤコビッシェって言うならわかるけどw

話変わるけどPINKになってから同サロではスレが落ちなくなったみたいよね
0456Usagi2024/01/01(月) 22:27:31.35ID:AIEEN8X10
大地震起きたけどみなさんご無事かしら
アタシは関東だから影響なくて済んだわ

恥ずかしながら>>454でさらっと大嘘書いてたことに気づいたから訂正するわ
実数の集合は完備束じゃなかったわ
例えば上に有界でない実数の集合にはもちろん上限がないものね
有界な閉集合の任意の部分集合はその閉集合の中に上限や下限があるわよね?
だからこれは完備束
こういうのコンパクトとか言うんでしたかしら?
ここらへんもアタシちゃんと勉強できてないから勉強しなくちゃだわ
0457陽気な名無しさん2024/01/24(水) 08:14:21.43ID:MoTO07CM0
二月になったらこのスレも2024版が新しく経つのかしら
0458陽気な名無しさん2024/01/26(金) 19:26:46.59ID:srJtQFFe0
立て直すにしてもウサギとウサギの同意人形がスレを私物化できないようルールを決めてからの方がよさそう
0459陽気な名無しさん2024/01/27(土) 07:12:29.73ID:BMw4kDF90
大学入試の数学の問題に完全に限定すればいいのよ
0460陽気な名無しさん2024/02/01(木) 20:54:42.30ID:1SlIB51A0
2月になったわね。
次スレはスレタイ通りのスレになるかしら?
0469Usagi2024/03/04(月) 00:54:30.64ID:qYyY1O6E0
>>468
>>463の問題よね? 簡単なの? 
あたしは解けないから教えて欲しいわ
0470陽気な名無しさん2024/03/06(水) 13:37:34.52ID:AA3KRUx50
とりあえず(1)だけなら簡単よ。
n=1,-3,-7 でしょ。
0471陽気な名無しさん2024/03/06(水) 14:05:19.75ID:AA3KRUx50
(2)も、4個以下であることまではすぐにわかるわね。
3個以下であることはちょっと大変そうだわ。
0472Usagi2024/03/06(水) 21:56:15.17ID:loblAJ060
そうね、アタシも考えたところまでとりあえず書いてみるわ
g(n) = p が素数だとすると p = n(n^2 + an + b) なので n = 1, -1, p, -p の可能性しかないわね
だから g(n) が素数となるとしたら
(イ)g(1) = a + b + 1 が素数である
(ロ)g(-1) = a - b - 1 が素数である
(ハ)g(p) = pとなる素数pがある ⟺ x^2 + ax + b - 1 = 0 が素数解を持つ
(ニ)g(-p) = pとなる素数pがある ⟺ x^2 - ax + b + 1 = 0 が素数解を持つ
の4つの可能性のどれかね

アタシは(ハ)と(ニ)の2次方程式の素数解は合計で高々2個ってことまではわかったわ
ふたつの2次方程式のうち整数解を持つのがひとつ以下なら、これは明らかに成り立つわ
ではどちらの2次方程式も整数解を持つとしましょう
すると(ハ)の判別式D_1 = a^2 - 4b + 4 と(ニ)の判別式D_2 = a^2 - 4b - 4 が両方とも平方数になるわ
なので、ある非負整数 j, k に対して j^2 = a^2 - 4b + 4 かつ k^2 = a^2 - 4b - 4 となるわ
j^2 - k^2 = 8 だけど、もし j = k + 1 なら j^2 - k^2 は奇数になるから、この可能性はないわ
だから j ≥ k + 2 だけど、もし k ≥ 2 なら
j^2 - k^2 = (j - k)(j + k) ≥ 2[(k+2)+k] = 4k + 4 ≥ 4⋅2 + 4 = 12 > 8
だから k = 0 か k = 1 となるけど、k = 0 なら j^2 = 8 となってjが整数でなくなるから、k = 1, j = 3の可能性しかないわ
つまり
D_1 = a^2 - 4b + 4 = 3^2 = 9
D_2 = a^2 - 4b - 4 = 1^2 = 1
ね。b = (a^2 - 5)/4 が整数だからaは奇数ね
そして(ハ)と(ニ)の2次方程式の解は、それぞれ (-a±3)/2 と (a±1)/2 になるわ
・a ≤ -1 なら、 (a±1)/2はどちらも0以下だから素数ではない
・a = 1 なら、(ハ)と(ニ)の2次方程式の解は、1, -2, 1, 0 となってこの中に素数はない
・a ≥ 3 なら、 (-a±3)/2はどちらも0以下だから素数ではない
したがってどの場合でも (-a±3)/2 と (a±1)/2のうち素数は合計で高々2個ね

ここまで書くだけでもかなり面倒くさくて時間がかかったわ
でもこの問題を解くには(イ)と(ロ)が両方成り立つ場合に(ハ)と(ニ)の2次方程式の素数解が合計で高々1個であることを示す必要があって、それが難しいわ…
0473陽気な名無しさん2024/03/07(木) 12:02:03.18ID:Spv5Sd+N0
>>472
あなたの考え方で導かれる結果は、
@(ハ)と(ニ)の2次方程式の素数解は合計で高々2個
だけではなく、さらに
A(ハ)と(ニ)の2次方程式が同時に素数解を持つことはない
ことまで言えていて、というか、まずAが言えてそこから@が導かれるのよね。
Aのほうを472の結果としておいた方が、そこから先やることが絞れそうよ。

(イ)と(ロ)が両方成り立つ場合、
a + b + 1 ≧ 2、a - b - 1 ≧ 2 が同時に成り立つから、
これを図示すれば、a ≧ 2 であることがすぐにわかるわ。
だからこの時、(ハ)の二次関数の軸は負になるので、
もし(ハ)の2次方程式に素数解があるとしても1つだけしかありえない。
そしてこの時Aより(ニ)の2次方程式には素数解はないことがわかる。

だから、示すべきことは、
(イ)と(ロ)が両方成り立つ場合に(ニ)の2次方程式の素数解が高々1個であること
だけでいいことになるわ。
まあ、それでも難しいとは思うけど。
0474Usagi2024/03/07(木) 15:36:34.24ID:tKm16Krx0
>>473
なるほどね! おかげでわかったかも
(ニ)の2次方程式がふたつの素数解p, qを持つとすると根と係数の関係から
p + q = a, pq = b + 1
となって、p, q ≥ 2 であることから
a - b - 1 = p + q - pq = 1 - (p - 1)(q - 1) ≤ 1 - (2 - 1)(2 - 1) = 0
となって、これは素数になり得ないから(ロ)は成り立たないわ!

結局、素数ってことを使ってるのって最初の4つの場合分けの時だけで
その後は2以上の整数であるってことしか使わないのね…
場合分けが多くて複雑だしとても25分で解いて解答書けないと思うわw
それとももっと簡単にできるのかしら?
0475陽気な名無しさん2024/03/07(木) 15:52:38.71ID:Spv5Sd+N0
>>474
>(ニ)の2次方程式がふたつの素数解p, qを持つとすると
あたしもこれ考えてできたわ。
(ロ)は成り立たないことを言うのではなく
a - b - 1 ≧ 2 に反することを導いたんだけど、
多分同じくらいの手間でしょうね。
東大だしなにかもっとエレガントな解法があるのかもしれないわね。
0476陽気な名無しさん2024/03/07(木) 15:54:36.23ID:Spv5Sd+N0
って、
(ロ)は成り立たないこと と a - b - 1 ≧ 2 に反すること って、同じことか!
0477陽気な名無しさん2024/03/07(木) 18:25:17.95ID:bFeS51t50
これ答案読む先生も大変ね…
0478Usagi2024/03/07(木) 23:49:40.13ID:tKm16Krx0
ところで>>466の東工大の5番の問題、ちょっと出題ミスじゃないかしら?
前スレの842さんが話題にしてくれたのと同じで、この問題文2通りに読めるわよね?
「あるnがあって、すべてのαに対して…」なのか「すべてのαに対して、あるnがあって…」なのか
この場合結果的にはどちらでも答えは変わらないと思うけど
このせいでどれだけ受験生が戸惑ったり時間を無駄にしたりしたかと思うと許せないわね!

ていうか問題自体なんか引っ掛けっぽくない?
f(x)の根のひとつをα、その共役複素数をα*とすると
f(α*) = (α*)^2 + aα* + b = (α^2)* + (aα)* + b* = (α^2 + aα + b)* = (f(α))* = 0* = 0
だからα*もf(x)の根になるわ

f(x)が実数でない根αを持つならα^n = 1となる正の整数nがあることからαの絶対値は1で
α = cosθ + i sinθ, sinθ≠ 0 と表せて、もうひとつの根はα* = cosθ - i sinθになるわ
根と係数の関係から a = -(α+α*) = -2cosθだけど、これが整数で sinθ≠ 0 であることから、cosθは0か1/2か-1/2ね
したがって
α, α* = ± i で (a, b) = (0, 1) となるか
α, α* = 1/2 ± i √3/2 で (a, b) = (-1, 1) となるか
α, α* = -1/2 ± i √3/2 で (a, b) = (1, 1) となるかね

けれどf(x)の根がすべて実数の場合は、根が1か-1かであればいいから
f(x) = (x-1)^2 で (a, b) = (-2, 1)
f(x) = (x-1)(x+1) で (a, b) = (0, -1)
f(x) = (x+1)^2 で (a, b) = (2, 1)
の3つの可能性があるわ

だからこの6つの組が答えよね

でも問題文の「複素数の範囲の」を読んで「実数でない」と読んでしまう人結構いそうじゃないかしら?
そう読んでしまうと答えは「 (0, 1), (-1, 1), (1, 1), および a^2 - 4b ≥ 0 を満たすすべての整数の組(a, b)」になるわね!
(でも読み間違った人でこう解答した人ってほとんどいなそう…)

勘違いされないような問題文にすることはできるはずなのに、これじゃ数学の問題っていうより国語の問題よ!
0479陽気な名無しさん2024/03/09(土) 11:05:38.81ID:sjcf/xa90
東工大ってその程度の学校なのよ
学生も教授もショボちん
この問題も有名問題の劣化版だし
0480Usagi2024/03/09(土) 14:57:27.82ID:TDVzPs0X0
あら、有名問題ってどんなのかしら?
この問題は聞かれていることを理解できれば、あとは簡単だけど
東工大って国語の試験がないから、数学が国語の試験を兼ねているのかしら?とか思ったわw
いうても東大京大に次ぐポジションだと思うけど
最近は万個枠をつくったり、医科歯科と合体して東京科学大学とかいうマンガに出てきそうな大学名になるとか面白いわよね
0481Usagi2024/03/09(土) 15:03:46.70ID:TDVzPs0X0
思ったけど性同一性障害の人は万個枠で受験できるはずよね
お茶の水女子大も性同一性障害の人が受験できるんだものね
性別ごとの枠があるのって結局は平等の名を掲げた差別じゃないかしら
オカマ枠も作りなさいよね!
0482陽気な名無しさん2024/03/12(火) 00:51:19.22ID:g6ge3Tcw0
ならビアン枠も作れって?
性的なことだけ枠を作るのは不公平だから、他の特性、
例えば障害者枠も作れとか、血液型Rhマイナス枠も作れとか、
言い出したらキリがないわ
そもそも○○枠とかいう枠なんて存在すべきではないと思うわ

ってか、スレチよ
0483陽気な名無しさん2024/03/12(火) 12:24:32.76ID:jmyCh1Bf0
p,q,r,sはいずれも絶対値が1の複素数であるが、
p+q+r+s,
pq+pr+ps+qr+qs+rs,
pqr+pqs+prs+qrs,
pqrs
はみな整数である。
p^n=1
をみたす正の整数nを何かひとつ求めなさい。
0484陽気な名無しさん2024/03/12(火) 12:25:38.37ID:jmyCh1Bf0
東工大が元ネタにしたであろう命題から無理やり問題こさえたけど、
東工大ならこれくらいでもいいんじゃない!?
問題として成立してなかったらごめん遊ばせ
0485陽気な名無しさん2024/03/13(水) 02:35:37.38ID:sXyH6jL+0
>>483
これを満たすp,q,r,sはいく通りかの選び方があるけど、
選び方によってnの候補もいろいろよね。
最小の正の整数nとは言ってないから、
選び方によらず成り立つようなものを答えればいいのかしら?
どんなに大きくてもいいのよね。
あるnが正解なら、そのnの正の整数倍もすべて正解になるものね。
0486陽気な名無しさん2024/03/13(水) 18:53:29.30ID:dvtPxuhH0
そうよ
こういうときどんなに大きくてもいいと聞いてすぐ1兆とか1京とか言い出すのって中高一貫男子校出身者の特徴なんですって
0487陽気な名無しさん2024/03/13(水) 20:45:16.34ID:sXyH6jL+0
>選び方によらず成り立つようなもの
だとしたら6の倍数でなきゃだから、
>1兆とか1京とか
は不適よねぇ。
0489Usagi2024/03/14(木) 03:19:03.47ID:73oD7prG0
ひゃだ!
この問題面白すぎてアタシ大興奮よ
入試に出たら大学受験生に解くのはたぶん無理よ
どのpでもp^n=1となる最小のnは120だと思うわ!
アタシは力技で解いたけど、きっともっと本質的でエレガントな解法があるに違いないわ
誰かこの問題の神秘を解き明かして!

>>486
そんな特徴いったいどこで聞いたのよw
0490陽気な名無しさん2024/03/14(木) 09:47:12.83ID:B2WNhsl50
>>489
120ってどうやって求めたの?
特に素因数5がなぜ必要なのか全然わからないわ
教えて!
0492Usagi2024/03/15(金) 22:13:39.00ID:QByfmoco0
>>490
まず力技で高校生的に解いたやり方を説明するわ
p, q, r, sは、x^4の係数が1で他の係数が整数のある4次式f(x)の根になるわよね
>>478の考え方から、もし実数でない根α = cosθ + i sinθがあるなら、その共役複素数α* = cosθ - i sinθも根となって
f(x)は (x-α)(x-α*) = x^2 - (α+α*)x + αα* = x^2 - 2 cosθx + 1 で割り切れるわね
そう考えると次の3つの可能性があるわ

(イ)f(x) = (x ± 1)(x ± 1)(x ± 1)(x ± 1)
(ロ)f(x) = (x ± 1)(x ± 1)(x^2 + cx + 1) ここでcは整数
(ハ)f(x) = (x^2 + ax + 1)(x^2 + bx + 1) 

(ロ)のx^2 + cx + 1の根は、478に書いた6つ(1の原始3乗根, 原始4乗根, 原始6乗根)のどれかになるわね

(ハ)の場合は、展開すると
f(x) = x^4 + (a+b)x^3 + (ab+2)x^2 + (a+b)x + 1
でこの係数が全て整数であることから
u = a+b
v = ab
が共に整数となるわね。a = - 2cosθだったことを思い出すと、-2 < a, b < 2 だから
方程式 g(t) = t^2 - ut + v = 0 が -2 < t < 2 となる実数根を持つ条件を考えると
D = u^2 - 4v ≥ 0 ⟺ v ≤ (1/4)u^2
軸: -2 < u/2 < 2 ⟺ -4 < u < 4
g(-2) > 0 ⟺ v > -2u - 4
g(2) > 0 ⟺ v > 2u - 4
これをuv平面に図示してみると、その領域内にある格子点は
(u, v) = (±1, 0), (0, 0), (0, -1), (0, -2), (0, -3), (-1, -1), (1, -1)
となるの。
0493Usagi2024/03/15(金) 22:14:07.11ID:QByfmoco0
・(u, v) = (±1, 0) の場合
a, b は t^2 ± t = 0 の根だから a, b = 0, ±1 で
f(x) = (x^2 + 1)(x^2 ± x + 1) だから、その根は478で出たものの中にあるわ

・(u, v) = (0, 0) の場合
a, b は t^2 = 0 の根だから a = b = 0 で
f(x) = (x^2 + 1)^2 だから、その根は478で出たものの中にあるわ

・(u, v) = (0, -1) の場合
a, b は t^2 - 1 = 0 の根だから a, b = ±1 で
f(x) = (x^2 - x + 1)(x^2 + x + 1) だから、その根は478で出たものの中にあるわ

・(u, v) = (0, -2) の場合
a, b は t^2 - 2 = 0 の根だから a, b = ±√2 で
f(x) = (x^2 - √2x + 1)(x^2 + √2x + 1)
これを解くと x = (±1±i)/√2 = e^{±πi/4}, e^{±3πi/4} で、これらは1の原始8乗根よ

・(u, v) = (0, -3) の場合
a, b は t^2 - 3 = 0 の根だから a, b = ±√3 で
f(x) = (x^2 - √3x + 1)(x^2 + √3x + 1)
これを解くと x = (±√3±i)/2 = e^{±πi/6}, e^{±5πi/6} で、これらは1の原始12乗根よ

・(u, v) = (-1, -1) の場合
a, b は t^2 + t - 1 = 0 の根だから a, b = (-1±√5)/2 で
f(x) = (x^2 - {(-1+√5)/2} x + 1)(x^2 - {(-1-√5)/2} x + 1) を解くと
x = {-1+√5 ± i √(10+2√5)}/4 = e^{±2πi/5} または
x = {-1-√5 ± i √(10-2√5)}/4 = e^{±4πi/5} で、これらは1の原始5乗根

・(u, v) = (1, -1) の場合
a, b は t^2 - t - 1 = 0 の根だから a, b = (1±√5)/2 で
f(x) = (x^2 - {(1+√5)/2} x + 1)(x^2 - {(1-√5)/2} x + 1) を解くと
x = {1+√5 ± i √(10-2√5)}/4 = e^{±πi/5} または
x = {1-√5 ± i √(10+2√5)}/4 = e^{±3πi/5} で、これらは1の原始10乗根

最後のふたつの場合については、もちろんこんな値覚えていないからWikiを参照したわw
https://en.wikipedia.org/wiki/Exact_trigonometric_values

というわけで p の可能性としては、1の原始1, 2, 3, 4, 5, 6, 8, 10, 12乗根の可能性があるから、その最小公倍数の120が答えよ
0494Usagi2024/03/15(金) 22:23:43.36ID:QByfmoco0
それで、考えたら秘密の一端が分かったかもしれないから書いてみるわ

ある自然数mについて、1の原始m乗根の数はφ(m)個になるわ
ここでφはオイラーの関数で、φ(m)はm以下でmと互いに素な自然数の数よ
1の原始m乗根φ(m)個の全てを根に持つモニック多項式Φ_m(x)は円分多項式といって、整数係数になることが知られているわ
もしφ(m) ≤ 4 なら f(x) = Φ_m(x) (x-1)^{4 - φ(m)} とおけば
これは整数係数の4次式だから、1の原始m乗根は問題のpでありうるわね

逆にφ(m) > 4 ならば、1の原始m乗根は整数係数の4次式f(x)の根ではありえないわ
なぜなら、Φ_m(x)は1の原始m乗根の最小多項式であることが知られているので
もしf(x)が1の原始m乗根を根に持つなら、5次以上のΦ_m(x)が4次のf(x)を割り切らなければいけないことになって矛盾するわ

つまり、もし問題の条件を満たすpが1のm乗根のものに限られるなら話が早いのよ
求める答えは LCM = { m | φ(m) ≤ 4 } となるわ
ここで右の集合が有限になるのか気になるところだけど、調べたらオイラーの関数には下界があるそうなの
一番詳しいのはlogが入り組んだ複雑なやつがあるみたいだけど、簡単なのだと
φ(m) ≥ √(m/2)
ってのがあるわ。
https://arxiv.org/pdf/0806.2068.pdf
もう少し便利なのだと、m ≠ 2, 6 なら
φ(m) ≥ √m
だって。
https://artofproblemsolving.com/community/c1610044h2360114_problem_331
φ(2)とφ(6)はわかるから、これを使えば、m > 16 なら φ(m) ≥ √m > √16 = 4 となるからφ(16)までを調べれば十分なの
結局 LCM { m | φ(m) ≤ 4 } = LCM {1, 2, 3, 4, 5, 6, 8, 10, 12} = 120 となるわ

ただ、うさぎがわからないのは、問題の条件を満たすpは1の整数乗根に限られるとすぐわかるのかということ
ここを誰か教えてくれないかしら?
0495Usagi2024/03/15(金) 22:35:21.13ID:QByfmoco0
(誤) 求める答えは LCM = { m | φ(m) ≤ 4 } となるわ
(正) 求める答えは n = LCM { m | φ(m) ≤ 4 } となるわ
0496陽気な名無しさん2024/03/16(土) 10:25:51.04ID:Bw1V77PI0
>>492-495
さすがによく考えてるわね

懐かしいわね
昔φ(n)→∞(n→∞)やったわよね
あのときもガミガミ言われたのよねアタシ
せっかくスレを盛り上げようとしたのに
なんの悪意もなかったのよ?
ただ、盛り上げようとした だけ だったのに
PTSD発症したわよ
0498陽気な名無しさん2024/03/22(金) 21:01:28.74ID:VH1Q8cNv0
受験雑誌読んできたわ
このスレで扱った東大の問題は超難問、東工大の問題は難問で、どちらも良問の判定だった
アタシ東大のそんなに難しくないと思ったんだけど受験報告読むと殆ど出来てないわねえ
素数を小さい方からp[1],p[2],…,p[n],…とするとき
n≧12⇒p[n]>3n
を示すのが阪大にあるんだけどこれも超難問と判定されてたわ
これは良問にはなってなかった
0499Usagi2024/03/23(土) 02:27:52.07ID:V0Fuiiih0
>>497
あら、出典教えてくれてありがとう
証明を見たけどわからないところがあるのよね
まずαの冪の既約多項式(irreducible polynomials)って書かれているけど、これはαの冪の最小多項式って意味でいいのかしら
こういう言葉遣いもするの?そうだとして話をすすめるけど
αの最小多項式の根をα, β, γ, …などとすると、α^k, β^k, γ^k, …を根とする多項式の係数は
根と係数の関係と対称式の基本定理を使えばαの最小多項式の係数から作れるから整数になるのよね
その既約成分を取り出せばα^kの最小多項式が作れて、その係数は整数になるってことよね
で、こういう係数の取りうる値は有界であって、その範囲はαのℚ上の次数に応じて決まるって書かれていると思うけど、ここがわからないの
なんで有界なの?誰か教えてくれると嬉しいわ
0500Usagi2024/03/23(土) 03:09:03.52ID:V0Fuiiih0
>>498
個人的にはあんまり納得いかない評価ね
東大のは、あたしには解けないくらい難しかったのは確かね
でも複雑なだけで美しさとかは感じないわ。素数っていうことは最初の部分でしか使わないし。
入試問題としては部分点を上げやすいとかそういうのが良いところなのかもしれないけど
東工大のは国語的にひどいから絶対に良問じゃないわ

阪大の問題は面白いわね
これを解くには、6で割った余りが0, 2, 3, 4の数は2か3で割り切れるから
2, 3以外の素数は6で割ると必ず1か5余るってことに気づくのがポイントね
5以降のどの素数についても、そのふたつ先の素数は必ず6以上大きくなることになるから
n ≥ 12が偶数なら p[n] ≥ p[12] + 6×(n-12)/2 = 37 + 3n - 36 = 3n + 1 > 3n
n ≥ 12が奇数なら p[n] ≥ p[13] + 6×(n-13)/2 = 41 + 3n - 39 = 3n + 2 > 3n
となるわね
この問題はスッキリした解き味で美しさを感じられるわ
0501陽気な名無しさん2024/03/23(土) 14:26:14.53ID:Mi1uN1rQ0
α[1],α[2],α[3],…,α[n]はみな絶対値が1なので、n変数のk次基本対称式P[k](x[1],…,x[n])に代入したときに、その絶対値がnCkで抑えられる
(したがってnC0,nC1,…,nCnのうち最大のものをとればαの次数nだけで決まる量によって|P[k](α[1],…,α[n])|が評価できる)
そして鳩ノ巣原理により1の冪根になると

上の問題について具体的に書けば
p,q,r,sはいずれも絶対値が1の複素数で
p+q+r+s,
pq+pr+ps+qr+qs+rs,
pqr+pqs+prs+qrs,
pqrs
はみな整数のとき、これらはある整数係数多項式の根である
それらの冪乗p^n,q^n,r^n,s^nを根とする4次多項式も整数係数である
その整数係数に現れる整数には実は制約があってp,q,r,sは絶対値が1であることから
|p^n+q^n+r^n+s^n|≦4
|p^nq^n+p^nr^n+p^ns^n+q^nr^n+q^ns^n+r^ns^n|≦6
|p^nq^r^n+p^nq^ns^n+p^nr^ns^n+q^nr^ns^n|≦4
|p^nq^nr^ns^n|≦1
となる範囲にしかその整数は存在しない
つまり、どのようなnについても、p^n,q^n,r^n,s^n を根とする整数係数多項式は必ず
x^4-4x^3-6x^2-4x-1
から
x^4+4x^3+6x^2+4x+1
まで
のうちのどれかになる(こういう書き方するか知らんけど)
0502Usagi2024/03/24(日) 02:48:58.55ID:uH/BWTOw0
なるほどね!
とてもわかりやすいわ。どうもありがとう
そうすると、この本の証明みたいに話を既約多項式に限定する必要ないわよね
例えばpがℚ上4次なら (x - p^i)(x - q^i)(x - r^i)(x - s^i) を考えれば良くて、別にこれが既約じゃなくても関係ないものね
作れる4次式が有限個しかないから、当然 p^i を根に持つ既約な因数を取り出しても有限個しかないけれど。
因数分解した時の因数の係数がどうなるのかは、よくわからないわ?
一般的に、因数分解すると現れる係数の絶対値って小さくなるものかしら?
まあ有限個しかないから、いずれにぜよ有界なんだけど
この本の証明、なんか無駄にわかりにくい気がするわ
0504陽気な名無しさん2024/04/08(月) 21:22:49.24ID:bw9WTlLe0
>>503
見た感じ大学入試か模試か何かのように見えるけど、
そこまで難しくないように見えるけど、
それでも出典は書いておくべきではないかしら。
何か一言あるとなおいいと思うわ。
0505Usagi2024/04/12(金) 22:09:54.17ID:RVcs+8RO0
>>504
これは>>465で出ているのと同じだから、今年の阪大の問題なんじゃないかしら
こんなふうに、オイラーの関数そのものが出題されるなんて驚いたわ
難しくないの? あたしには相当難しい問題に思えるけど
だって(1)は中国の剰余定理を証明せよ、というのと同じことでしょ?
それを15分くらいで証明して解答を書き上げなきゃいけないわけよね
はっきり言ってほとんどの人には不可能じゃないかしら
(2)は別に難しくないけど…
なんか、良い問題を思いつかなかったから、大学数学を安易に流用しただけに見えるわ
こういうのは大学入試問題として非常に悪問だと思うわ
ていうか高校までで習うことで厳密に証明を書けるのかあたし疑問なんだけど
0508陽気な名無しさん2024/04/18(木) 20:11:42.34ID:bAdmkEHl0
>>507
アンタじゃ無理でしょ
うさ子の方が知能高いと思うわよ
0510陽気な名無しさん2024/04/28(日) 20:24:07.77ID:ZIe1gFBT0
オイラ関数
0511陽気な名無しさん 転載ダメ2024/04/28(日) 21:34:55.04ID:ZIe1gFBT0

Slot オイラー
🌸🌸💯
💯🎴🍜
🎴💰😜
(LA: 3.93, 3.83, 2.97)
0514陽気な名無しさん2024/04/29(月) 02:10:54.07ID:8wP4VKNt0
保守
0515Usagi2024/04/29(月) 03:44:46.48ID:XSUhhfGP0
みんな保守ありがとう

>>509難しかったわ
A_k = 農{i = 1}^k a_i, B_k = 農{i = 1}^k b_i と書くわね
0 < m < nである、あるmに対し A_m ≤ B_m が成り立つと仮定して矛盾を導くわね
もし b_m/a_m < 1 ならば、条件からすべての k ≤ m に対して b_k/a_k < 1、つまり a_k > b_k となり
A_m = 農{i = 1}^m a_i > 農{i = 1}^m b_i = B_m となって、A_m ≤ B_m に反するわ
したがって b_m/a_m ≥ 1となるけれど、条件からすべての k > m に対して b_k/a_k > 1、つまり a_k < b_k となり
A_n = A_m + 農{i = m+1}^n a_i < B_m + 農{i = m+1}^n b_i = B_n となり、条件 A_n = B_n に矛盾するわ
QED

これ、(A_k, B_k) を結んでいくと、原点と (A_n, B_n) = (A_n, A_n) を通る、だんだん傾きを増していく折線グラフになって
下に凸になるから、A_m > B_m は視覚的には明らかなんだけど、ちゃんと証明しようとすると難しかったわ
もっと簡単に書けるのかしら?
0516Usagi2024/04/29(月) 04:02:40.77ID:XSUhhfGP0
ひゃだ、アタシの専ブラだとちゃんと表示されてるんだけど
ブラウザで開くと、上の書き込みの総和記号のシグマが文字化けして農になっているわw
0520陽気な名無しさん2024/04/29(月) 22:58:40.80ID:gGVVqbld0
落ちそう
0521陽気な名無しさん2024/04/30(火) 11:29:28.02ID:PAzaop7E0
一瞬荒れてあっという間に整理されたわね
0523陽気な名無しさん2024/05/09(木) 08:35:32.84ID:aznnZFbN0
2003年京大前期の第四問
f(x)=(x^100+1)^100+(x^2+1)^100+1 は x^2+x+1で割り切れるか。
0524陽気な名無しさん2024/05/14(火) 16:02:59.46ID:SNNJKEQ50
>>522 って、特にどうということもない普通の話よね
0525陽気な名無しさん2024/05/30(木) 13:15:42.32ID:SX5upyuz0
>>523 誰も解けないの?
0526Usagi2024/06/01(土) 01:33:07.95ID:eL/W4aCN0
解けたけどアタシばっか解くのもどうかなって思って様子見てたの
それに>>515みたいにせっかく解いても誰も何のコメントもくれなかったりするし…

x^2+x+1 = 0 の根のひとつをωとすると ω^3 - 1 = (ω-1)(ω^2+ω+1) = 0 となるから ω^3 = 1 よ
そして、ω+1 = -ω^2 だから
(ω^100+1)^100 = ((ω^3)^33⋅ω+1)^100 = (1^33⋅ω+1)^100
= (ω+1)^100 = (-ω^2)^100 = ω^200 = (ω^3)^66⋅ω^2 = ω^2
また、ω^2+1 = -ω だから
(ω^2+1)^100 = (-ω)^100 = ω^100 = (ω^3)^33⋅ω = ω
したがって
f(ω) = (ω^100+1)^100 + (ω^2+1)^100 + 1 = ω^2 + ω + 1 = 0
ωは x^2+x+1 = 0 の任意の根だから、因数定理からf(x)は x^2+x+1 で割り切れるわ

これって1の3乗根をどれだけ見慣れてるかってだけで、後はただのパズルよね
こんな問題で数学的思考力が測れるのか、甚だ疑問に思うわ

ついでに書くと>>509は画像検索したら「やさしい理系数学」って本にある問題だとわかったの
本屋で見たら解答が3つ載っていて、三番目のはアタシが515の終わりに書いたようなものだったわ
アタシはそれしか思いつかなくて、でもそんなんで答えとしてOKなのかしらと思って
がんばって言語化してみたのが最初に書いたものなの
だから無駄があったわね
本の一番目の解答が基本これと一緒だけど、背理法使ってなかったわ

(1) b_m/a_m < 1の場合
すべての i ≤ m について b_i/a_i < b_m/a_m < 1 だから a_i > b_i となるので A_m > B_m が言える
(2) b_m/a_m ≥ 1の場合
すべての i ≥ m+1 について b_i/a_i > b_m/a_m ≥ 1 だから a_i < b_i
したがって 農{i = m+1}^n a_i < 農{i = m+1}^n b_i なので
A_m = A_n - 農{i = m+1}^n a_i > B_n - 農{i = m+1}^n b_i = B_m が言える

ってやれば良かったのね。この問題の方がずっと思考力が要るわ
0527Usagi2024/06/01(土) 02:30:15.91ID:eL/W4aCN0
>>513
(1)の式の一般化が成り立つと仮定すれば、条件を満たすのは 2^a 3^b (a ≥ 1, b ≥ 0) の形のものに限られるから
答えは、8, 16, 32, 64, 6, 12, 24, 48, 96, 18, 36, 72, 54 になるわ
(なんで5以上に限定するのかしら。1, 2, 4をうっかり含めたら減点ってこと? 意味不明ね)

まず n = 2^a ならf(n)がnの約数となるのはすぐ確認できるわ。
nが2以外の素因数pを持つ場合は、(1)の式の一般化が成り立つとすると p-1はf(n)の因数となるから
f(n)がnの約数なら、p-1がnの因数でもあることになるわ。
p-1は偶数だから、nも偶数となって n = 2^a⋅m(a ≥ 1、mは奇数)と書けるわ。
ここでpが5以上の素数ならf(n)はnの約数とならないことを示すわ。
この場合、p-1 = 2k とすると 2 ≤ k < p となるわ。
もしkが素因数2を持つなら、p-1は4で割れるからf(n)は2^{a+1}を因数に持つことになり、nの約数にならない。
もしkが2以外の素因数qを持つなら、nもqを素因数に持つことになり、q < p だからqとpは異なる素数なので
f(n)は(q-1)(p-1)で割り切れ、q-1もp-1も偶数だから、この場合もf(n)は2^{a+1}を因数に持ち、nの約数にならない。
だからnが2以外の素因数を持つなら3しかないの。

こんなのちゃんと書いてたらいくら時間があっても足りないと思うんだけど、答えだけ求めればいいのかしら?
(1)の一般化だって証明していないし
というか(1)の模範解答が気になるわ
きっと(1)を解けた受験者はほぼ0人よね
0528Usagi2024/06/01(土) 02:34:50.25ID:eL/W4aCN0
あ、526の農はシグマの文字化けよ
ついでに大学入試出典じゃないけど問題出していいかしら
任意の正の整数について、4で割ると1余る約数の個数は、4で割ると3余る約数の個数以上であることを示せ
0529陽気な名無しさん2024/06/02(日) 08:58:04.88ID:2A1uzBXw0
nは奇数で考えればいい。
f(n)=nの約数dのうち4で割ると1余るdの個数マイナス4で割ると3余るdの個数
とする。g(d)=1(d≡1 mod4), -1(d≡3 mod4)とすると、
f(n)=Σ[dはnの約数を動くことを以後d|nと書く]g(d)
となる。gは奇数c,dに対してg(cd)=g(c)g(d)が成り立つ。
このようなもののΣ[d|n]という和もまたそのような性質をもつらしい。
(参考 https://mathnote.info/entry/2020/04/05/Dirichlet-convolution-product)
つまり、互いに素な奇数m,nに対してf(mn)=f(m)f(n)となる。
知らんけど。
奇素数d≡1 mod4に対してf(d^k)=k+1、
奇素数d≡3 mod4に対してf(d^k)=0,1,0,1,…≧0
0530Usagi2024/06/02(日) 13:08:34.45ID:k3zXdr+70
あら、すぐに解かれちゃったわ
姐さんさすがね!
アタシの解き方も基本同じだけど、アタシはディリクレの畳み込みというものは知らなかったからお陰で勉強になったわ
アタシは
a(n) = 4で割ると1余るnの約数の個数
b(n) = 4で割ると3余るnの約数の個数
とおいたの。(姐さんの f は f(n) = a(n) - b(n) となるわ)
奇素nについて、nの異なる素因数の個数に関する帰納法で a(n) - b(n) ≥ 0 を示したわ
n = m⋅p^k(pはmの因数でない素数)とすると
a(n) = a(m⋅p^k) = a(m)a(p^k) + b(m)b(p^k)
b(n) = b(m⋅p^k) = a(m)b(p^k) + b(m)a(p^k)
となるのはすぐわかるから
a(n) - b(n) = a(m⋅p^k) - b(m⋅p^k)
= a(m)a(p^k) + b(m)b(p^k) - (a(m)b(p^k) + b(m)a(p^k))
= (a(m) - b(m))(a(p^k) - b(p^k))
≥ 0⋅0 = 0
となるわ。姐さんの書き方だと f(m⋅p^k) = f(m)f(p^k) ということね

関係無いけど姐さんのシグマは専ブラでもふつうのブラウザでもシグマになってるのね
試しに姐さんのΣをコピペしてみるわ
0531Usagi2024/06/02(日) 13:09:30.47ID:k3zXdr+70
あ、今度はちゃんとΣになったわ! これからはこれをコピペさせてもらうわ
0532陽気な名無しさん2024/06/02(日) 14:17:04.02ID:2A1uzBXw0
>>509
a/c < b/d のとき (a+b)/(c+d) はそのあいだにある、
というだけじゃないのか…
その問題集の解答ではアタシの硬化した脳細胞には染み込んでこないわ…
0533Usagi2024/06/02(日) 14:45:27.32ID:k3zXdr+70
>>532
たしか二番目の解答がそれを使ってた気がするわ
0534陽気な名無しさん2024/06/02(日) 15:54:29.12ID:2A1uzBXw0
小学校でやることだものね
薄い食塩水と濃い食塩水を混ぜるとどうなりますか?って
0535Usagi2024/06/02(日) 20:06:58.22ID:k3zXdr+70
そう言われたらひどく簡単な問題に思えてきたわw
0536陽気な名無しさん2024/06/05(水) 09:54:10.08ID:tTQNFaI60
a[1]〜a[n]は全て正の整数
x[k]は0以上a[k]以下の整数
x[1]+x[2]+…+x[n]が偶数になる確率は1/2以上
とゆうことか
簡単に示せるのかな?
0537Usagi2024/06/08(土) 17:57:25.45ID:yOPEuQNF0
>>536
何これ、独り言? それとも問題なの? 問題としても設定がよくわからないわ
0538陽気な名無しさん2024/06/10(月) 20:10:05.87ID:1bPQVA2G0
>>528よね
奇数を素因数分解したときの素因数≡3(mod4)の部分をp[1]^a[1]p[2]^a[2]…p[n]^a[n]とする
約数≡1(mod4)となるのはp[1]^x[1]p[2]^x[2]…p[n]^x[n]のx[1]+x[2]+…+x[n]が偶数になるとき
アタシの解き方>>529より筋が良い気がするわ
0539Usagi2024/06/12(水) 00:42:09.93ID:K0wL7A7u0
>>538
なるほど、そういう意味だったのね。解読ありがとう。
nに関する帰納法で考えれば示せるけれど、そうすると>>530と同じことになっちゃうわよね
それ以上に簡単にできるのかってことだけど、どうなのかしら
ちなみに、a[1], …, a[n]の中にひとつでも奇数があれば、x[1]+x[2]+…+x[n]が偶数になる場合の数と奇数になる場合の数は等しくなるわね
だからこの和が偶数になる場合の数が奇数になる場合の数のより多いのって、a[1], …, a[n]が全て偶数の時だけになるわね
0540陽気な名無しさん2024/06/18(火) 23:28:37.95ID:k06agafe0
任意の正の整数について、7で割ると1か2か4余る約数の個数は、7で割ると3か5か6余る約数の個数以上であることを示せ
0541陽気な名無しさん2024/06/19(水) 07:52:36.92ID:/RjfW53g0
>>540 が解けたわけではないけれど、
>>528>>540 とを見ると、
「任意の正の整数について、nで割ると(mod n で平方数)余る約数の個数は、nで割ると(mod n で平方数でない数)余る約数の個数以上である」
が一般に言えるのかしら。
平方剰余の相互法則とかと何か関係あるのかしらね。
0542陽気な名無しさん2024/06/19(水) 07:56:44.23ID:/RjfW53g0
mod n でってより、(ℤ/nℤ)*の平方数、非平方数って言うべきだったかしらね。
nと互いに素なもの限定にしたいんだからこっちの方が良さそうな気がするわね。
0543陽気な名無しさん2024/06/19(水) 08:36:16.97ID:k2IaGVwa0
大学受験でおなじみ、赤本(1954年創刊)の
「表紙が」リニューアル

きっかけは出版社の若手社員のひとこと
「表紙文字の大きさなどにプレッシャーを感じる」「威圧感がある」
現役高校生らにもヒアリングの上、変更に。

旧赤本を使って大学に合格した、関西大学の大学生らは
「新しい方が親しみ感じるかな」
「古い方がいい。このプレッシャー感が受験のやる気になる」
https://news.yahoo.co.jp/articles/af5cd5ec7ee96a4e7bb8e4c834f405a897cd2243

こんな事にまで過敏にならないとダメなのね。アホみたい。
これで来年以降の受験生のレベルが下がったら面白いわね
0546陽気な名無しさん2024/06/22(土) 09:17:12.63ID:48o23CXd0
すごいね
0547陽気な名無しさん2024/06/22(土) 21:10:31.78ID:wut9sDxW0
面白い入試問題見つけたわ

s[n,k](x[1],x[2],…,x[n])をn変数k次基本対称式とする
s[n,0](x[1],x[2],…,x[n])=1
s[3,2](x,y,z)=xy+yz+zx
など

数列{a[n]}は
0≦a[n]≦1
lim[n→∞]a[n]=1
をみたす

数列{b[n]}を
b[n]=Σ[k=0→n] (-1)^k s[n,k](a[1],a[2],…,a[n])/(k+1)
とする

lim[n→∞] n*b[n] を求めよ
0548Usagi2024/06/23(日) 13:40:45.89ID:PmCwvjon0
>>540は面白いわね!
基本的に>>544の姐さんの言うとおりだと思うわ
{1, 2, 4} が (ℤ/7ℤ)* = {1, 2, 3, 4, 5, 6} の正規部分群になっていて、商群を考えると
(ℤ/7ℤ)*/{1, 2, 4} = {{1, 2, 4}, {3, 5, 6}} ≅ (ℤ/4ℤ)* = {1, 3}
という同型があるから全く同じことなのね
つまり、「7で割ると」を省略して書くと
・1か2か4余る素数の冪はすべて1か2か4余る
・3か5か6余る素数の冪には、1か2か4余るものと3か5か6余るものが交互に現れる
そして
・1か2か4余る整数同士、また3か5か6余る整数同士をかけると、1か2か4余る整数になる
・1か2か4余る整数と3か5か6余る整数をかけると、3か5か6余る整数になる
だから>>530
a(n) = 7で割ると1か2か4余る約数の個数
b(n) = 7で割ると3か5か6余る約数の個数
とすれば全く同じに解けるわ

平方剰余の相互法則と何か関係があるのかはわからないけど
0549Usagi2024/06/23(日) 13:56:58.89ID:PmCwvjon0
>>547
これは難しいわ
ちくりんスレ(京大銭湯サークルスレ)の328で出された京大院試より難しいわ!と思ったけど
もしかして似た方法で解けるのかしら? 答えは1になる予感がするわ
0550陽気な名無しさん2024/06/24(月) 21:10:28.93ID:hILDDZO00
なるほど…
lim[n→∞] n ∫[0→π/4] (tanθ)^2n dθ
も求められるわけか
0552Usagi2024/06/26(水) 01:34:18.79ID:28tGxFHC0
>>551 すごいわ、使わせてもらうわ
>>550
そうね。x = tanθとおけば dx/dθ= 1/cos²θ = 1+tan²θ= 1+x² だから置換積分すると
n ∫[0→π/4] tan²ⁿθ dθ = n ∫[0→1] x²ⁿ/(1+x²) dx
となるわ。f(x) = 1/(1+x²) とすると、これは閉区間 [0, 1] で定義されている連続関数で f(1) = 1/2 だから
京大入試の問題の解き方を使うとこの極限は 1/4 になるわね
0553Usagi2024/06/26(水) 02:02:05.45ID:28tGxFHC0
>>547については
f[n](x) = Σ[k=0→n] (-1)^k s[n,k](a[1],a[2],…,a[n]) x^k
とおくと b[n] = ∫[0→1] f[n](x) dx となるから、京大の問題に似てるなと思ったのよ

lim[n→∞] a[n] = 1 なので a[k] = 0 となるkは存在するとしても有限個だけど
f[n](x) = Π[k=1→n] (1-a[k]x) なので、もし a[k] = 0 なら、f[n]の値に影響しないわ
なぜ a[k] > 0 という設定ではなく a[k] ≥ 0 という設定なのか解せないけれど
要するに常に a[k] > 0 という設定でも問題は変わらないわね
そうすると、f[n](x) はy切片が1で x = 1/a[k] (k = 1, 2, …, n) を根に持つn次式ということになるわ
1/a[k] ≥ 1 だから 0 ≤ x ≤ 1 のときは、f[n](x)のグラフは単調減少で、0 ≤ f[n](x) ≤ 1 となるわ
これを0から1まで積分したものにnを掛けたものの極限を求めろってことね

というわけで問題の意味は理解したんだけど、これすごく難しいんじゃないかしら
収束することを示したいんだけど、京大の問題みたいに簡単にできないわ
ていうか本当に収束するのかしら?
これ本当に大学入試なの? 大学入試で基本対称式なんて言葉使わないと思うんだけど
一部を書き換えたの?
もし大学入試なら何か誘導があって上手く解けるのかしら
0554陽気な名無しさん ぴっかぴっかぴっかぴっかぴっかぴっかぴっかぴっかちゅー2024/06/26(水) 19:04:23.83ID:CbEXrae20
>>553
>これを0から1まで積分したものにnを掛けたものの極限を求めろってことね

そうよ

>収束することを示したいんだけど、京大の問題みたいに簡単にできないわ

アタシは京大の方が難しいと思うわ
アタシの感覚では、ね

>ていうか本当に収束するのかしら?

それが収束するのよ!

>これ本当に大学入試なの? 大学入試で基本対称式なんて言葉使わないと思うんだけど
>一部を書き換えたの?

ええ、書き換えたわよ?

>もし大学入試なら何か誘導があって上手く解けるのかしら

ええ、おっしゃる通りよ?
上手い誘導がついてたわ
そのせいで自分で考えるところがほぼない問題と化してたわ

>というわけで問題の意味は理解したんだけど、これすごく難しいんじゃないかしら

そこそこ難しいと思うわ
なぜならアタシが書き換えた上に誘導も取り払ってるからね
でも難易度は京大の院試とどっこいどっこいな気がするわ
0555陽気な名無しさん2024/06/26(水) 20:23:37.85ID:G27wHTZo0
微積の教科書の真ん中あたりにありそうな話よね
きっと1-a[k]x≦exp(-a[k]x)とするんでしょうけど
でもこのまま続けると高校では習わないようなことを使わなければならないような…アタシの勘違い?
ほんとに大学入試の問題なの!?

京大の院試、アタシはパッと見
Weierstrassの多項式近似を使えばf(x)は多項式Σa[k]x^kと見做せばよくて
n∫Σa[k]x^(k+2n)dx=Σa[k]*n/(k+2n+1)→Σa[k]/2=f(1)/2
と思ったんだけどUsagiの軽やかな解き方は流石よね

普通の精神状態で>>552の1/(1+x^2)をまさか多項式で近似しようなんて思わないから
抽象的に考えることの大切さを痛感するわ
0557Usagi2024/06/27(木) 22:25:22.14ID:ni//TpJ60
できたかも。。

553に書いたように、f[n](x) = Π[k=1→n] (1-a[k]x) とおくわ
以下、0 ≤ x ≤ 1 の範囲で考えるわ
各kに対して、0 ≤ 1-x ≤ 1-a[k]x が成り立つから f[n](x) ≥ (1-x)^n
したがって
n b[n] = n ∫[0→1] f[n](x) dx ≥ n ∫[0→1] (1-x)^n dx = n/(n+1) → 1 (n→ ∞)

次に A[n] = (1/n)Σ[k=1→n] a[n] とおくと、0 ≤ A[n] ≤ 1 ね
lim[n→∞] a[n] = 1 なので、あるところから先のnに対しては常に A[n] ≠ 0 となるから、以下ではそういうnを考えるわ
1-a[1], …, 1-a[n] はすべて非負だから、n個の数の相加相乗平均の不等式から
f[n](x) = Π[k=1→n] (1-a[k]x) ≤ [ (1/n)Σ[k=1→n] (1-a[k]x) ]^n = (1-A[n]x)^n
したがって
n b[n] = n ∫[0→1] f[n](x) dx ≤ n ∫[0→1] (1-A[n]x)^n dx = [n/(n+1)] ⋅ [ {1 - (1-A[n])^(n+1)}/A[n] ]
n→ ∞ のとき A[n] → 1 であり (1-A[n])^(n+1) ≤ 1-A[n] → 0 となることから、上の式の右辺は → 1

以上から、はさみうちの原理で n b[n] → 1 よ

これで大丈夫かしら?
とはいえ、n個の数の相加相乗平均の不等式が成り立つことと、
lim[n→∞] a[n] = 1なら lim[n→∞] A[n] = 1 であることを示す必要はあるけれど

>>556
素敵ね。アタシも欲しいわ!
0559陽気な名無しさん2024/06/28(金) 09:04:02.35ID:/AHs3aED0
元の問題とは微妙に変えてるのね
∪sagi賢いわねぇ
0560陽気な名無しさん2024/07/10(水) 15:11:50.69ID:3CMp+IHY0
難しいわ
0561陽気な名無しさん2024/07/13(土) 13:42:27.19ID:xiaDZvgu0
難しいわ
0562Usagi2024/07/14(日) 02:07:25.07ID:mJMFks/c0
誰も問題出さないから、アタシが難しいけどここの人が好きそうな問題出しちゃおうかしら
pを素数、nを正の整数とするとき lim[n→∞] {Σ[i=0→n] (pn)C(pi)}/2^(pn) を求めて!
Cは二項係数よ
大学入試じゃないから、高校数学で解けるのかはわからないわ
0563陽気な名無しさん2024/07/14(日) 16:04:12.80ID:8XLvrrhQ0
pΣ[k=0→n](pn)C(pk)
=Σ[j=0→pn](pn)Cj χ(j) (jがpの倍数のときχ(j)=p, jがpの倍数ではないときχ(j)=0)
=Σ[j=0→pn](pn)Cj Σ[k=0→p-1]ζ^(kj) (ζ=e^(2πi/p))
=Σ[k=0→p-1]Σ[j=0→pn](pn)Cj (ζ^k)^j
=Σ[k=0→p-1](1+ζ^k)^(pn)

k≠0のとき(1+ζ^k)^(pn)/2^(pn)→0なので
(1/2^(pn))Σ[k=0→p-1](1+ζ^k)^(pn)→1

∴Σ[k=0→n](pn)C(pk)/2^(pn)→1/p
0564Usagi2024/07/15(月) 06:22:40.31ID:Bwo0eR5r0
あら〜お見事!
けっこう難しいと思ったのにすぐ解かれちゃったわ
アタシも同じやり方で解いたけど、姐さんの簡潔で全く無駄のない書き方には感心するわ
アタシが書くともっと言葉で説明しちゃうのよね
0565陽気な名無しさん2024/07/15(月) 08:43:27.30ID:EceMiIJl0
ブタの貯金箱割ったとき表向きの硬貨の枚数がpの倍数になる確率ってこと?
0566陽気な名無しさん2024/07/15(月) 14:39:49.98ID:bACtxwxX0
>>564
簡潔で全く無駄のない書き方は、わかっている人が見れば感心するんだけど、
わかっていない人が見ると読解するのが大変なのよ。骨が折れるのよ。
だからうさぎ姐さんのように言葉で説明してくれるとありがたい人は多いと思うわ。
あたしも含めて。
うさぎ姐さん、言葉で説明してくださらないかしら?
0567陽気な名無しさん2024/07/15(月) 21:50:09.76ID:vPLsje2b0
むりよ
0568Usagi2024/07/17(水) 00:48:44.34ID:YLUJmRmP0
>>565
あら、確かにそういうふうに理解できるわね!
上では硬貨の枚数をpnとしてpの倍数の場合を考えていたけれど
姐さんのおっしゃる通り、その場合に限定する必要はないわよね
より一般的に書くとこうね
lim[m→∞] (Σ[0 ≤ j ≤ m かつ jはpの倍数] mCj)/2^m = 1/p

>>566
いうてもそれほど変わらないけど、じゃあすぐ上に書いたものを補足説明しながら示すわ
ζ = e^(2πi/p) = cos(2π/p) + i sin(2π/p) とおくと
ζ^0, ζ^1, …, ζ^(p-1) が1のp個のp乗根になるわ。二項定理から
(1+ζ^k)^m = Σ[j=0→m] mCj 1^(m-j) (ζ^k)^j = Σ[j=0→m] mCj ζ^(kj)
となるけれど、両辺の k = 0 から k = p-1 までの総和をとると
(★) Σ[k=0→p-1] (1+ζ^k)^m = Σ[j=0→m] mCj [ζ^0 + ζ^j + ζ^(2j) + … + ζ^{(p-1)j}]
となるわ。ここでjについてふたつの場合に分けて考えるわ。
(イ) jがpの倍数の場合
どのkに対しても ζ^(kj) = 1 だから
ζ^0 + ζ^j + ζ^(2j) + … + ζ^{(p-1)j} = p
となるわ
(ロ)jがpの倍数でない場合
このとき集合として
{ ζ^0, ζ^j, ζ^(2j), …, ζ^{(p-1)j} } = { ζ^0, ζ^1, …, ζ^(p-1) }
となるわ。なぜなら、もし0 ≤ k < l ≤ p-1 で ζ^(kj) = ζ^(lj) ならば ζ^{(l-k)j} = 1 となって
ζが1のp乗根であることから(l-k)jがpで割り切れなければいけないけれど
1 ≤ l-k ≤ p-1 でpは素数だからjがpで割り切れることになり、jがpの倍数でないという仮定に反するから。
というわけで
ζ^0 + ζ^j + ζ^(2j) + … + ζ^{(p-1)j} = ζ^0 + ζ^1 + … + ζ^(p-1) = (ζ^p - 1)/(ζ-1) = 0/(ζ-1) = 0
となるわ。
(イ)と(ロ)から(★)は
Σ[k=0→p-1] (1+ζ^k)^m = pΣ[0 ≤ j ≤ m かつ jはpの倍数] mCj
となるから、
(Σ[0 ≤ j ≤ m かつ jはpの倍数] mCj)/2^m = (1/p)Σ[k=0→p-1] (1+ζ^k)^m/2^m
となるわ。
k ≠ 0 のときζ^kは実数でないから、複素数平面上で、原点と1と1+ζ^kの三点は三角形を作るわよね
三辺の長さを考えると三角不等式から |1+ζ^k| < |1| + |ζ^k| = 1 + 1 = 2 なので
|(1+ζ^k)^m/2^m| = |(1+ζ^k)/2|^m → 0 (m → ∞) となるから (1+ζ^k)^m/2^m → 0 なの
だから k = 0 の場合だけが残って結局1/pになるの

複素数列の収束の話になるから高校数学の範囲ではないと思うけど、複素数を使わないでも解けるのかしら?
でも565の姐さんの言い換えを見ると、1/pに収束するのって当たり前っぽそうな気もするけどそうなのかしら?
なんなら、表向きの硬貨の枚数がpで割って1余る確率とか、2余る確率とか、どれも1/pに収束するのかしら?
そんな気がするけど示せるかしら?
それに、pが素数の場合だけ確率が1/pになるとしたら不自然よね
でよく考えてみたら、pが素数でなくても1/pになるんじゃないかと思うの
これを示すには、pが素数でなくても、jがpの倍数でないときは必ず
ζ^0 + ζ^j + ζ^(2j) + … + ζ^{(p-1)j} = 0
となることを示せばいいわね。誰か証明してくれないかしら?
また難しい代数の話とかになっちゃうのかしらw
0569陽気な名無しさん2024/07/17(水) 01:44:35.00ID:8J76QXQN0
等比数列の和というだけじゃないの?
ζ^0 + ζ^j + ζ^(2j) + … + ζ^{(p-1)j}
=((ζ^j)^p-1)/(ζ^j-1)
0570Usagi2024/07/17(水) 02:00:43.78ID:YLUJmRmP0
あら!難しいこと考えなくても最初からそうすれば良かったわね。素晴らしいわ!
0571陽気な名無しさん2024/07/17(水) 08:03:22.14ID:1CN8sey50
pで割って1余る確率、2余る確率、3余る確率…
どれが一番大きいのかしら?
0572陽気な名無しさん2024/07/17(水) 19:13:53.81ID:JM3eIFwi0
>>568
とてもわかりやすいわ。ありがとう。
0573Usagi2024/07/18(木) 00:36:36.01ID:VBvYn4X30
とりあえず思いついたことメモしとくわ
pを素数とは限らない2以上の整数として
P_m(r) = m枚の硬貨が入ったブタの貯金箱を割ったとき表向きの硬貨の枚数がpで割ってr余る確率
とするわ
硬貨に1からmまで番号が振ってあるとして、表向きの硬貨の枚数がpで割って1余る場合は
・硬貨1が表で、残りのm-1枚のうち表向きの硬貨の枚数がpで割り切れる場合
・硬貨1が裏、硬貨2が表で、残りのm-2枚のうち表向きの硬貨の枚数がpで割り切れる場合
・硬貨1と硬貨2が裏、硬貨3が表で、残りのm-3枚のうち表向きの硬貨の枚数がpで割り切れる場合
 :
というふうに分けられるわ。
硬貨1から硬貨(k-1)までが裏で硬貨kが表の確率は1/2^kで
残りの(m-k)枚のうち表向きの硬貨の枚数がpで割り切れる確率はP_(m-k)(0)だから
P_m(1) = Σ[k=1→m] (1/2^k) P_(m-k)(0)
となると思うの。まだちゃんと示したわけじゃないけど
m → ∞ のとき P_m(0) → 1/p であることを考えると P_m(1) → 1/p になりそうよ
ならば同様に P_m(2) = Σ[k=1→m] (1/2^k) P_(m-k)(1) などとなるから、すべて 1/p に収束するんじゃないかしら

こんな複雑な計算しなくても当たり前に1/pだって言えるのかしら
0574陽気な名無しさん2024/07/18(木) 01:07:36.88ID:ybHJ3nlU0
アタシもまだ詳しく考えてないけど、3で割って1余るところの和って
(1+1)^3n=3nC0+3nC1+3nC2+…
ω^2(1+ω)^3n=3nC0ω^2+3nC1+3nC2ω^ 4+…
ω(1+ω^2)^3n=3nC0ω+3nC1+3nC2ω^5+…
を足したらいいのかしら?え、違う?どうなのよ?言ってみなさいよ

とすると一般にpで割ってr余るところの和も{a[k]}(k=0,1,…p-1)という(rに依って異なるかもしれないがしかしどのrに対しても必ずa[0]=0である)数列があって
Σ[k=0→p-1] ζ^a[k] (1+ζ^k)^(pn)
と書けるのではないかしら?
0575陽気な名無しさん2024/07/18(木) 01:49:02.30ID:7S/Jchxm0
たんにa[k]=-krとするだけ…?
そんな上手くはいかないかしら
ごめんなさい、もう寝ます
0576Usagi2024/07/18(木) 13:52:33.27ID:slDvVOYr0
>>574
ひゃー、姐さんの数学センス素晴らしいわ〜、よくそんなこと思いついたわね!
>>575
あなたのおっしゃる通りよ、ていうかこの方針で解けるとしたらそれしか可能性ないもの。
実際、a[k] = -kr としたら
Σ[k=0→p-1] ζ^a[k] (1+ζ^k)^m
= Σ[j=0→m] mCj [ζ^(0+a[1]) + ζ^(j+a[1]) + ζ^(2j+a[2]) + … + ζ^{(p-1)j+a[p-1]}]
= Σ[j=0→m] mCj [ζ^0 + ζ^(j-r) + ζ^(2j-2r) + … + ζ^{(p-1)j-(p-1)r}]
= Σ[j=0→m] mCj [1 + ζ^(j-r) + (ζ^(j-r))^2 + … + (ζ^(j-r))^(p-1)]
ここで j-r ≡ 0 (mod p) なら ζ^(j-r) = 1 だから
1 + ζ^(j-r) + (ζ^(j-r))^2 + … + (ζ^(j-r))^(p-1) = p
そして j-r ≢ 0 (mod p) ならば、ζ^(j-r) ≠ 1 だから等比数列の和で
1 + ζ^(j-r) + (ζ^(j-r))^2 + … + (ζ^(j-r))^(p-1) = [(ζ^(j-r))^p - 1)]/[ζ^(j-r) - 1]
= [(ζ^p)^(j-r) - 1)]/[ζ^(j-r) - 1] = (1^(j-r) - 1)/[ζ^(j-r) - 1] = 0
となるから、pで割ってr余るjのところだけ残るわ!
0577陽気な名無しさん2024/07/18(木) 20:19:11.14ID:1XNouwCy0
なるほど…恐ろしく上手くいくわね
どうもありがとう

それはそうとアタシ>>573のスマートなやり方に俄然興味湧いてきたわ
要するに{P[m]}がm→∞で収束するとき
P[1]/2^(m-1)+P[2]/2^(m-2)+P[3]/2^(m-3)+…+P[m-1]/2
がどこに収束するか?ってことよね?
見た目は完全に大学一年生の夏学期の問題だけど、
ε-N論法覚えたばかりのチーチーパッパでも解けるのかしら?
それとも色々と微妙に高度なこと知ってないと難しい?
Usagiがどこまで考えてるのか教えてほしいわん
0578Usagi2024/07/19(金) 13:56:54.11ID:PcX9eeqk0
正確にはそれに P[0]/2^m も加えたものね
これは最初の(m-1)枚が全部裏でm枚目が表の場合に対応するわ。P[0]も定義されているから問題ないわ。
チーチーパッパのレベルは知らないけど、高度なことは必要ないと思うわ
一般的に {P[m]} がAに収束する数列だとして考えてみましょう。
εを任意の正数とするわ。
{P[m]} は収束するから有界なので、ある正数Bがあってすべてのmに対して |P[m]| < B となるわ。
ここで正の整数Kを 2B/2^K < ε/3 となるようにとるわ。
P[m] → A なので、ある正の整数Mがあって m ≥ M ならば |P[m] - A| < ε/3 となるわ。
今、N = M + K とおくわ。m ≥ N を任意にとると
|Σ[i=1→m] P[m-i]/2^i - A(1 - 1/2^m)|
= |Σ[i=1→m] P[m-i]/2^i - AΣ[i=1→m] 1/2^i|
= |Σ[i=1→m] (P[m-i]-A)/2^i|
≤ Σ[i=1→m] |(P[m-i]-A)/2^i|
となるけど、i = 1, …, K については m-i ≥ M なので |P[m-i] - A| < ε/3 だから
Σ[i=1→K] |(P[m-i]-A)/2^i| < (ε/3) Σ[i=1→K] 1/2^i < ε/3
そして |A| ≤ B だから |P[m-i]-A| ≤ |P[m-i]| + |A| < 2B なので
Σ[i=K+1→m] |(P[m-i]-A)/2^i| < 2BΣ[i=K+1→m] 1/2^i < 2B/2^K < ε/3
以上から
|Σ[i=1→m] P[m-i]/2^i - A(1 - 1/2^m)| < (ε/3) + (ε/3)
そして |A| ≤ B かつ m > K なので |A/2^m| < 2B/2^K < ε/3 なので
|Σ[i=1→m] P[m-i]/2^i - A|
= |Σ[i=1→m] P[m-i]/2^i - A(1 - 1/2^m) - A/2^m|
≤ |Σ[i=1→m] P[m-i]/2^i - A(1 - 1/2^m)| + |A/2^m|
< (ε/3) + (ε/3) + (ε/3) = ε
まとめると、どんなに小さな正数εに対しても、ある自然数Nがあって
m ≥ N ならば |Σ[i=1→m] P[m-i]/2^i - A| < ε
となって、これは Σ[i=1→m] P[m-i]/2^i がAに収束することを意味するわ。
0579陽気な名無しさん2024/07/19(金) 18:18:14.85ID:cKSSx9C/0
まあそうよね
数列{a[n]}と数列{b[n]}から新しい数列{c[n]}を
c[n]=Σ[k=1→n-1]a[n-k]b[k]
と作ると{c[n]}にはなにか知られた性質があるのかしらと想像をたくましくしてしまったわ

1/pとは言わないまでも収束することが簡単に示せたら素晴らしいのにね
そこがやや瑕瑾かしら
0580陽気な名無しさん2024/07/22(月) 20:13:42.29ID:zgnqJHus0
a(n),b(n)を>>530のものとする
lim[n→∞](Σ[k=1→n]a(k) - Σ[k=1→n]b(k))/n
を求めよ
0581陽気な名無しさん2024/07/22(月) 20:18:17.11ID:zgnqJHus0
実数列{a[n]},{b[n]}はn→∞で
a[n]→a, b[n]→b
に収束するものとする
{c[n]}を>>579のものとする
lim[n→∞]c[n]/n
を求めよ
0582Usagi2024/07/28(日) 00:25:22.38ID:ffb4PU1T0
>>581は解けたと思うわ

補題 a[n] → a ならば (Σ[j=1→n] |a[j]-a|)/n → 0

証明
εを任意の正数とするわ
{a[n]}は収束するから有界だから、ある M > 0 があって任意のjに対して |a[j]| ≤ M となるわ
このとき |a| ≤ M となるので、|a[j]-a| ≤ |a[j]| + |a| ≤ 2M となるわ
a[n] → a なので、ある自然数Nがあって j > N なら |a[j]-a| < ε/2 となるわ
自然数nを n > N かつ n > 4MN/ε となるようにとると
Σ[j=1→n] |a[j]-a|
= Σ[j=1→N] |a[j]-a| + Σ[j=N+1→n] |a[j]-a|
< Σ[j=1→N] 2M + Σ[j=N+1→n] (ε/2)
= 2MN + (n-N)(ε/2)
< n(ε/2) + n(ε/2) = nε
したがって (Σ[j=1→n] |a[j]-a|)/n < ε
QED

ちなみにこれで>>557で仮定していた次のことを示したことになるわね
a[n] → a ならば (1/n)Σ[k=1→n] a[k] → a

なぜなら
|(1/n)Σ[k=1→n] a[k] - a| = |(1/n)Σ[k=1→n] (a[k] - a)|
≤ (Σ[k=1→n] |a[k] - a|)/n → 0 となるから
0583Usagi2024/07/28(日) 00:27:57.97ID:ffb4PU1T0
581に戻るわ
{a[n]}と{b[n]}はどちらも収束するから有界だから
ある M > 0 があって任意のnに対して |a[n]| ≤ M かつ |b[n]| ≤ M となるわ
このとき |a| ≤ M と |b| ≤ M も成り立つわね
εを任意の正数とするわ
a[n] → a かつ b[n] → b なので、補題から、ある自然数Nがあって、m ≥ N なら
 Σ[k=1→m] |a[k] - a| < mε/(2M) かつ Σ[k=1→m] |b[k] - b| < mε/(2M)
となるわ。もし n ≥ N+1 ならば
 |c[n] - (n-1)ab|
= |Σ[k=1→n-1] (a[n-k]b[k] - ab)|
≤ Σ[k=1→n-1] |a[n-k]b[k] - ab|
= Σ[k=1→n-1] |a[n-k]b[k] - ab[k] + ab[k] - ab|
= Σ[k=1→n-1] (|b[k]|⋅|a[n-k] - a| + |a|⋅|b[k] - b|)
≤ Σ[k=1→n-1] (M |a[n-k] - a| + M |b[k] - b|)
= M Σ[k=1→n-1] |a[n-k] - a| + M Σ[k=1→n-1] |b[k] - b|
= M Σ[k=1→n-1] |a[k] - a| + M Σ[k=1→n-1] |b[k] - b|
< M(n-1)ε/(2M) + M(n-1)ε/(2M)  
= (n-1)ε
したがって |c[n]/(n-1) - ab| < ε となるから、lim[n→∞] c[n]/(n-1) = ab なので
lim[n→∞] c[n]/n = lim[n→∞] (c[n]/(n-1))⋅((n-1)/n)
= (lim[n→∞] c[n]/(n-1))⋅(lim[n→∞] (n-1)/n) = ab⋅1 = ab

こんなんでいいかしら
c[n]の定義がΣ[k=1→n]なせいで無駄にややこしくなってる気がするわ
もっとサクッとできるの? 想定していた解法を解説して欲しいわね
>>580は難しそうよね。誰か解いてくれないかしら
0584陽気な名無しさん2024/07/28(日) 14:38:12.48ID:PbbhhG+U0
あら、お見事ね
一箇所等号でいいのかやや疑問があるけど、素晴らしいわ

Σ[k=1→n-1]|a[n-k]b[k]-ab|/n
の真ん中のほう(a[ ]b[ ]の番号がどちらも大きいとこ)だけ
(M+|a|)εで評価すれば補題はいらないかもしれないわね
n≧Nで|a[n]-a|<ε,|b[n]-b|<εとする
n≧N^2のとき真ん中のほうは
Σ[√n≦k≦n-√n]|a[n-k]b[k]-ab|/n
≦Σ(|b[k]||a[n-k]-a|+|a||b[k]-b|)/n
≦(M+|a|)ε
端のほうは
Σ[1≦k<√n]|a[n-k]b[k]-ab|/n≦(M^2+|ab|)/√n→0
Σ[n-√n<k<n]|a[n-k]b[k]-ab|/n≦(M^2+|ab|)/√n→0

もしもa[n]→aならばΣ[k=1→n]a[k]/n→aを使うなら
Σ[k=1→n-1]a[n-k]b[k]
=Σ[k=1→n-1](a[n-k]-a+a)b[k]
=Σ[k=1→n-1](a[n-k]-a)b[k]+aΣ[k=1→n-1]b[k]
と変形してもよさそうね

なんでΣ[k=1→n-1]なのかしらね
微積の授業の演習問題だったのよこれ
昔教わってた先生に聞くしか…
0586Usagi2024/07/28(日) 16:07:18.70ID:ffb4PU1T0
解説ありがとう、とても勉強になるわ
おっしゃる通り
= Σ[k=1→n-1] (|b[k]|⋅|a[n-k] - a| + |a|⋅|b[k] - b|)
の = は ≤ の間違いだったわ
確かに補題を使うなら姐さんの変形の方がエレガントね
まあ、あたしは |c[n] - (n-1)ab| を評価しようってとこからスタートして
評価が難しいなと思って補題を作ったからこういう流れになったのよね
0587陽気な名無しさん2024/07/28(日) 23:59:20.41ID:xxMBCIhI0
実はアタシは>>580の答えを用意しているわけではない
ただ、うさ子が持ってきた問題の背後にあるmathematical elegance を使えば解けるのではないか
と思っただけ
だから真剣に解かなくてもかまわないわよ
0588陽気な名無しさん がんばれ 日本!!2024/07/29(月) 08:44:21.76ID:q/B0BIxw0
普通に解こうとすると
Σ[1≤4k+1≤n] {n/(4k+1)}
をどうしたらいいかが難しいわよね
(注:{α}はαの小数部分)

そもそも
Σ[k=1→n] {n/k}
ってどう求めるの?
0589Usagi2024/07/29(月) 14:52:26.15ID:ezBU9wix0
Σ[1≤4k+1≤n] {n/(4k+1)} ってどうやって出て何を表している式なの? 説明して欲しいわ
0590陽気な名無しさん2024/07/29(月) 20:57:06.29ID:C53cbMwP0
Σ[k=1→n] a(k) = Σ[1≤4k+1≤n] [n/(4k+1)]
Σ[k=1→n] b(k) = Σ[1≤4k+3≤n] [n/(4k+3)]
よね?だから
Σ[k=1→n] a(k) - Σ[k=1→n] b(k)
= Σ[1≤4k+1≤n] [n/(4k+1)] - Σ[1≤4k+3≤n] [n/(4k+3)]
= [n/1] - [n/3] + [n/5] - [n/7] + [n/9] - … + [n/★]
= n(1 - 1/3 + 1/5 - 1/7 + 1/9 - … + 1/★)
   - Σ[1≤4k+1≤n] {n/(4k+1)} + Σ[1≤4k+3≤n] {n/(4k+3)}
             (∵ [α] = α - {α})
なのでもしも
(1/n) Σ[1≤4k+1≤n] {n/(4k+1)} - (1/n) Σ[1≤4k+3≤n] {n/(4k+3)}
→ 0 (n→∞)
がいえるなら求めたい極限は
1 - 1/3 + 1/5 - 1/7 + 1/9 - … → π/4
になる気がするわ

とすると
lim[n→∞] (1/n) Σ[1≤4k+1≤n] {n/(4k+1)}
lim[n→∞] (1/n) Σ[1≤4k+3≤n] {n/(4k+3)}
をどうするかが問題よね
0591陽気な名無しさん2024/07/30(火) 08:16:03.56ID:AS/D5cpK0
あ、簡単か
lim[n→∞] (1/n) Σ[1≤4k+1≤n] {n/(4k+1)}
= lim[n→∞] (1/n) Σ[1≤4k+3≤n] {n/(4k+3)}
= (1/4) ∫[0→1] {1/x} dx
でいいのかな?あん?
0592Usagi2024/07/31(水) 09:00:20.52ID:F9Svxacb0
>>590
なるほど、姐さんすごいわね。
1〜2行目は確かにそうね! 全然気づかなかったわ
1 - 1/3 + 1/5 - 1/7 + 1/9 - … → π/4
は条件収束だから
([n/1] - [n/3] + [n/5] - [n/7] + … + [n/★])/n
= (1 - 1/3 + 1/5 - 1/7 + … + 1/★)
 - Σ[1≤4k+1≤n] {n/(4k+1)}/n + Σ[1≤4k+3≤n] {n/(4k+3)}/n
の極限を取ったものも成り立つと言って大丈夫なのかちょっと疑問に思ったけど、よく考えたら大丈夫そうね

>>591はリーマン積分の定義を使っているのよね
f(x) = {1/x} は連続関数ではないけれど、不連続点は可算無限個だからリーマン積分が定義されるということでいいかしら?
0 ≤ {1/x} < 1 だから 0 ≤ ∫[0→1] {1/x} dx < 1 となるかしら?
具体的な値はわからないけれど。

ちなみにだけど>>588のΣ[k=1→n] {n/k} は n → ∞ のとき収束しないわね
nが素数の場合を考えると、k = 1, n のとき {n/k} = 0 で、それ以外のとき {n/k} ≥ 1/k だから
Σ[k=1→n] {n/k} ≥ 1/2 + 1/3 + … + 1/(n-1) → ∞ (n → ∞) となるわ
Σ[1≤4k+1≤n] {n/(4k+1)} も同様ね。(1/n) が掛かってないと収束しないのね。
0593陽気な名無しさん2024/07/31(水) 15:21:32.51ID:W2h10E2j0
>Σ[k=1→n] a(k) = Σ[1≤4k+1≤n] [n/(4k+1)]

ここkwskお願いします🥺
0594陽気な名無しさん2024/07/31(水) 18:14:43.07ID:hVxk+clU0
 ┌○┐
 │お|ハ,,ハ
 │断|゚ω゚ )  お断りします
 │り _| //
 └○┘ (⌒)
    し⌒
0595Usagi2024/08/01(木) 00:45:46.37ID:OEDliddA0
ちょっとw、そういう意地悪はやめましょうよ
アタシもこの式すぐには意味がわからなかったわよ?

>>593
例えば、Σ[k=1→10] a(k) は1から10までの数の、4で割ると1余るnの約数の個数の合計だけど
そういう約数は10以下だから、1, 5, 9 のどれかよね
1は1から10までのすべての数の約数として [10/1] = 10 回現れる
5は5と10の約数として [10/5] = 2 回現れる
9は9の約数として [10/9] = 1 回現れる
だから合計で [10/1] + [10/5] + [10/9] = 13個になるの
0597陽気な名無しさん2024/08/05(月) 15:35:31.67ID:0ygBF+My0
むりよ
■ このスレッドは過去ログ倉庫に格納されています

ニューススポーツなんでも実況